Sei sulla pagina 1di 66

NCLEX Questions 1 C.

see the health care provider immediately


1. Which individual is at greatest risk for D. visit the health care provider within one (1)
developing hypertension? week for a BP check
A. 45-year-old African American attorney
B. 60-year-old Asian American shop owner 7. The hospital has sounded the call for a
C. 40-year-old Caucasian nurse disaster drill on the evening shift. Which of these
D. 55-year-old Hispanic teacher clients would the nurse put first on the list to be
discharged in order to make a room available for
2. A child who ingested 15 maximum strength a new admission?
acetaminophen tablets 45 minutes ago is seen A. A middle-aged client with a history of being
in the emergency department. Which of these ventilator dependent for over seven (7) years
orders should the nurse do first? and admitted with bacterial pneumonia five days
A. Gastric lavage PRN ago.
B. Acetylcysteine (Mucomyst) for age per B. A young adult with diabetes mellitus Type 2
pharmacy for over ten (10) years and admitted with
C. Start an IV Dextrose 5% with 0.33% normal antibiotic-induced diarrhea 24 hours ago.
saline to keep vein open C. An elderly client with a history of
D. Activated charcoal per pharmacy hypertension, hypercholesterolemia, and lupus,
and was admitted with Stevens-Johnson
3. Which complication of cardiac catheterization syndrome that morning.
should the nurse monitor for in the initial 24
hours after the procedure? D. An adolescent with a positive HIV test and
A. angina at rest admitted for acute cellulitis of the lower leg 48
B. thrombus formation hours ago.
C. dizziness 8. A client has been newly diagnosed with
D. falling blood pressure hypothyroidism and will take levothyroxine
(Synthroid) 50 mcg/day by mouth. As part of the
4. A client is admitted to the emergency room teaching plan, the nurse emphasizes that this
with renal calculi and is complaining of moderate medication:
to severe flank pain and nausea. The clients A. Should be taken in the morning
temperature is 100.8 degrees Fahrenheit. The B. May decrease the clients energy level
priority nursing goal for this client is: C. Must be stored in a dark container
A. Maintain fluid and electrolyte balance D. Will decrease the clients heart rate
B. Control nausea
C. Manage pain 9. A 3-year-old child comes to the pediatric clinic
D. Prevent urinary tract infection after the sudden onset of findings that include
irritability, thick muffled voice, croaking on
5. What would the nurse expect to see while inspiration, hot to touch, sit leaning forward,
assessing the growth of children during their tongue protruding, drooling and suprasternal
school age years? retractions. What should the nurse do first?
A. Decreasing amounts of body fat and muscle A. Prepare the child for X-ray of upper airways
mass B. Examine the childs throat
B. Little change in body appearance from year to C. Collect a sputum specimen
year D. Notify the healthcare provider of the childs
C. Progressive height increase of 4 inches each status
year
D. Yearly weight gain of about 5.5 pounds per 10. In children suspected to have a diagnosis of
year diabetes, which one of the following complaints
would be most likely to prompt parents to take
6. At a community health fair, the blood pressure their school age child for evaluation?
of a 62-year-old client is 160/96 mmHg. The A. Polyphagia
client states My blood pressure is usually much B. Dehydration
lower. The nurse should tell the client to C. Bedwetting
A. go get a blood pressure check within the next D. Weight loss
48 to 72 hours
B. check blood pressure again in two (2) months 11. A client comes to the clinic for treatment of
recurrent pelvic inflammatory disease. The D. Exercise of joints
nurse recognizes that this condition most
frequently follows which type of infection? 16. The nurse is performing a neurological
A. Trichomoniasis assessment on a client post right CVA. Which
B. Chlamydia finding, if observed by the nurse, would warrant
C. Staphylococcus immediate attention?
D. Streptococcus A. Decrease in level of consciousness
B. Loss of bladder control
12. An RN who usually works in a spinal C. Altered sensation to stimuli
rehabilitation unit is floated to the emergency D. Emotional ability
department. Which of these clients should the
charge nurse assign to this RN? 17. A child who has recently been diagnosed
A. A middle-aged client who says I took too with cystic fibrosis is in a pediatric clinic where a
many diet pills and my heart feels like it is nurse is performing an assessment. Which later
racing out of my chest. finding of this disease would the nurse not
B. A young adult who says I hear songs from expect to see at this time?
heaven. I need money for beer. I quit drinking A. Positive sweat test
two (2) days ago for my family. Why are my B. Bulky greasy stools
arms and legs jerking? C. Moist, productive cough
C. An adolescent who has been on pain D. Meconium ileus
medications terminal cancer with an initial
assessment finding pupils and a relaxed 18. The home health nurse visits a male client to
respiratory rate of 10, provide wound care and finds the client lethargic
D. An elderly client who reports having taken a and confused. His wife states he fell down the
large crack hit 10 minutes prior to walking into stairs 2 hours ago. The nurse should
the emergency room. A. Place a call to the clients health care provider
for instructions
13. When teaching a client with coronary artery B. Send him to the emergency room for
disease about nutrition, the nurse should evaluation
emphasize C. Reassure the clients wife that the symptoms
A. Eating three (3) balanced meals a day are transient
B. Adding complex carbohydrates D. Instruct the clients wife to call the doctor if his
C. Avoiding very heavy meals symptoms become worse
D. Limiting sodium to 7 gms per day
19. Which of the following should the nurse
14. Which of these findings indicate that a pump implement to prepare a client for a KUB (Kidney,
to deliver a basal rate of 10 ml per hour plus Ureter, Bladder) radiograph test?
PRN for pain break through for morphine drip is A. Client must be NPO before the examination
not working? B. Enema to be administered prior to the
A. The client complains of discomfort at the IV examination
insertion site C. Medicate client with Lasix 20 mg IV 30
B. The client states I just cant get relief from minutes prior to the examination
my pain. D. No special orders are necessary for this
C. The level of drug is 100 ml at 8 AM and is 80 examination
ml at noon
D. The level of the drug is 100 ml at 8 AM and is 20. The nurse is giving discharge teaching to a
50 ml at noon client seven (7) days post myocardial infarction.
He asks the nurse why he must wait six (6)
15. The nurse is speaking at a community weeks before having sexual intercourse. What is
meeting about personal responsibility for health the best response by the nurse to this question?
promotion. A participant asks about chiropractic A. You need to regain your strength before
treatment for illnesses. What should be the attempting such exertion.
focus of the nurses response? B. When you can climb 2 flights of stairs without
A. Electrical energy fields problems, it is generally safe.
B. Spinal column manipulation C. Have a glass of wine to relax you, then you
C. Mind-body balance can try to have sex.
D. If you can maintain an active walking A. All striated muscles
program, you will have less risk. B. The cerebellum
C. The kidneys
21. A triage nurse has these four (4) clients D. The leg bones
arrive in the emergency department within 15
minutes. Which client should the triage nurse 26. The nurse anticipates that for a family who
send back to be seen first? practices Chinese medicine the priority goal
A. A 2-month-old infant with a history of rolling would be to:
off the bed and has bulging fontanels with crying A. Achieve harmony
B. A teenager who got a singed beard while B. Maintain a balance of energy
camping C. Respect life
C. An elderly client with complaints of frequent D. Restore yin and yang
liquid brown colored stools
D. A middle-aged client with intermittent pain 27. During an assessment of a client with
behind the right scapula cardiomyopathy, the nurse finds that the systolic
blood pressure has decreased from 145 to 110
22. While planning care for a toddler, the nurse mm Hg and the heart rate has risen from 72 to
teaches the parents about the expected 96 beats per minute and the client complains of
developmental changes for this age. Which periodic dizzy spells. The nurse instructs the
statement by the mother shows that she client to
understands the childs developmental needs? A. Increase fluids that are high in protein
A. I want to protect my child from any falls. B. Restrict fluids
B. I will set limits on exploring the house. C. Force fluids and reassess blood pressure
C. I understand the need to use those new D. Limit fluids to non-caffeine beverages
skills.
D. I intend to keep control over our child. 28. The nurse prepares the client for insertion of
a pulmonary artery catheter (Swan-Ganz
catheter). The nurse teaches the client that the
23. The nurse is preparing to administer an catheter will be inserted to provide information
enteral feeding to a client via a nasogastric about:
feeding tube. The most important action of the A. Stroke volume
nurse is B. Cardiac output
A. Verify correct placement of the tube C. Venous pressure
B. Check that the feeding solution matches the D. Left ventricular functioning
dietary order
C. Aspirate abdominal contents to determine the 29. A nurse enters a clients room to discover
amount of last feeding remaining in stomach that the client has no pulse or respirations. After
D. Ensure that feeding solution is at room calling for help, the first action the nurse should
temperature take is:
A. Start a peripheral IV
24. The nurse is caring for a client with a serum B. Initiate high-quality chest compressions
potassium level of 3.5 mEq/L. The client is C. Establish an airway
placed on a cardiac monitor and receives 40 D. Obtain the crash cart
mEq KCL in 1000 ml of 5% dextrose in water IV.
Which of the following EKG patterns indicates to 30. A client is receiving digoxin (Lanoxin) 0.25
the nurse that the infusions should be mg daily. The health care provider has written a
discontinued? new order to give metoprolol (Lopressor) 25 mg
A. Narrowed QRS complex B.I.D. In assessing the client prior to
B. Shortened PR interval administering the medications, which of the
C. Tall peaked T waves following should the nurse report immediately to
D. Prominent U waves the health care provider?
A. Blood pressure 94/60
25. A nurse prepares to care for a 4-year-old B. Heart rate 76
newly admitted for rhabdomyosarcoma. The C. Urine output 50 ml/hour
nurse should alert the staff to pay more attention D. Respiratory rate 16
to the function of which area of the body?
31. While assessing a 1-month-old infant, which 37. A nurse is to administer meperidine
finding should the nurse report immediately? hydrochloride (Demerol) 100 mg, atropine
A. Abdominal respirations sulfate (Atropisol) 0.4 mg, and promethazine
B. Irregular breathing rate hydrochloride (Phenergan) 50 mg IM to a pre-
C. Inspiratory grunt operative client. Which action should the nurse
D. Increased heart rate with crying take first?
A. Raise the side rails on the bed
32. The nurse practicing in a maternity setting B. Place the call bell within reach
recognizes that the post mature fetus is at risk C. Instruct the client to remain in bed
due to D. Have the client empty bladder
A. Excessive fetal weight
B. Low blood sugar levels
C. Depletion of subcutaneous fat 38. Which of these statements best describes
D. Progressive placental insufficiency the characteristic of an effective reward-
feedback system?
33. The nurse is caring for a client who had a A. Specific feedback is given as close to the
total hip replacement four (4) days ago. Which event as possible
assessment requires the nurses immediate B. Staff is given feedback in equal amounts over
attention? time
A. I have bad muscle spasms in my lower leg of C. Positive statements are to precede a negative
the affected extremity. statement
B. I just cant catch my breath over the past D. Performance goals should be higher than
few minutes and I think I am in grave danger. what is attainable
C. I have to use the bedpan to pass my water at
least every 1 to 2 hours. 39. A client with multiple sclerosis plans to begin
D. It seems that the pain medication is not an exercise program. In addition to discussing
working as well today. the benefits of regular exercise, the nurse
should caution the client to avoid activities which
34. A client has been taking furosemide (Lasix) A. Increase the heart rate
for the past week. The nurse recognizes which B. Lead to dehydration
finding may indicate the client is experiencing a C. Are considered aerobic
negative side effect from the medication? D. May be competitive
A. Weight gain of 5 pounds
B. Edema of the ankles 40. During the evaluation of the quality of home
C. Gastric irritability care for a client with Alzheimers disease, the
D. Decreased appetite priority for the nurse is to reinforce which
statement by a family member?
35. A client who is pregnant comes to the clinic A. At least two (2) full meals a day is eaten.
for a first visit. The nurse gathers data about her B. We go to a group discussion every week at
obstetric history, which includes 3-year-old twins our community center.
at home and a miscarriage 10 years ago at 12 C. We have safety bars installed in the bathroom
weeks gestation. How would the nurse and have 24-hour alarms on the doors.
accurately document this information? D. The medication is not a problem to have it
A. Gravida 4 para 2 taken three (3) times a day.
B. Gravida 2 para 1
C. Gravida 3 para 1 NCLEX Questions 2
D. Gravida 3 para 2
1. A nurse is reviewing a patients medication
36. The nurse is caring for a client with a venous during shift change. Which of the following
stasis ulcer. Which nursing intervention would medication would be contraindicated if the
be most effective in promoting healing? patient were pregnant? Select all that apply:
A. Apply dressing using sterile technique A. Warfarin (Coumadin)
B. Improve the clients nutrition status B. Finasteride (Propecia, Proscar)
C. Initiate limb compression therapy C. Celecoxib (Celebrex)
D. Begin proteolytic debridement D. Clonidine (Catapres)
E. Transdermal nicotine (Habitrol)
F. Clofazimine(Lamprene) 8. You are taking the history of a 14-year-old girl
2. A nurse is reviewing a patients past medical who has a (BMI) of 18. The girl reports inability
history (PMH). The history indicates the patient to eat, induced vomiting and severe
has photosensitive reactions to medications. constipation. Which of the following would you
Which of the following drugs is associated with most likely suspect?
photosensitive reactions? Select all that apply: A. Multiple sclerosis
A. Ciprofloxacin (Cipro) B. Anorexia nervosa
B. Sulfonamide C. Bulimia nervosa
C. Norfloxacin (Noroxin) D. Systemic sclerosis
D. Sulfamethoxazole and Trimethoprim 9. A 24-year-old female is admitted to the ER for
(Bactrim) confusion. This patient has a history of a
E. Isotretinoin (Accutane) myeloma diagnosis, constipation, intense
F. Nitro-Dur patch abdominal pain, and polyuria. Based on the
3. A patient tells you that her urine is starting to presenting signs and symptoms, which of the
look discolored. If you believe this change is due following would you most likely suspect?
to medication, which of the following of the A. Diverticulosis
patients medication does not cause urine B. Hypercalcaemia
discoloration? C. Hypocalcemia
A. Sulfasalazine D. Irritable bowel syndrome
B. Levodopa 10. Rhogam is most often used to treat____
C. Phenolphthalein mothers that have a ____ infant.
D. Aspirin A. RH positive, RH positive
4. You are responsible for reviewing the nursing B. RH positive, RH negative
units refrigerator. Which of the following drug, if C. RH negative, RH positive
found inside the fridge, should be removed? D. RH negative, RH negative
A. Nadolol (Corgard) 11. A new mother has some questions about
B. Opened (in-use) Humulin N injection phenylketonuria (PKU). Which of the following
C. Urokinase (Kinlytic) statements made by a nurse is not correct
D. Epoetin alfa IV (Epogen) regarding PKU?
5. A 34-year-old female has recently been A. A Guthrie test can check the necessary lab
diagnosed with an autoimmune disease. She values.
has also recently discovered that she is B. The urine has a high concentration of
pregnant. Which of the following is the only phenylpyruvic acid
immunoglobulin that will provide protection to C. Mental deficits are often present with PKU.
the fetus in the womb? D. The effects of PKU are reversible.
A. IgA 12. A patient has taken an overdose of aspirin.
B. IgD Which of the following should a nurse most
C. IgE closely monitor for during acute management of
D. IgG this patient?
6. A second year nursing student has just A. Onset of pulmonary edema
suffered a needlestick while working with a B. Metabolic alkalosis
patient that is positive for AIDS. Which of the C. Respiratory alkalosis
following is the most significant action that D. Parkinsons disease type symptoms
nursing student should take? 13. A 50-year-old blind and deaf patient have
A. Immediately see a social worker. been admitted to your floor. As the charge
B. Start prophylactic AZT treatment. nurse, your primary responsibility for this patient
C. Start prophylactic Pentamidine treatment. is?
D. Seek counseling. A. Let others know about the patients deficits.
7. A thirty-five-year-old male has been an B. Communicate with your supervisor your
insulin-dependent diabetic for five years and patient safety concerns.
now is unable to urinate. Which of the following C. Continuously update the patient on the social
would you most likely suspect? environment.
A. Atherosclerosis D. Provide a secure environment for the patient.
B. Diabetic nephropathy 14. A patient is getting discharged from a skilled
C. Autonomic neuropathy nursing facility (SNF). The patient has a history
D. Somatic neuropathy of severe COPD and PVD. The patient is
primarily concerned about his ability to breathe B. The life span of RBC is 60 days.
easily. Which of the following would be the best C. The life span of RBC is 90 days.
instruction for this patient? D. The life span of RBC is 120 days.
A. Deep breathing techniques to increase 21. A 65-year-old man has been admitted to the
oxygen levels. hospital for spinal stenosis surgery. When
B. Cough regularly and deeply to clear airway should the discharge training and planning begin
passages. for this patient?
C. Cough following bronchodilator utilization. A. Following surgery
D. Decrease CO2 levels by increased oxygen B. Upon admit
take output during meals. C. Within 48 hours of discharge
15. A nurse is caring for an infant that has D. Preoperative discussion
recently been diagnosed with a congenital heart 22. A 5-year-old child and has been recently
defect. Which of the following clinical signs admitted to the hospital. According to
would most likely be present? Erik Eriksons psychosocial development stages,
A. Slow pulse rate the child is in which stage?
B. Weight gain A. Trust vs. mistrust
C. Decreased systolic pressure B. Initiative vs. guilt
D. Irregular WBC lab values C. Autonomy vs. shame and doubt
16. A mother has recently been informed that D. Intimacy vs. isolation
her child has Downs syndrome. You will be 23. A toddler is 26 months old and has been
assigned to care for the child at shift change. recently admitted to the hospital. According to
Which of the following characteristics is not Erikson, which of the following stages is the
associated with Downs syndrome? toddler in?
A. Simian crease A. Trust vs. mistrust
B. Brachycephaly B. Initiative vs. guilt
C. Oily skin C. Autonomy vs. shame and doubt
D. Hypotonicity D. Intimacy vs. isolation
17. A client with myocardial infarction is 24. A young adult is 20 years old and has been
receiving tissue plasminogen activator, alteplase recently admitted to the hospital. According to
(Activase, tPA). While on the therapy, the nurse Erikson, which of the following stages is the
plans to prioritize which of the following? adult in?
A. Observe for neurological changes. A. Trust vs. mistrust
B. Monitor for any signs of renal failure. B. Initiative vs. guilt
C. Check the food diary. C. Autonomy vs. shame
D. Observe for signs of bleeding. D. Intimacy vs. isolation
18. A patient asks a nurse, My doctor 25. A nurse is making rounds taking vital signs.
recommended I increase my intake of folic acid. Which of the following vital signs is abnormal?
What type of foods contain the highest A. 11-year-old male: 90 BPM, 22 RPM, 100/70
concentration of folic acids? mmHg
A. Green vegetables and liver B. 13-year-old female: 105 BPM, 22 RPM,
B. Yellow vegetables and red meat 105/50 mmHg
C. Carrots C. 5-year-old male: 102 BPM, 24 RPM, 90/65
D. Milk mmHg
19. A nurse is putting together a presentation on D. 6-year-old female: 100 BPM, 26 RPM, 90/70
meningitis. Which of the following mmHg
microorganisms has not been linked to 26. When you are taking a patients history, she
meningitis in humans? tells you she has been depressed and is dealing
A. S. pneumoniae with an anxiety disorder. Which of the following
B. H. influenzae medications would the patient most likely be
C. N. meningitidis taking?
D. Cl. difficile A. Amitriptyline (Elavil)
20. A nurse is administering blood to a patient B. Calcitonin
who has a low hemoglobin count. The patient C. Pergolide mesylate (Permax)
asks how long to RBCs last in my body? The D. Verapamil (Calan)
correct response is. 27. Which of the following conditions would a
A. The life span of RBC is 45 days. nurse not administer erythromycin?
A. Campylobacteriosis infection last two (2) months. The patient also has the
B. Legionnaires disease heater running in his house 24 hours a day,
C. Pneumonia even on warm days. Which of the following tests
D. Multiple Sclerosis is most likely to be performed?
28. A patients chart indicates a history of A. FBC (full blood count)
hyperkalemia. Which of the following would you B. ECG (electrocardiogram)
not expect to see with this patient if this C. Thyroid function tests
condition were acute? D. CT scan
A. Decreased HR 35. A 20-year-old female attending college is
B. Paresthesias found unconscious in her dorm room. She has a
C. Muscle weakness of the extremities fever and a noticeable rash. She has just been
D. Migraines admitted to the hospital. Which of the following
29. A patients chart indicates a history of tests is most likely to be performed first?
ketoacidosis. Which of the following would you A. Blood sugar check
not expect to see with this patient if this B. CT scan
condition were acute? C. Blood cultures
A. Vomiting D. Arterial blood gases
B. Extreme Thirst 36. A 28-year-old male has been found
C. Weight gain wandering around in a confused pattern. The
D. Acetone breath smell male is sweaty and pale. Which of the following
30. A patients chart indicates a history of tests is most likely to be performed first?
meningitis. Which of the following would you not A. Blood sugar check
expect to see with this patient if this condition B. CT scan
were acute? C. Blood cultures
A. Increased appetite D. Arterial blood gases
B. Vomiting 37. A mother is inquiring about her childs ability
C. Fever to potty train. Which of the following factors is
D. Poor tolerance of light the most important aspect of toilet training?
31. A nurse if reviewing a patients chart and A. The age of the child
notices that the patient suffers from B. The childs ability to understand instruction.
conjunctivitis. Which of the following C. The overall mental and physical abilities of
microorganisms is related to this condition? the child.
A. Yersinia pestis D. Frequent attempts with positive
B. Helicobacter pylori reinforcement.
C. Vibrio cholerae 38. A parent calls the pediatric clinic and is
D. Haemophilus aegyptius frantic about the bottle of cleaning fluid her child
32. A nurse if reviewing a patients chart and drank 20 minutes. Which of the following is the
notices that the patient suffers from Lyme most important instruction the nurse can give the
disease. Which of the following microorganisms parent?
is related to this condition? A. This too shall pass.
A. Borrelia burgdorferi B. Take the child immediately to the ER
B. Streptococcus pyogenes C. Contact the Poison Control Center quickly
C. Bacillus anthracis D. Give the child syrup of ipecac
D. Enterococcus faecalis 39. A nurse is administering a shot of Vitamin K
33. A fragile 87-year-old female has recently to a 30 day-old infant. Which of the following
been admitted to the hospital with increased target areas is the most appropriate?
confusion and falls over last two (2) weeks. She A. Gluteus maximus
is also noted to have a mild left hemiparesis. B. Gluteus minimus
Which of the following tests is most likely to be C. Vastus lateralis
performed? D. Vastus medialis
A. FBC (full blood count) 40. A nurse has just started her rounds
B. ECG (electrocardiogram) delivering medication. A new patient on her
C. Thyroid function tests rounds is a 4-year-old boy who is non-verbal.
D. CT scan This child does not have on any identification.
34. An 84-year-old male has What should the nurse do?
been losing mobility and gaining weight over the A. Contact the provider
B. Ask the child to write their name on paper. 6. A nurse is assessing a clinic patient with a
C. Ask a coworker about the identification of the diagnosis of hepatitis A. Which of the
child. following is the most likely route of
D. Ask the father who is in the room the childs transmission?
name. A. Sexual contact with an infected partner.
B. Contaminated food.
NCLEX Questions 3 C. Blood transfusion.
1. A patient is admitted to the hospital with a D. Illegal drug use.
diagnosis of primary hyperparathyroidism. A 7. A leukemia patient has a relative who
nurse checking the patients lab results wants to donate blood for transfusion. Which
would expect which of the following changes of the following donor medical conditions
in laboratory findings? would prevent this?
A. Elevated serum calcium. A. A history of hepatitis C five years previously.
B. Low serum parathyroid hormone (PTH). B. Cholecystitis requiring cholecystectomy one
C. Elevated serum vitamin D. year previously.
D. Low urine calcium. C. Asymptomatic diverticulosis.
2. A patient with Addisons disease asks a D. Crohns disease in remission.
nurse for nutrition and diet advice. Which of 8. A physician has diagnosed acute gastritis
the following diet modifications is NOT in a clinic patient. Which of the following
recommended? medications would be contraindicated for
A. A diet high in grains. this patient?
B. A diet with adequate caloric intake. A. Naproxen sodium (Naprosyn).
C. A high protein diet. B. Calcium carbonate.
D. A restricted sodium diet. C. Clarithromycin (Biaxin).
3. A patient with a history of diabetes D. Furosemide (Lasix).
mellitus is on the second post-operative day 9. The nurse is conducting nutrition
following cholecystectomy. She has counseling for a patient with cholecystitis.
complained of nausea and isnt able to eat Which of the following information is
solid foods. The nurse enters the room to important to communicate?
find the patient confused and shaky. Which A. The patient must maintain a low-calorie diet.
of the following is the most likely explanation B. The patient must maintain a high protein/low
for the patients symptoms? carbohydrate diet.
A. Anesthesia reaction. C. The patient should limit sweets and sugary
B. Hyperglycemia. drinks.
C. Hypoglycemia. D. The patient should limit fatty foods.
D. Diabetic ketoacidosis. 10. A patient admitted to the hospital with
4. A nurse assigned to the emergency myocardial infarction develops severe
department evaluates a patient who pulmonary edema. Which of the following
underwent fiberoptic colonoscopy 18 hours symptoms should the nurse expect the
previously. The patient reports increasing patient to exhibit?
abdominal pain, fever, and chills. Which of A. Slow, deep respirations.
the following conditions poses the most B. Stridor.
immediate concern? C. Bradycardia.
A. Bowel perforation. D. Air hunger.
B. Viral gastroenteritis. 11. A nurse caring for several patients on the
C. Colon cancer. cardiac unit is told that one is scheduled for
D. Diverticulitis. implantation of an automatic internal
5. A patient is admitted to the same day cardioverter-defibrillator. Which of the
surgery unit for liver biopsy. Which of the following patients is most likely to have this
following laboratory tests assesses procedure?
coagulation? A. A patient admitted for myocardial infarction
A. Partial thromboplastin time. without cardiac muscle damage.
B. Prothrombin time. B. A post-operative coronary bypass patient,
C. Platelet count. recovering on schedule.
D. Hemoglobin C. A patient with a history of ventricular
tachycardia and syncopal episodes.
D. A patient with a history of atrial tachycardia Which of the following signs or symptoms
and fatigue. would be cause for concern?
12. A patient is scheduled for a magnetic A. Bulging anterior fontanel.
resonance imaging (MRI) scan for suspected B. Repeated vomiting.
lung cancer. Which of the following is a C. Signs of sleepiness at 10 PM.
contraindication to the study for this patient? D. Inability to read short words from a distance
A. The patient is allergic to shellfish. of 18 inches.
B. The patient has a pacemaker. 17. A nonimmunized child appears at the
C. The patient suffers from claustrophobia. clinic with a visible rash. Which of the
D. The patient takes anti-psychotic medication. following observations indicates the child
13. A nurse calls a physician with the may have rubeola (measles)?
concern that a patient has developed a A. Small blue-white spots are visible on the oral
pulmonary embolism. Which of the following mucosa.
symptoms has the nurse most likely B. The rash begins on the trunk and spreads
observed? outward.
A. The patient is somnolent with decreased C. There is low-grade fever.
response to the family. D. The lesions have a teardrop-on-a-rose-petal
B. The patient suddenly complains of chest pain appearance.
and shortness of breath. 18. A child is seen in the emergency
C. The patient has developed a wet cough and department for scarlet fever. Which of the
the nurse hears crackles on auscultation of the following descriptions of scarlet fever is NOT
lungs. correct?
D. The patient has a fever, chills, and loss of A. Scarlet fever is caused by infection with group
appetite. A Streptococcus bacteria.
14. A patient comes to the emergency B. Strawberry tongue is a characteristic sign.
department with abdominal pain. Work-up C. Petechiae occur on the soft palate.
reveals the presence of a rapidly enlarging D. The pharynx is red and swollen.
abdominal aortic aneurysm. Which of the 19. A child weighing 30 kg arrives at the
following actions should the nurse expect? clinic with diffuse itching as the result of an
A. The patient will be admitted to the medicine allergic reaction to an insect bite.
unit for observation and medication. Diphenhydramine (Benadryl) 25 mg 3 times a
B. The patient will be admitted to the day day is prescribed. The correct pediatric dose
surgery unit for sclerotherapy. is 5 mg/kg/day. Which of the following best
C. The patient will be admitted to the surgical describes the prescribed drug dose?
unit and resection will be scheduled. A. It is the correct dose.
D. The patient will be discharged home to follow- B. The dose is too low.
up with his cardiologist in 24 hours. C. The dose is too high.
15. A patient with leukemia is receiving D. The dose should be increased or decreased,
chemotherapy that is known to depress bone depending on the symptoms.
marrow. A CBC (complete blood count) 20. The mother of a 2-month-old infant brings
reveals a platelet count of 25,000/microliter. the child to the clinic for a well-baby check.
Which of the following actions related She is concerned because she feels only one
specifically to the platelet count should be testis in the scrotal sac. Which of the
included in the nursing care plan? following statements about the undescended
A. Monitor for fever every 4 hours. testis is the most accurate?
B. Require visitors to wear respiratory masks A. Normally, the testes are descended by birth.
and protective clothing. B. The infant will likely require surgical
C. Consider transfusion of packed red blood intervention.
cells. C. The infant probably has with only one testis.
D. Check for signs of bleeding, including D. Normally, the testes descend by one year of
examination of urine and stool for blood. age.
16. A nurse in the emergency department is 21. A child is admitted to the hospital with a
observing a 4-year-old child for signs of diagnosis of Wilms tumor, stage II. Which of
increased intracranial pressure after a fall the following statements most accurately
from a bicycle, resulting in head trauma. describes this stage?
A. The tumor is less than 3 cm. in size and hereditary risk factors for developing
requires no chemotherapy. atherosclerosis?
B. The tumor did not extend beyond the kidney A. Family history of heart disease.
and was completely resected. B. Overweight.
C. The tumor extended beyond the kidney but C. Smoking.
was completely resected. D. Age.
D. The tumor has spread into the abdominal 27. Claudication is a well-known effect of
cavity and cannot be resected. peripheral vascular disease. Which of the
22. A teen patient is admitted to the hospital following facts about claudication is correct?
by his physician who suspects a diagnosis Select all that apply:
of acute glomerulonephritis. Which of the A. It results when oxygen demand is greater
following findings is consistent with this than oxygen supply.
diagnosis? Note: More than one answer may B. It is characterized by pain that often occurs
be correct. during rest.
A. Urine specific gravity of 1.040. C. It is a result of tissue hypoxia.
B. Urine output of 350 ml in 24 hours. D. It is characterized by cramping and
C. Brown (tea-colored) urine. weakness.
D. Generalized edema. 28. A nurse is providing discharge
23. Which of the following conditions most information to a patient with peripheral
commonly causes acute glomerulonephritis? vascular disease. Which of the following
A. A congenital condition leading to renal information should be included in
dysfunction. instructions?
B. Prior infection with group A Streptococcus A. Walk barefoot whenever possible.
within the past 10-14 days. B. Use a heating pad to keep feet warm.
C. Viral infection of the glomeruli. C. Avoid crossing the legs.
D. Nephrotic syndrome. D. Use antibacterial ointment to treat skin
24. An infant with hydrocele is seen in the lesions at risk of infection.
clinic for a follow-up visit at 1 month of age. 29. A patient who has been diagnosed with
The scrotum is smaller than it was at birth, vasospastic disorder (Raynauds disease)
but fluid is still visible on illumination. Which complains of cold and stiffness in the
of the following actions is the physician fingers. Which of the following descriptions
likely to recommend? is most likely to fit the patient?
A. Massaging the groin area twice a day until the A. An adolescent male.
fluid is gone. B. An elderly woman.
B. Referral to a surgeon for repair. C. A young woman.
C. No treatment is necessary; the fluid is D. An elderly man.
reabsorbing normally. 30. A 23-year-old patient in the 27th week of
D. Keeping the infant in a flat, supine position pregnancy has been hospitalized on
until the fluid is gone. complete bed rest for 6 days. She
25. A nurse is caring for a patient with experiences sudden shortness of breath,
peripheral vascular disease (PVD). The accompanied by chest pain. Which of the
patient complains of burning and tingling of following conditions is the most likely cause
the hands and feet and cannot tolerate touch of her symptoms?
of any kind. Which of the following is the A. Myocardial infarction due to a history of
most likely explanation for these symptoms? atherosclerosis.
A. Inadequate tissue perfusion leading to nerve B. Pulmonary embolism due to deep vein
damage. thrombosis (DVT).
B. Fluid overload leading to compression of C. Anxiety attack due to worries about her
nerve tissue. babys health.
C. Sensation distortion due to psychiatric D. Congestive heart failure due to fluid overload.
disturbance. 31. Thrombolytic therapy is frequently used
D. Inflammation of the skin on the hands and in the treatment of suspected stroke. Which
feet. of the following is a significant complication
26. A patient in the cardiac unit is concerned associated with thrombolytic therapy?
about the risk factors associated with A. Air embolus.
atherosclerosis. Which of the following are B. Cerebral hemorrhage.
C. Expansion of the clot. D. Physical activity should be minimized.
D. Resolution of the clot. 37. A child is admitted to the hospital several
32. An infant is brought to the clinic by his days after stepping on a sharp object that
mother, who has noticed that he holds his punctured her athletic shoe and entered the
head in an unusual position and always flesh of her foot. The physician is concerned
faces to one side. Which of the following is about osteomyelitis and has ordered
the most likely explanation? parenteral antibiotics. Which of the following
A. Torticollis, with shortening of the actions is done immediately before the
sternocleidomastoid muscle. antibiotic is started?
B. Craniosynostosis, with premature closure of A. The admission orders are written.
the cranial sutures. B. A blood culture is drawn.
C. Plagiocephaly, with flattening of one side of C. A complete blood count with differential is
the head. drawn.
D. Hydrocephalus, with increased head size. D. The parents arrive.
33. An adolescent brings a physicians note 38. A two-year-old child has sustained an
to school stating that he is not to participate injury to the leg and refuses to walk. The
in sports due to a diagnosis of Osgood- nurse in the emergency department
Schlatter disease. Which of the following documents swelling of the lower affected leg.
statements about the disease is correct? Which of the following does the nurse
A. The condition was caused by the students suspect is the cause of the childs
competitive swimming schedule. symptoms?
B. The student will most likely require surgical A. Possible fracture of the tibia.
intervention. B. Bruising of the gastrocnemius muscle.
C. The student experiences pain in the inferior C. Possible fracture of the radius.
aspect of the knee. D. No anatomic injury, the child wants his
D. The student is trying to avoid participation in mother to carry him.
physical education. 39. A toddler has recently been diagnosed
34. The clinic nurse asks a 13-year-old with cerebral palsy. Which of the following
female to bend forward at the waist with information should the nurse provide to the
arms hanging freely. Which of the following parents? Note: More than one answer may
assessments is the nurse most likely be correct.
conducting? A. Regular developmental screening is important
A. Spinal flexibility. to avoid secondary developmental delays.
B. Leg length disparity. B. Cerebral palsy is caused by injury to the
C. Hypostatic blood pressure. upper motor neurons and results in motor
D. Scoliosis. dysfunction, as well as possible ocular and
35. A clinic nurse interviews a parent who is speech difficulties.
suspected of abusing her child. Which of the C. Developmental milestones may be slightly
following characteristics is the nurse LEAST delayed but usually will require no additional
likely to find in an abusing parent? intervention.
A. Low self-esteem. D. Parent support groups are helpful for sharing
B. Unemployment. strategies and managing health care issues.
C. Self-blame for the injury to the child. 40. A child has recently been diagnosed with
D. Single status. Duchennes muscular dystrophy. The
36. A nurse is assigned to the pediatric parents are receiving genetic counseling
rheumatology clinic and is assessing a child prior to planning another pregnancy. Which
who has just been diagnosed with juvenile of the following statements includes the
idiopathic arthritis. Which of the following most accurate information?
statements about the disease is most A. Duchennes is an X-linked recessive disorder,
accurate? so daughters have a 50% chance of being
A. The child has a poor chance of recovery carriers and sons a 50% chance of developing
without joint deformity. the disease.
B. Most children progress to adult rheumatoid B. Duchennes is an X-linked recessive disorder,
arthritis. so both daughters and sons have a 50% chance
C. Nonsteroidal anti-inflammatory drugs are the of developing the disease.
first choice in treatment.
C. Each child has a 1 in 4 (25%) chance of C. The pain is psychological because your foot
developing the disorder. is no longer there.
D. Sons only have a 1 in 4 (25%) chance of D. The pain and itching are due to the infection
developing the disorder. you had before the surgery.
6. A client with cancer of the pancreas has
NCLEX Questions 4 undergone a Whipple procedure. The nurse
1. The primary reason for rapid continuous is aware that during the Whipple procedure,
rewarming of the area affected by frostbite is the doctor will remove the:
to: A. Head of the pancreas
A. Lessen the amount of cellular damage B. Proximal third section of the small intestines
B. Prevent the formation of blisters C. Stomach and duodenum
C. Promote movement D. Esophagus and jejunum
D. Prevent pain and discomfort 7. The physician has ordered a minimal-
2. A client recently started on hemodialysis bacteria diet for a client with neutropenia.
wants to know how the dialysis will take the The client should be taught to AVOID eating:
place of his kidneys. The nurses response is A. Packed fruits
based on the knowledge that hemodialysis B. Salt
works by: C. Fresh raw pepper
A. Passing water through a dialyzing membrane D. Ketchup
B. Eliminating plasma proteins from the blood 8. A client is discharged home with a
C. Lowering the pH by removing nonvolatile prescription for Coumadin (sodium warfarin).
acids The client should be instructed to:
D. Filtering waste through a dialyzing membrane A. Have a Protime done monthly
3. During a home visit, a client with AIDS B. Eat more fruits and vegetables
tells the nurse that he has been exposed to C. Drink more liquids
measles. Which action by the nurse is most D. Avoid crowds
appropriate? 9. The nurse is assisting the physician with
A. Administer an antibiotic removal of a central venous catheter. To
B. Contact the physician for an order for immune facilitate removal, the nurse should instruct
globulin the client to:
C. Administer an antiviral A. Perform the Valsalva maneuver as the
D. Tell the client that he should remain in catheter is advanced
isolation for 2 weeks B. Turn his head to the left side and hyperextend
4. A client hospitalized with MRSA the neck
(methicillin-resistant staph aureus) is placed C. Take slow, deep breaths as the catheter is
on contact precautions. Which statement is removed
true regarding precautions for infections D. Turn his head to the right while maintaining a
spread by contact? sniffing position
A. The client should be placed in a room with 10. A client has an order for streptokinase.
negative pressure. Before administering the medication, the
B. Infection requires close contact; therefore, the nurse should assess the client for:
door may remain open. A. Allergies to pineapples and bananas
C. Transmission is highly likely, so the client B. A history of streptococcal infections
should wear a mask at all times. C. Prior therapy with phenytoin
D. Infection requires skin-to-skin contact and is D. A history of alcohol abuse
prevented by hand washing, gloves, and a 11. The nurse is providing discharge
gown. teaching for the client with leukemia. The
5. A client who is admitted with an above- client should be told to avoid:
the-knee amputation tells the nurse that his A. Using oil- or cream-based soaps
foot hurts and itches. Which response by the B. Flossing between the teeth
nurse indicates understanding of phantom C. The intake of salt
limb pain? D. Using an electric razor
A. The pain will go away in a few days. 12. The nurse is changing the ties of the
B. The pain is due to peripheral nervous system client with a tracheotomy. The safest method
interruptions. I will get you some pain of changing the tracheotomy ties is to:
medication.
A. Apply the new tie before removing the old
one.
B. Have a helper present.
C. Hold the tracheotomy with the nondominant
hand while removing the old tie.
D. Ask the doctor to suture the tracheostomy in
place.
13. The nurse is monitoring a client following
a lung resection. The hourly output from the
chest tube was 300mL. The nurse should
give priority to:
A. Turning the client to the left side
B. Milking the tube to ensure patency
C. Slowing the intravenous infusion
D. Notifying the physician
14. The infant is admitted to the unit with
tetrology of falot. The nurse would anticipate
an order for which medication?
A. Digoxin
B. Epinephrine
C. Aminophylline
D. Atropine
15. The nurse is educating the ladys club in
self-breast exam. The nurse is aware that
most malignant breast masses occur in the
Tail of Spence. On the diagram below, select
where the Tail of Spence is.

16. The toddler is admitted with a cardiac


anomaly. The nurse is aware that the infant
with a ventricular septal defect will:
A. Tire easily
B. Grow normally
C. Need more calories
D. Be more susceptible to viral infections
17. The nurse is monitoring a client with a
history of stillborn infants. The nurse is
aware that a nonstress test can be ordered
for this client to:
A. Determine lung maturity
B. Measure the fetal activity
C. Show the effect of contractions on fetal heart
rate
D. Measure the wellbeing of the fetus
18. The nurse is evaluating the client who
was admitted 8 hours ago for induction of
labor. The following graph is noted on the
monitor. Which action should be taken first
by the nurse?

A. Atrial flutter
B. A sinus rhythm
C. Ventricular tachycardia
D. Atrial fibrillation
20. A client with clotting disorder has an
order to continue Lovenox (enoxaparin)
injections after discharge. The nurse should
teach the client that Lovenox injections
should:
A. Be injected into the deltoid muscle
B. Be injected into the abdomen
C. Aspirate after the injection
D. Clear the air from the syringe before
injections
21. The nurse has a preop order to
administer Valium (diazepam) 10mg and
Phenergan (promethazine) 25mg. The correct
method of administering these medications
is to:
A. Administer the medications together in one
syringe
B. Administer the medication separately
C. Administer the Valium, wait 5 minutes, and
then inject the Phenergan
D. Question the order because they cannot be
given at the same time
22. A client with frequent urinary tract
infections asks the nurse how she can
prevent the reoccurrence. The nurse should
teach the client to:
A. Douche after intercourse
B. Void every 3 hours
C. Obtain a urinalysis monthly
D. Wipe from back to front after voiding
A. Instruct the client to push 23. Which task should be assigned to the
B. Perform a vaginal exam nursing assistant?
C. Turn off the Pitocin infusion A. Placing the client in seclusion
D. Place the client in a semi-Fowlers position B. Emptying the Foley catheter of the
19. The nurse notes the following on the ECG preeclamptic client
monitor. The nurse would evaluate the C. Feeding the client with dementia
cardiac arrhythmia as: D. Ambulating the client with a fractured hip
24. The client has recently returned from A. Thrombocytopenia.
having a thyroidectomy. The nurse should B. Idiopathic thrombocytopenic purpura (ITP).
keep which of the following at the bedside? C. Disseminated intravascular coagulation
A. A tracheotomy set (DIC).
B. A padded tongue blade D. Heparin-associated thrombosis and
C. An endotracheal tube thrombocytopenia (HATT).
D. An airway 32. A 16-year-old patient involved in a motor
25. The physician has ordered a vehicle accident arrives in the ED
histoplasmosis test for the elderly client. The unconscious and severely hypotensive. Hes
nurse is aware that histoplasmosis is suspected to have several fractures of his
transmitted to humans by: pelvis and legs. Which of the following
A. Cats parenteral fluids is the best choice for his
B. Dogs current condition?
C. Turtles A. Fresh frozen plasma.
D. Birds B. 0.9% sodium chloride solution.
26. Whats the first intervention for a patient C. Lactated Ringers solution.
experiencing chest pain and an p02 of 89%? D. Packed red blood cells.
A. Administer morphine. 33. Corticosteroids are potent suppressors
B. Administer oxygen. of the bodys inflammatory response. Which
C. Administer sublingual nitroglycerin. of the following conditions or actions do
D. Obtain an electrocardiogram (ECC) they suppress?
27. Which of the following signs and A. Cushing syndrome.
symptoms usually signifies rapid expansion B. Pain receptors.
and impending rupture of an abdominal C. Immune response.
aortic aneurysm? D. Neural transmission.
A. Abdominal pain. 34. A patient infected with human
B. Absent pedal pulses. immunodeficiency virus (HIV) begins
C. Chest pain. zidovudine therapy. Which of the following
D. Lower back pain. statements best describes this drugs
28. In which of the following types of action?
cardiomyopathy does cardiac output remain A. It destroys the outer wall of the virus and kills
normal? it.
A. Dilated. B. It interferes with viral replication.
B. Hypertrophic. C. It stimulates the immune system.
C. Obliterative. D. It promotes excretion of viral antibodies.
D. Restrictive. 35. A 20-year-old patient is being treated for
29. Which of the following interventions pneumonia. He has a persistent cough and
should be your first priority when treating a complains of severe pain on coughing. What
patient experiencing chest pain while could you tell him to help him reduce his
walking? discomfort?
A. Have the patient sit down. A. Hold your cough as much as possible.
B. Get the patient back to bed. B. Place the head of your bed flat to help with
C. Obtain an ECG. coughing.
D. Administer sublingual nitroglycerin. C. Restrict fluids to help decrease the amount
30. Which of the following positions would of sputum.
best aid breathing for a patient with acute D. Splint your chest wall with a pillow for
pulmonary edema? comfort.
A. Lying flat in bed. 36. A 19-year-old patient comes to the ED
B. Left side-lying position. with acute asthma. His respiratory rate is 44
C. High Fowlers position. breaths/minute, and he appears to be in
D. Semi-Fowlers position. acute respiratory distress. Which of the
31. A pregnant woman arrives at the following actions should you take first?
emergency department (ED) with abruptio A. Take a full medical history.
placentae at 34 weeks gestation. Shes at B. Give a bronchodilator by nebulizer.
risk for which of the following blood C. Apply a cardiac monitor to the patient.
dyscrasias? D. Provide emotional support for the patient.
37. A firefighter who was involved in D. Pain control.
extinguishing a house fire is being treated 44. After a liver biopsy, place the patient in
for smoke inhalation. He develops severe which of the following positions?
hypoxia 48 hours after the incident, requiring A. Left side-lying, with the bed flat.
intubation and mechanical ventilation. Which B. Right side-lying, with the bed flat.
of the following conditions has he most C. Left side-lying, with the bed in semi-Fowlers
likely developed? position.
A. Acute respiratory distress syndrome (ARDS). D. Right side-lying, with the bed in semi-Fowlers
B. Atelectasis. position.
C. Bronchitis. 45. Which of the following potentially serious
D. Pneumonia. complications could occur with therapy for
38. Which of the following measures best hypothyroidism?
determines that a patient who had a A. Acute hemolytic reaction.
pneumothorax no longer needs a chest B. Angina or cardiac arrhythmia.
tube? C. Retinopathy.
A. You see a lot of drainage from the chest tube. D. Thrombocytopenia.
B. Arterial blood gas (ABG) levels are normal. 46. Adequate fluid replacement and
C. The chest X-ray continues to show the lung is vasopressin replacement are objectives of
35% deflated. therapy for which of the following disease
D. The water-seal chamber doesnt fluctuate processes?
when no suction is applied. A. Diabetes mellitus.
39. Which of the following nursing B. Diabetes insipidus.
interventions should you use to prevent C. Diabetic ketoacidosis.
footdrop and contractures in a patient D. Syndrome of inappropriate antidiuretic
recovering from a subdural hematoma? hormone secretion (SIADH).
A. High-top sneakers. 47. Patients with Type 1 diabetes mellitus
B. Low-dose heparin therapy. may require which of the following changes
C. Physical therapy consultation. to their daily routine during periods of
D. Sequential compressive device. infection?
40. Which of the following signs of increased A. No changes.
intracranial pressure (ICP) would appear first B. Less insulin.
after head trauma? C. More insulin.
A. Bradycardia. D. Oral diabetic agents.
B. Large amounts of very dilute urine. 48. On a follow-up visit after having a vaginal
C. Restlessness and confusion. hysterectomy, a 32-year-old patient has a
D. Widened pulse pressure. decreased hematocrit level. Which of the
41. When giving intravenous (I.V.) phenytoin, following complications does this suggest?
which of the following methods should you A. Hematoma.
use? B. Hypovolemia.
A. Use an in-line filter. C. Infection.
B. Withhold other anticonvulsants. D. Pulmonary embolus (PE).
C. Mix the drug with saline solution only. 49. A patient has partial-thickness burns to
D. Flush the I.V. catheter with dextrose solution. both legs and portions of his trunk. Which of
42. After surgical repair of a hip, which of the the following I.V. fluids is given first?
following positions is best for the patients A. Albumin.
legs and hips? B. D5W.
A. Abduction. C. Lactated Ringers solution.
B. Adduction. D. 0.9% sodium chloride solution with 2 mEq of
C. Prone. potassium per 100 ml.
D. Subluxated. 50. Which of the following techniques is
43. Which of the following factors should be correct for obtaining a wound culture
the primary focus of nursing management in specimen from a surgical site?
a patient with acute pancreatitis? A. Thoroughly irrigate the wound before
A. Nutrition management. collecting the specimen.
B. Fluid and electrolyte balance. B. Use a sterile swab and wipe the crusty area
C. Management of hypoglycemia. around the outside of the wound.
C. Gently roll a sterile swab from the center of dyspnea. Which of the following actions is
the wound outward to collect drainage. the first the nurse should perform?
D. Use a sterile swab to collect drainage from A. Ask the patient to lie down on the exam table.
the dressing. B. Draw blood for chemistry panel and arterial
blood gas (ABG).
NCLEX Questions 5 C. Send the patient for a chest x-ray.
1. A nurse is administering IV furosemide to D. Check blood pressure.
a patient admitted with congestive heart 7. A clinic patient has recently been
failure. After the infusion, which of the prescribed nitroglycerin for treatment of
following symptoms is NOT expected? angina. He calls the nurse complaining of
A. Increased urinary output. frequent headaches. Which of the following
B. Decreased edema. responses to the patient is correct?
C. Decreased pain. A. Stop taking the nitroglycerin and see if the
D. Decreased blood pressure. headaches improve.
2. There are a number of risk factors B. Go to the emergency department to be
associated with coronary artery disease. checked because nitroglycerin can cause
Which of the following is a modifiable risk bleeding in the brain.
factor? C. Headaches are a frequent side effect of
A. Obesity. nitroglycerine because it causes vasodilation.
B. Heredity. D. The headaches are unlikely to be related to
C. Gender. the nitroglycerin, so you should see your doctor
D. Age. for further investigation.
3. Tissue plasminogen activator (t-PA) is 8. A patient received surgery and
considered for treatment of a patient who chemotherapy for colon cancer, completing
arrives in the emergency department therapy three (3) months previously, and she
following onset of symptoms of myocardial is now in remission. At a follow-up
infarction. Which of the following is a appointment, she complains of fatigue
contraindication for treatment with t-PA? following activity and difficulty with
A. Worsening chest pain that began earlier in concentration at her weekly bridge games.
the evening. Which of the following explanations could
B. History of cerebral hemorrhage. account for her symptoms?
C. History of prior myocardial infarction. A. The symptoms may be the result of anemia
D. Hypertension. caused by chemotherapy.
4. Following myocardial infarction, a B. The patient may be immunosuppressed.
hospitalized patient is encouraged to C. The patient may be depressed.
practice frequent leg exercises and ambulate D. The patient may be dehydrated.
in the hallway as directed by his physician. 9. A clinic patient has a hemoglobin
Which of the following choices reflects the concentration of 10.8 g/dL and reports
purpose of exercise for this patient? sticking to a strict vegetarian diet. Which of
A. Increases fitness and prevents future heart the follow nutritional advice is appropriate?
attacks. A. The diet is providing adequate sources of iron
B. Prevents bedsores. and requires no changes.
C. Prevents DVT (deep vein thrombosis). B. The patient should add meat to her diet; a
D. Prevent constipations. vegetarian diet is not advised.
5. A patient arrives in the emergency C. The patient should use iron cookware to
department with symptoms of myocardial prepare foods, such as dark-green, leafy
infarction, progressing to cardiogenic shock. vegetables and legumes, which are high in iron.
Which of the following symptoms should the D. A cup of coffee or tea should be added to
nurse expect the patient to exhibit with every meal.
cardiogenic shock? 10. A hospitalized patient is receiving packed
A. Hypertension. red blood cells (PRBCs) for treatment of
B. Bradycardia. severe anemia. Which of the following is the
C. Bounding pulse. most accurate statement?
D. Confusion. A. Transfusion reaction is most likely
6. A patient with a history of congestive heart immediately after the infusion is completed.
failure arrives at the clinic complaining of
B. PRBCs are best infused slowly through a statements by family members indicates that
20g. IV catheter. the family needs more education?
C. PRBCs should be flushed with a 5% dextrose A. We will bring in books and magazines for
solution. entertainment.
D. A nurse should remain in the room during the B. We will bring in personal care items for
first 15 minutes of infusion. comfort.
11. A patient who has received C. We will bring in fresh flowers to brighten the
chemotherapy for cancer treatment is given room.
an injection of Epoetin. Which of the D. We will bring in family pictures and get well
following should reflect the findings in a cards.
complete blood count (CBC) drawn several 17. A nurse is caring for a patient with acute
days later? lymphoblastic leukemia (ALL). Which of the
A. An increase in neutrophil count. following is the most likely age range of the
B. An increase in hematocrit. patient?
C. An increase in platelet count. A. 3-10 years.
D. An increase in serum iron. B. 25-35 years.
12. A patient is admitted to the hospital with C. 45-55 years.
suspected polycythemia vera. Which of the D. over 60 years.
following symptoms is consistent with the 18. A patient is admitted to the oncology unit
diagnosis? Select all that applies. for diagnosis of suspected Hodgkins
A. Weight loss. disease. Which of the following symptoms is
B. Increased clotting time. typical of Hodgkins disease?
C. Hypertension. A. Painful cervical lymph nodes.
D. Headaches. B. Night sweats and fatigue.
13. A nurse is caring for a patient with a C. Nausea and vomiting.
platelet count of 20,000/microliter. Which of D. Weight gain.
the following is an important intervention? 19. The Hodgkins disease patient described
A. Observe for evidence of spontaneous in the question above undergoes a lymph
bleeding. node biopsy for definitive diagnosis. If the
B. Limit visitors to family only. diagnosis of Hodgkins disease were correct,
C. Give aspirin in case of headaches. which of the following cells would the
D. Impose immune precautions. pathologist expect to find?
14. A nurse in the emergency department A. Reed-Sternberg cells.
assesses a patient who has been taking B. Lymphoblastic cells.
long-term corticosteroids to treat renal C. Gauchers cells.
disease. Which of the following is a typical D. Rieders cells
side effect of corticosteroid treatment? Note: 20. A patient is about to undergo bone
More than one answer may be correct. marrow aspiration and biopsy and expresses
A. Hypertension. fear and anxiety about the procedure. Which
B. Cushingoid features. of the following is the most effective nursing
C. Hyponatremia. response?
D. Low serum albumin. A. Warn the patient to stay very still because the
15. A nurse is caring for patients in the smallest movement will increase her pain.
oncology unit. Which of the following is the B. Encourage the family to stay in the room for
most important nursing action when caring the procedure.
for a neutropenic patient? C. Stay with the patient and focus on slow, deep
A. Change the disposable mask immediately breathing for relaxation.
after use. D. Delay the procedure to allow the patient to
B. Change gloves immediately after use. deal with her feelings.
C. Minimize patient contact. 21. A 43-year-old African American male is
D. Minimize conversation with the patient. admitted with sickle cell anemia. The nurse
16. A patient is undergoing the induction plans to assess circulation in the lower
stage of treatment for leukemia. The nurse extremities every 2 hours. Which of the
teaches family members about infectious following outcome criteria would the nurse
precautions. Which of the following use?
A. Body temperature of 99F or less
B. Toes moved in active range of motion 27. Clients with sickle cell anemia are taught
C. Sensation reported when soles of feet are to avoid activities that cause hypoxia and
touched hypoxemia. Which of the following activities
D. Capillary refill of < 3 seconds would the nurse recommend?
22. A 30-year-old male from Haiti is brought A. A family vacation in the Rocky Mountains
to the emergency department in sickle cell B. Chaperoning the local boys club on a snow-
crisis. What is the best position for this skiing trip
client? C. Traveling by airplane for business trips
A. Side-lying with knees flexed D. A bus trip to the Museum of Natural History
B. Knee-chest 28. The nurse is conducting an admission
C. High Fowlers with knees flexed assessment of a client with vitamin B12
D. Semi-Fowlers with legs extended on the bed deficiency. Which of the following would the
23. A 25-year-old male is admitted in sickle nurse include in the physical assessment?
cell crisis. Which of the following A. Palpate the spleen
interventions would be of highest priority for B. Take the blood pressure
this client? C. Examine the feet for petechiae
A. Taking hourly blood pressures with D. Examine the tongue
mechanical cuff 29. An African American female comes to the
B. Encouraging fluid intake of at least 200mL per outpatient clinic. The physician suspects
hour vitamin B12 deficiency anemia. Because
C. Position in high Fowlers with knee gatch jaundice is often a clinical manifestation of
raised this type of anemia, what body part would be
D. Administering Tylenol as ordered the best indicator?
24. Which of the following foods would the A. Conjunctiva of the eye
nurse encourage the client in sickle cell B. Soles of the feet
crisis to eat? C. Roof of the mouth
A. Peaches D. Shins
B. Cottage cheese 30. The nurse is conducting a physical
C. Popsicle assessment on a client with anemia. Which
D. Lima beans of the following clinical manifestations would
25. A newly admitted client has sickle cell be most indicative of the anemia?
crisis. The nurse is planning care based on A. BP 146/88
assessment of the client. The client is B. Respirations 28 shallow
complaining of severe pain in his feet and C. Weight gain of 10 pounds in 6 months
hands. The pulse oximetry is 92. Which of D. Pink complexion
the following interventions would be 31. The nurse is teaching the client with
implemented first? Assume that there are polycythemia vera about prevention of
orders for each intervention. complications of the disease. Which of the
A. Adjust the room temperature following statements by the client indicates a
B. Give a bolus of IV fluids need for further teaching?
C. Start O2 A. I will drink 500mL of fluid or less each day.
D. Administer meperidine (Demerol) 75 mg IV B. I will wear support hose when I am up.
push C. I will use an electric razor for shaving.
26. The nurse is instructing a client with iron- D. I will eat foods low in iron.
deficiency anemia. Which of the following 32. A 33-year-old male is being evaluated for
meal plans would the nurse expect the client possible acute leukemia. Which of the
to select? following would the nurse inquire about as a
A. Roast beef, gelatin salad, green beans, and part of the assessment?
peach pie A. The client collects stamps as a hobby.
B. Chicken salad sandwich, coleslaw, French B. The client recently lost his job as a postal
fries, ice cream worker.
C. Egg salad on wheat bread, carrot sticks, C. The client had radiation for treatment of
lettuce salad, raisin pie Hodgkins disease as a teenager.
D. Pork chop, creamed potatoes, corn, and D. The clients brother had leukemia as a child.
coconut cake 33. An African American client is admitted
with acute leukemia. The nurse is assessing
for signs and symptoms of bleeding. Where 39. A client with a pituitary tumor has had a
is the best site for examining for the transsphenoidal hypophysectomy. Which of
presence of petechiae? the following interventions would be
A. The abdomen appropriate for this client?
B. The thorax A. Place the client in Trendelenburg position for
C. The earlobes postural drainage
D. The soles of the feet B. Encourage coughing and deep breathing
34. A client with acute leukemia is admitted every 2 hours
to the oncology unit. Which of the following C. Elevate the head of the bed 30
would be most important for the nurse to D. Encourage the Valsalva maneuver for bowel
inquire? movements
A. Have you noticed a change in sleeping 40. The client with a history of diabetes
habits recently? insipidus is admitted with polyuria,
B. Have you had a respiratory infection in the polydipsia, and mental confusion. The
last 6 months? priority intervention for this client is:
C. Have you lost weight recently? A. Measure the urinary output
D. Have you noticed changes in your B. Check the vital signs
alertness? C. Encourage increased fluid intake
35. Which of the following would be the D. Weigh the client
priority nursing diagnosis for the adult client 41. A client with hemophilia has a nosebleed.
with acute leukemia? Which nursing action is most appropriate to
A. Oral mucous membrane, altered related to control the bleeding?
chemotherapy A. Place the client in a sitting position with the
B. Risk for injury related to thrombocytopenia head hyperextended
C. Fatigue related to the disease process B. Pack the nares tightly with gauze to apply
D. Interrupted family processes related to life- pressure to the source of bleeding
threatening illness of a family member C. Pinch the soft lower part of the nose for a
36. A 21-year-old male with Hodgkins minimum of 5 minutes
lymphoma is a senior at the local university. D. Apply ice packs to the forehead and back of
He is engaged to be married and is to begin a the neck
new job upon graduation. Which of the 42. A client has had a unilateral
following diagnoses would be a priority for adrenalectomy to remove a tumor. To
this client? prevent complications, the most important
A. Sexual dysfunction related to radiation measurement in the
therapy immediate postoperative period for the nurse
B. Anticipatory grieving related to terminal illness to take is:
C. Tissue integrity related to prolonged bed rest A. Blood pressure
D. Fatigue related to chemotherapy B. Temperature
37. A client has autoimmune C. Output
thrombocytopenic purpura. To determine the D. Specific gravity
clients response to treatment, the nurse 43. A client with Addisons disease has been
would monitor: admitted with a history of nausea and
A. Platelet count vomiting for the past 3 days. The client is
B. White blood cell count receiving IV glucocorticoids (Solu-Medrol).
C. Potassium levels Which of the following interventions would
D. Partial prothrombin time (PTT) the nurse implement?
38. The home health nurse is visiting a client A. Glucometer readings as ordered
with autoimmune thrombocytopenic purpura B. Intake/output measurements
(ATP). The clients platelet count currently is C. Sodium and potassium levels monitored
80, It will be most important to teach the D. Daily weights
client and family about: 44. A client had a total thyroidectomy
A. Bleeding precautions yesterday. The client is complaining of
B. Prevention of falls tingling around the mouth and in the fingers
C. Oxygen therapy and toes. What would the nurses next action
D. Conservation of energy be?
A. Obtain a crash cart
B. Check the calcium level A. Examine the perianal area with a flashlight 2
C. Assess the dressing for drainage or 3 hours after the child is asleep
D. Assess the blood pressure for hypertension B. Scrape the skin with a piece of cardboard and
45. A 32-year-old mother of three is brought bring it to the clinic
to the clinic. Her pulse is 52, there is a weight C. Obtain a stool specimen in the afternoon
gain of 30 pounds in 4 months, and the client D. Bring a hair sample to the clinic for evaluation
is wearing two sweaters. The client is 5. The nurse is teaching the mother
diagnosed with hypothyroidism. Which of regarding treatment for enterobiasis. Which
the following nursing diagnoses is of highest instruction should be given regarding the
priority? medication?
A. Impaired physical mobility related to A. Treatment is not recommended for children
decreased endurance less than 10 years of age.
B. Hypothermia r/t decreased metabolic rate B. The entire family should be treated.
C. Disturbed thought processes r/t interstitial C. Medication therapy will continue for 1 year.
edema D. Intravenous antibiotic therapy will be ordered.
D. Decreased cardiac output r/t bradycardia 6. The registered nurse is making
assignments for the day. Which client should
NCLEX Questions 6 be assigned to the pregnant nurse?
1. The client is having an arteriogram. During A. The client receiving linear accelerator
the procedure, the client tells the nurse, Im radiation therapy for lung cancer
feeling really hot. Which response would be B. The client with a radium implant for cervical
best? cancer
A. You are having an allergic reaction. I will get C. The client who has just been administered
an order for Benadryl. soluble brachytherapy for thyroid cancer
B. That feeling of warmth is normal when the D. The client who returned from placement of
dye is injected. iridium seeds for prostate cancer
C. That feeling of warmth indicates that the 7. The nurse is planning room assignments
clots in the coronary vessels are dissolving. for the day. Which client should be assigned
D. I will tell your doctor and let him explain to to a private room if only one is available?
you the reason for the hot feeling that you are A. The client with Cushings disease
experiencing. B. The client with diabetes
2. The nurse is observing several healthcare C. The client with acromegaly
workers providing care. Which action by the D. The client with myxedema
healthcare worker indicates a need for 8. The nurse caring for a client in the
further teaching? neonatal intensive care unit administers
A. The nursing assistant wears gloves while adult-strength Digitalis to the 3-pound infant.
giving the client a bath. As a result of her actions, the baby suffers
B. The nurse wears goggles while drawing blood permanent heart and brain damage. The
from the client. nurse can be charged with:
C. The doctor washes his hands before A. Negligence
examining the client. B. Tort
D. The nurse wears gloves to take the clients C. Assault
vital signs. D. Malpractice
3. The client is having electroconvulsive 9. Which assignment should not be
therapy for treatment of severe depression. performed by the licensed practical nurse?
Which of the following indicates that the A. Inserting a Foley catheter
clients ECT has been effective? B. Discontinuing a nasogastric tube
A. The client loses consciousness. C. Obtaining a sputum specimen
B. The client vomits. D. Starting a blood transfusion
C. The clients ECG indicates tachycardia. 10. The client returns to the unit from
D. The client has a grand mal seizure. surgery with a blood pressure of 90/50, pulse
4. The 5-year-old is being tested for 132, and respirations 30. Which action by the
enterobiasis (pinworms). To collect a nurse should receive priority?
specimen for assessment of pinworms, the A. Continuing to monitor the vital signs
nurse should teach the mother to: B. Contacting the physician
C. Asking the client how he feels
D. Asking the LPN to continue the post-op care C. A child whose pupils are fixed and dilated and
11. Which nurse should be assigned to care his parents, and a client with a frontal head
for the postpartum client with preeclampsia? injury
A. The RN with 2 weeks of experience in D. The client who arrives with a large puncture
postpartum wound to the abdomen and the client with chest
B. The RN with 3 years of experience in labor pain
and delivery 16. The nurse is caring for a 6-year-old client
C. The RN with 10 years of experience in admitted with a diagnosis of conjunctivitis.
surgery Before administering eye drops, the
D. The RN with 1 year of experience in the nurse should recognize that it is essential to
neonatal intensive care unit consider which of the following?
12. Which information should be reported to A. The eye should be cleansed with warm water,
the state Board of Nursing? removing any exudate, before instilling the
A. The facility fails to provide literature in both eyedrops.
Spanish and English. B. The child should be allowed to instill his own
B. The narcotic count has been incorrect on the eye drops.
unit for the past 3 days. C. The mother should be allowed to instill the
C. The client fails to receive an itemized account eyedrops.
of his bills and services received during his D. If the eye is clear from any redness or
hospital stay. edema, the eyedrops should be held.
D. The nursing assistant assigned to the client 17. The nurse is discussing meal planning
with hepatitis fails to feed the client and give the with the mother of a 2-year-old toddler.
bath. Which of the following statements, if made
13. The nurse is suspected of charting by the mother, would require a need for
medication administration that he did not further instruction?
give. After talking to the nurse, the charge A. It is okay to give my child white grape juice
nurse should: for breakfast.
A. Call the Board of Nursing B. My child can have a grilled cheese sandwich
B. File a formal reprimand for lunch.
C. Terminate the nurse C. We are going on a camping trip this
D. Charge the nurse with a tort weekend, and I have bought hot dogs to grill for
14. The home health nurse is planning for the his lunch.
days visits. Which client should be seen D. For a snack, my child can have ice cream.
first? 18. A 2-year-old toddler is admitted to the
A. The 78-year-old who had a gastrectomy 3 hospital. Which of the following nursing
weeks ago and has a PEG tube interventions would you expect?
B. The 5-month-old discharged 1 week ago with A. Ask the parent/guardian to leave the room
pneumonia who is being treated with amoxicillin when assessments are being performed.
liquid suspension B. Ask the parent/guardian to take the childs
C. The 50-year-old with MRSA being treated favorite blanket home because anything from
with Vancomycin via a PICC line the outside should not be brought into the
D. The 30-year-old with an exacerbation of hospital.
multiple sclerosis being treated with cortisone C. Ask the parent/guardian to room-in with the
via a centrally placed venous catheter child.
15. The emergency room is flooded with D. If the child is screaming, tell him this is
clients injured in a tornado. Which clients inappropriate behavior.
can be assigned to share a room in the 19. Which instruction should be given to the
emergency department during the disaster? client who is fitted for a behind-the-ear
A. A schizophrenic client having visual and hearing aid?
auditory hallucinations and the client with A. Remove the mold and clean every week.
ulcerative colitis B. Store the hearing aid in a warm place.
B. The client who is 6 months pregnant with C. Clean the lint from the hearing aid with a
abdominal pain and the client with facial toothpick.
lacerations and a broken arm D. Change the batteries weekly.
20. A priority nursing diagnosis for a child A. Fetal heart tones 160bpm
being admitted from surgery following a B. A moderate amount of straw-colored fluid
tonsillectomy is: C. A small amount of greenish fluid
A. Body image disturbance D. A small segment of the umbilical cord
B. Impaired verbal communication 27. The client is admitted to the unit. A
C. Risk for aspiration vaginal exam reveals that she is 2cm dilated.
D. Pain Which of the following statements would the
21. A client with bacterial pneumonia is nurse expect her to make?
admitted to the pediatric unit. What would A. We have a name picked out for the baby.
the nurse expect the admitting assessment B. I need to push when I have a contraction.
to reveal? C. I cant concentrate if anyone is touching me.
A. High fever D. When can I get my epidural?
B. Nonproductive cough 28. The client is having fetal heart rates of
C. Rhinitis 90110 bpm during the contractions. The
D. Vomiting and diarrhea first action the nurse should take is:
22. The nurse is caring for a client admitted A. Reposition the monitor
with epiglottitis. Because of the possibility of B. Turn the client to her left side
complete obstruction of the airway, which of C. Ask the client to ambulate
the following should the nurse have D. Prepare the client for delivery
available? 29. In evaluating the effectiveness of IV
A. Intravenous access supplies Pitocin for a client with secondary dystocia,
B. A tracheostomy set the nurse should expect:
C. Intravenous fluid administration pump A. A painless delivery
D. Supplemental oxygen B. Cervical effacement
23. A 25-year-old client with Graves disease C. Infrequent contractions
is admitted to the unit. What would the nurse D. Progressive cervical dilation
expect the admitting assessment to reveal? 30. A vaginal exam reveals a footling breech
A. Bradycardia presentation. The nurse should take which of
B. Decreased appetite the following actions at this time?
C. Exophthalmos A. Anticipate the need for a Caesarean section
D. Weight gain B. Apply the fetal heart monitor
24. The nurse is providing dietary C. Place the client in Genupectoral position
instructions to the mother of an 8-year-old D. Perform an ultrasound exam
child diagnosed with celiac disease. Which 31. A vaginal exam reveals that the cervix is
of the following foods, if selected by the 4cm dilated, with intact membranes and a
mother, would indicate her understanding of fetal heart tone rate of 160170 bpm. The
the dietary instructions? nurse decides to apply an external fetal
A. Ham sandwich on whole-wheat toast monitor. The rationale for this
B. Spaghetti and meatballs implementation is:
C. Hamburger with ketchup A. The cervix is closed.
D. Cheese omelet B. The membranes are still intact.
25. The nurse is caring for an 80-year-old C. The fetal heart tones are within normal limits.
with chronic bronchitis. Upon the morning D. The contractions are intense enough for
rounds, the nurse finds an O2 sat of 76%. insertion of an internal monitor.
Which of the following actions should the 32. The following are all nursing diagnoses
nurse take first? appropriate for a gravida 1 para 0 in labor.
A. Notify the physician Which one would be most appropriate for
B. Recheck the O2 saturation level in 15 the primigravida as she completes the early
minutes phase of labor?
C. Apply oxygen by mask A. Impaired gas exchange related to
D. Assess the pulse hyperventilation
26. A gravida 3 para 0 is admitted to the labor B. Alteration in placental perfusion related to
and delivery unit. The doctor performs an maternal position
amniotomy. Which observation would the C. Impaired physical mobility related to fetal-
nurse be expected to make after the monitoring equipment
amniotomy?
D. Potential fluid volume deficit related to B. Oral contraceptives
decreased fluid intake C. Diaphragm
33. As the client reaches 8 cm dilation, the D. Contraceptive sponge
nurse notes late decelerations on the fetal 40. The doctor suspects that the client has
monitor. The FHR baseline is 165175 bpm an ectopic pregnancy. Which symptom is
with variability of 02bpm. What is the most consistent with a diagnosis of ectopic
likely explanation of this pattern? pregnancy?
A. The baby is asleep. A. Painless vaginal bleeding
B. The umbilical cord is compressed. B. Abdominal cramping
C. There is a vagal response. C. Throbbing pain in the upper quadrant
D. There is uteroplacental insufficiency. D. Sudden, stabbing pain in the lower quadrant
34. The nurse notes variable decelerations 41. The nurse is teaching a pregnant client
on the fetal monitor strip. The most about nutritional needs during pregnancy.
appropriate initial action would be to: Which menu selection will best meet the
A. Notify her doctor nutritional needs of the pregnant client?
B. Start an IV A. Hamburger pattie, green beans, French fries,
C. Reposition the client and iced tea
D. Readjust the monitor B. Roast beef sandwich, potato chips, baked
35. Which of the following is a characteristic beans, and cola
of a reassuring fetal heart rate pattern? C. Baked chicken, fruit cup, potato salad,
A. A fetal heart rate of 170180 bpm coleslaw, yogurt, and iced tea
B. A baseline variability of 2535 bpm D. Fish sandwich, gelatin with fruit, and coffee
C. Ominous periodic changes 42. The client with hyperemesis gravidarum
D. Acceleration of FHR with fetal movements is at risk for developing:
36. The rationale for inserting a French A. Respiratory alkalosis without dehydration
catheter every hour for the client with B. Metabolic acidosis with dehydration
epidural anesthesia is: C. Respiratory acidosis without dehydration
A. The bladder fills more rapidly because of the D. Metabolic alkalosis with dehydration
medication used for the epidural. 43. A client tells the doctor that she is about
B. Her level of consciousness is such that she is 20 weeks pregnant. The most definitive sign
in a trancelike state. of pregnancy is:
C. The sensation of the bladder filling is A. Elevated human chorionic gonadotropin
diminished or lost. B. The presence of fetal heart tones
D. She is embarrassed to ask for the bedpan C. Uterine enlargement
that frequently. D. Breast enlargement and tenderness
37. A client in the family planning clinic asks 44. The nurse is caring for a neonate whose
the nurse about the most likely time for her mother is diabetic. The nurse will expect the
to conceive. The nurse explains that neonate to be:
conception is most likely to occur when: A. Hypoglycemic, small for gestational age
A. Estrogen levels are low. B. Hyperglycemic, large for gestational age
B. Luteinizing hormone is high. C. Hypoglycemic, large for gestational age
C. The endometrial lining is thin. D. Hyperglycemic, small for gestational age
D. The progesterone level is low. 45. Which of the following instructions
38. A client tells the nurse that she plans to should be included in the nurses teaching
use the rhythm method of birth control. The regarding oral contraceptives?
nurse is aware that the success of the A. Weight gain should be reported to the
rhythm method depends on the: physician.
A. Age of the client B. An alternate method of birth control is needed
B. Frequency of intercourse when taking antibiotics.
C. Regularity of the menses C. If the client misses one or more pills, two pills
D. Range of the clients temperature should be taken per day for 1 week.
39. A client with diabetes asks the nurse for D. Changes in the menstrual flow should be
advice regarding methods of birth control. reported to the physician.
Which method of birth control is most 46. The nurse is discussing breastfeeding
suitable for the client with diabetes? with a postpartum client. Breastfeeding is
A. Intrauterine device
contraindicated in the postpartum client on the nurses assessment the client is in
with: which phase of labor?
A. Diabetes A. Active
B. Positive HIV B. Latent
C. Hypertension C. Transition
D. Thyroid disease D. Early
47. A client is admitted to the labor and 3. A newborn with narcotic abstinence
delivery unit complaining of vaginal bleeding syndrome is admitted to the nursery.
with very little discomfort. The nurses first Nursing care of the newborn should include:
action should be to: A. Teaching the mother to provide tactile
A. Assess the fetal heart tones stimulation
B. Check for cervical dilation B. Wrapping the newborn snugly in a blanket
C. Check for firmness of the uterus C. Placing the newborn in the infant seat
D. Obtain a detailed history D. Initiating an early infant-stimulation program
48. A client telephones the emergency room 4. A client elects to have epidural anesthesia
stating that she thinks that she is in labor. to relieve the discomfort of labor. Following
The nurse should tell the client that labor has the initiation of epidural anesthesia, the
probably begun when: nurse should give priority to:
A. Her contractions are 2 minutes apart. A. Checking for cervical dilation
B. She has back pain and a bloody discharge. B. Placing the client in a supine position
C. She experiences abdominal pain and C. Checking the clients blood pressure
frequent urination. D. Obtaining a fetal heart rate
D. Her contractions are 5 minutes apart. 5. The nurse is aware that the best way to
49. The nurse is teaching a group of prenatal prevent postoperative wound infection in the
clients about the effects of cigarette smoke surgical client is to:
on fetal development. Which characteristic is A. Administer a prescribed antibiotic
associated with babies born to mothers who B. Wash her hands for 2 minutes before care
smoked during pregnancy? C. Wear a mask when providing care
A. Low birth weight D. Ask the client to cover her mouth when she
B. Large for gestational age coughs
C. Preterm birth, but appropriate size for 6. The elderly client is admitted to the
gestation emergency room. Which symptom is the
D. Growth retardation in weight and length client with a fractured hip most likely to
50. The physician has ordered an injection of exhibit?
RhoGam for the postpartum client whose A. Pain
blood type is A negative but whose baby is O B. Disalignment
positive. To provide postpartum prophylaxis, C. Cool extremity
RhoGam should be administered: D. Absence of pedal pulses
A. Within 72 hours of delivery 7. The nurse knows that a 60-year-old female
B. Within 1 week of delivery clients susceptibility to osteoporosis is
C. Within 2 weeks of delivery most likely related to:
D. Within 1 month of delivery A. Lack of exercise
B. Hormonal disturbances
NCLEX Questions 7 C. Lack of calcium
1. After the physician performs an D. Genetic predisposition
amniotomy, the nurses first action should 8. A 2-year-old is admitted for repair of a
be to assess the: fractured femur and is placed in Bryants
A. Degree of cervical dilation traction. Which finding by the nurse
B. Fetal heart tones indicates that the traction is working
C. Clients vital signs properly?
D. Clients level of discomfort A. The infant no longer complains of pain.
2. A client is admitted to the labor and B. The buttocks are 15 off the bed.
delivery unit. The nurse performs a vaginal C. The legs are suspended in the traction.
exam and determines that the clients cervix D. The pins are secured within the pulley.
is 5 cm dilated with 75% effacement. Based 9. A client with a fractured hip has been
placed in Bucks traction. Which statement is
true regarding balanced skeletal traction? 15. An elderly client with an abdominal
Balanced skeletal traction: surgery is admitted to the unit following
A. Utilizes a Steinman pin surgery. In anticipation of complications of
B. Requires that both legs be secured anesthesia and narcotic administration, the
C. Utilizes Kirschner wires nurse should:
D. Is used primarily to heal the fractured hips A. Administer oxygen via nasal cannula
10. The client is admitted for an open B. Have narcan (naloxone) available
reduction internal fixation of a fractured hip. C. Prepare to administer blood products
Immediately following surgery, the nurse D. Prepare to do cardio resuscitation
should give priority to assessing the: 16. Which roommate would be most suitable
A. Serum collection (Davol) drain for the 6-year-old male with a fractured femur
B. Clients pain in Russells traction?
C. Nutritional status A. 16-year-old female with scoliosis
D. Immobilizer B. 12-year-old male with a fractured femur
11. Which statement made by the family C. 10-year-old male with sarcoma
member caring for the client with a D. 6-year-old male with osteomyelitis
percutaneous gastrostomy tube indicates 17. A client with osteoarthritis has a
understanding of the nurses teaching? prescription for Celebrex (celecoxib). Which
A. I must flush the tube with water after instruction should be included in the
feedings and clamp the tube. discharge teaching?
B. I must check placement four times per day. A. Take the medication with milk.
C. I will report to the doctor any signs of B. Report chest pain.
indigestion. C. Remain upright after taking for 30 minutes.
D. If my father is unable to swallow, I will D. Allow 6 weeks for optimal effects.
discontinue the feeding and call the clinic. 18. A client with a fractured tibia has a
12. The nurse is assessing the client with a plaster-of-Paris cast applied to immobilize
total knee replacement 2 the fracture. Which action by the nurse
hours postoperative. Which information indicates understanding of a plaster-of-Paris
requires notification of the doctor? cast? The nurse:
A. Bleeding on the dressing is 3cm in diameter. A. Handles the cast with the fingertips
B. The client has a temperature of 100.6F B. Petals the cast
(38.1C). C. Dries the cast with a hair dryer
C. The clients hematocrit is 26%. D. Allows 24 hours before bearing weight
D. The urinary output has been 60 during the 19. The teenager with a fiberglass cast asks
last 2 hours. the nurse if it will be okay to allow his friends
13. The nurse is caring for the client with a 5- to autograph his cast. Which response would
year-old diagnosis of plumbism. Which be best?
information in the health history is most A. It will be alright for your friends to autograph
likely related to the development of the cast.
plumbism? B. Because the cast is made of plaster,
A. The client has traveled out of the country in autographing can weaken the cast.
the last 6 months. C. If they dont use chalk to autograph, it is
B. The clients parents are skilled stained-glass okay.
artists. D. Autographing or writing on the cast in any
C. The client lives in a house built in 1 form will harm the cast.
D. The client has several brothers and sisters. 20. The nurse is assigned to care for the
14. A client with a total hip replacement client with a Steinmann pin. During pin care,
requires special equipment. Which she notes that the LPN uses sterile gloves
equipment would assist the client with a total and Q-tips to clean the pin. Which action
hip replacement with activities of daily should the nurse take at this time?
living? A. Assisting the LPN with opening sterile
A. High-seat commode packages and peroxide
B. Recliner B. Telling the LPN that clean gloves are allowed
C. TENS unit C. Telling the LPN that the registered nurse
D. Abduction pillow should perform pin care
D. Asking the LPN to clean the weights and D. Ask the doctor to perform a complete blood
pulleys with peroxide count before starting the medication.
21. A child with scoliosis has a spica cast 2. The client is admitted to the hospital with
applied. Which action specific to the spica hypertensive crises. Diazoxide (Hyperstat) is
cast should be taken? ordered. During administration, the nurse
A. Check the bowel sounds should:
B. Assess the blood pressure A. Utilize an infusion pump
C. Offer pain medication B. Check the blood glucose level
D. Check for swelling C. Place the client in Trendelenburg position
22. The client with a cervical fracture is D. Cover the solution with foil
placed in traction. Which type of traction will 3. The 6-month-old client with a ventral
be utilized at the time of discharge? septal defect is receiving Digitalis for
A. Russells traction regulation of his heart rate. Which finding
B. Bucks traction should be reported to the doctor?
C. Halo traction A. Blood pressure of 126/80
D. Crutchfield tong traction B. Blood glucose of 110 mg/dL
23. A client with a total knee replacement has C. Heart rate of 60 bpm
a CPM (continuous passive motion device) D. Respiratory rate of 30 per minute
applied during the post-operative period. 4. The client admitted with angina is given a
Which statement made by the nurse prescription for nitroglycerin. The client
indicates understanding of the CPM should be instructed to:
machine? A. Replenish his supply every 3 months
A. Use of the CPM will permit the client to B. Take one every 15 minutes if pain occurs
ambulate during the therapy. C. Leave the medication in the brown bottle
B. The CPM machine controls should be D. Crush the medication and take with water
positioned distal to the site. 5. The client is instructed regarding foods
C. If the client complains of pain during the that are low in fat and cholesterol. Which diet
therapy, I will turn off the machine and call the selection is lowest in saturated fats?
doctor. A. Macaroni and cheese
D. Use of the CPM machine will alleviate the B. Shrimp with rice
need for physical therapy after the client is C. Turkey breast
discharged. D. Spaghetti
24. A client with a fractured hip is being 6. The client is admitted with left-sided
taught correct use of the walker. The nurse is congestive heart failure. In assessing the
aware that the correct use of the walker is client for edema, the nurse should check the:
achieved if the: A. Feet
A. Palms rest lightly on the handles B. Neck
B. Elbows are flexed 0 C. Hands
C. Client walks to the front of the walker D. Sacrum
D. Client carries the walker 7. The nurse is checking the clients central
25. When assessing a laboring client, the venous pressure. The nurse should place the
nurse finds a prolapsed cord. The nurse zero of the manometer at the:
should: A. Phlebostatic axis
A. Attempt to replace the cord B. PMI
B. Place the client on her left side C. Erbs point
C. Elevate the clients hips D. Tail of Spence
D. Cover the cord with a dry, sterile gauze 8. The physician orders lisinopril (Zestril)
and furosemide (Lasix) to be administered
NCLEX Questions 8 concomitantly to the client with
1. The client presents to the clinic with a hypertension. The nurse should:
serum cholesterol of 275 mg/dL and is A. Question the order
placed on rosuvastatin (Crestor). Which B. Administer the medications
instruction should be given to the client? C. Administer separately
A. Report muscle weakness to the physician. D. Contact the pharmacy
B. Allow six months for the drug to take effect. 9. The best method of evaluating the amount
C. Take the medication with fruit juice. of peripheral edema is:
A. Weighing the client daily administering leucovorin calcium to a client
B. Measuring the extremity receiving Trimetrexate is to:
C. Measuring the intake and output A. Treat iron-deficiency anemia caused by
D. Checking for pitting chemotherapeutic agents
10. A client with vaginal cancer is being B. Create a synergistic effect that shortens
treated with a radioactive vaginal implant. treatment time
The clients husband asks the nurse if he can C. Increase the number of circulating neutrophils
spend the night with his wife. The nurse D. Reverse drug toxicity and prevent tissue
should explain that: damage
A. Overnight stays by family members is against 15. A 4-month-old is brought to the well-baby
hospital policy. clinic for immunization. In addition to the
B. There is no need for him to stay because DPT and polio vaccines, the baby should
staffing is adequate. receive:
C. His wife will rest much better knowing that he A. HibTITER
is at home. B. Mumps vaccine
D. Visitation is limited to 30 minutes when the C. Hepatitis B vaccine
implant is in place. D. MMR
11. The nurse is caring for a client 16. The physician has prescribed Nexium
hospitalized with a facial stroke. Which diet (esomeprazole) for a client with erosive
selection would be suited to the client? gastritis. The nurse should administer the
A. Roast beef sandwich, potato chips, pickle medication:
spear, iced tea A. 30 minutes before meals
B. Split pea soup, mashed potatoes, pudding, B. With each meal
milk C. In a single dose at bedtime
C. Tomato soup, cheese toast, Jello, coffee D. 30 minutes after meals
D. Hamburger, baked beans, fruit cup, iced tea 17. A client on the psychiatric unit is in an
12. The physician has uncontrolled rage and is threatening other
prescribed Novolog insulin for a client with clients and staff. What is the most
diabetes mellitus. Which statement indicates appropriate action for the nurse to take?
that the client knows when the peak action of A. Call security for assistance and prepare to
the insulin occurs? sedate the client.
A. I will make sure I eat breakfast within 10 B. Tell the client to calm down and ask him if he
minutes of taking my insulin. would like to play cards.
B. I will need to carry candy or some form of C. Tell the client that if he continues his behavior
sugar with me all the time. he will be punished.
C. I will eat a snack around three oclock each D. Leave the client alone until he calms down.
afternoon. 18. When the nurse checks the fundus of a
D. I can save my dessert from supper for a client on the first postpartum day, she notes
bedtime snack. that the fundus is firm, is at the level of the
13. The nurse is teaching basic infant care to umbilicus, and is displaced to the right. The
a group of first-time parents. The nurse next action the nurse should take is to:
should explain that a sponge bath is A. Check the client for bladder distention
recommended for the first 2 weeks of life B. Assess the blood pressure for hypotension
because: C. Determine whether an oxytocic drug was
A. New parents need time to learn how to hold given
the baby. D. Check for the expulsion of small clots
B. The umbilical cord needs time to separate. 19. A client is admitted to the hospital with a
C. Newborn skin is easily traumatized by temperature of 99.8F, complaints of blood-
washing. tinged hemoptysis, fatigue, and night
D. The chance of chilling the baby outweighs the sweats. The clients symptoms are
benefits of bathing. consistent with a diagnosis of:
14. A client with leukemia is receiving A. Pneumonia
Trimetrexate. After reviewing the clients B. Reaction to antiviral medication
chart, the physician orders Wellcovorin C. Tuberculosis
(leucovorin calcium). The rationale for D. Superinfection due to low CD4 count
20. The client is seen in the clinic for disease. Which side effect is most often
treatment of migraine headaches. The drug associated with this drug?
Imitrex (sumatriptan succinate) is prescribed A. Urinary incontinence
for the client. Which of the following in the B. Headaches
clients history should be reported to the C. Confusion
doctor? D. Nausea
A. Diabetes 26. A client is admitted to the labor and
B. Prinzmetals angina delivery unit in active labor. During
C. Cancer examination, the nurse notes a papular
D. Cluster headaches lesion on the perineum. Which initial action
21. The client with suspected meningitis is is most appropriate?
admitted to the unit. The doctor is A. Document the finding
performing an assessment to determine B. Report the finding to the doctor
meningeal irritation and spinal nerve root C. Prepare the client for a C-section
inflammation. A positive Kernigs sign is D. Continue primary care as prescribed
charted if the nurse notes: 27. A client with a diagnosis of HPV is at risk
A. Pain on flexion of the hip and knee for which of the following?
B. Nuchal rigidity on flexion of the neck A. Hodgkins lymphoma
C. Pain when the head is turned to the left side B. Cervical cancer
D. Dizziness when changing positions C. Multiple myeloma
22. The client with Alzheimers disease is D. Ovarian cancer
being assisted with activities of daily living 28. During the initial interview, the client
when the nurse notes that the client uses her reports that she has a lesion on the
toothbrush to brush her hair. The nurse is perineum. Further investigation reveals a
aware that the client is exhibiting: small blister on the vulva that is painful to
A. Agnosia touch. The nurse is aware that the most
B. Apraxia likely source of the lesion is:
C. Anomia A. Syphilis
D. Aphasia B. Herpes
23. The client with dementia is experiencing C. Gonorrhea
confusion late in the afternoon and before D. Condylomata
bedtime. The nurse is aware that the client is 29. A client visiting a family planning clinic is
experiencing what is known as: suspected of having an STI. The best
A. Chronic fatigue syndrome diagnostic test for treponema pallidum is:
B. Normal aging A. Venereal Disease Research Lab (VDRL)
C. Sundowning B. Rapid plasma reagin (RPR)
D. Delusions C. Florescent treponemal antibody (FTA)
24. The client with confusion says to the D. Thayer-Martin culture (TMC)
nurse, I havent had anything to eat all day 30. A 15-year-old primigravida is admitted
long. When are they going to bring with a tentative diagnosis of HELLP
breakfast? The nurse saw the client in the syndrome. Which laboratory finding is
day room eating breakfast with other clients associated with HELLP syndrome?
30 minutes before this conversation. Which A. Elevated blood glucose
response would be best for the nurse to B. Elevated platelet count
make? C. Elevated creatinine clearance
A. You know you had breakfast 30 minutes D. Elevated hepatic enzymes
ago. 31. The nurse is assessing the deep tendon
B. I am so sorry that they didnt get you reflexes of a client with preeclampsia. Which
breakfast. Ill report it to the charge nurse. method is used to elicit the biceps reflex?
C. Ill get you some juice and toast. Would you A. The nurse places her thumb on the muscle
like something else? inset in the antecubital space and taps the
D. You will have to wait a while; lunch will be thumb briskly with the reflex hammer.
here in a little while. B. The nurse loosely suspends the clients arm
25. The doctor has prescribed Exelon in an open hand while tapping the back of the
(rivastigmine) for the client with Alzheimers clients elbow.
C. The nurse instructs the client to dangle her C. Alteration in skin integrity
legs as the nurse strikes the area below the D. Ineffective individual coping
patella with the blunt side of the reflex hammer. 38. The nurse is caring for a client with
D. The nurse instructs the client to place her ascites. Which is the best method to use for
arms loosely at her side as the nurse strikes the determining early ascites?
muscle insert just above the wrist. A. Inspection of the abdomen for enlargement
32. A primigravida with diabetes is admitted B. Bimanual palpation for hepatomegaly
to the labor and delivery unit at 34 weeks C. Daily measurement of abdominal girth
gestation. Which doctors order should the D. Assessment for a fluid wave
nurse question? 39. The client arrives in the emergency
A. Magnesium sulfate 4gm (25%) IV department after a motor vehicle accident.
B. Brethine 10 mcg IV Nursing assessment findings include BP
C. Stadol 1 mg IV push every 4 hours as needed 80/34, pulse rate 120, and respirations 20.
prn for pain Which is the clients most appropriate
D. Ancef 2gm IVPB every 6 hours priority nursing diagnosis?
33. A diabetic multigravida is scheduled for A. Alteration in cerebral tissue perfusion
an amniocentesis at 32 weeks gestation to B. Fluid volume deficit
determine the L/S ratio and phosphatidyl C. Ineffective airway clearance
glycerol level. The L/S ratio is 1:1 and the D. Alteration in sensory perception
presence of phosphatidylglycerol is noted. 40. The home health nurse is visiting an 18-
The nurses assessment of this data is: year-old with osteogenesis imperfecta.
A. The infant is at low risk for congenital Which information obtained on the visit
anomalies. would cause the most concern? The client:
B. The infant is at high risk for intrauterine A. Likes to play football
growth retardation. B. Drinks several carbonated drinks per day
C. The infant is at high risk for respiratory C. Has two sisters with sickle cell tract
distress syndrome. D. Is taking acetaminophen to control pain
D. The infant is at high risk for birth trauma. 41. The nurse working the organ transplant
34. Which observation in the newborn of a unit is caring for a client with a white blood
diabetic mother would require immediate cell count of During evening visitation, a
nursing intervention? visitor brings a basket of fruit. What action
A. Crying should the nurse take?
B. Wakefulness A. Allow the client to keep the fruit
C. Jitteriness B. Place the fruit next to the bed for easy access
D. Yawning by the client
35. The nurse caring for a client receiving C. Offer to wash the fruit for the client
intravenous magnesium sulfate must closely D. Tell the family members to take the fruit home
observe for side effects associated with drug 42. The nurse is caring for the client
therapy. An expected side effect of following a laryngectomy when suddenly the
magnesium sulfate is: client becomes nonresponsive and pale, with
A. Decreased urinary output a BP of 90/40 systolic. The initial nurses
B. Hypersomnolence action should be to:
C. Absence of knee jerk reflex A. Place the client in Trendelenburg position
D. Decreased respiratory rate B. Increase the infusion of Dextrose in normal
36. The client has elected to have epidural saline
anesthesia to relieve labor pain. If the client C. Administer atropine intravenously
experiences hypotension, the nurse would: D. Move the emergency cart to the bedside
A. Place her in Trendelenburg position 43. The client admitted 2 days earlier with a
B. Decrease the rate of IV infusion lung resection accidentally pulls out the
C. Administer oxygen per nasal cannula chest tube. Which action by the nurse
D. Increase the rate of the IV infusion indicates understanding of the management
37. A client has cancer of the pancreas. The of chest tubes?
nurse should be most concerned about A. Order a chest x-ray
which nursing diagnosis? B. Reinsert the tube
A. Alteration in nutrition C. Cover the insertion site with a Vaseline gauze
B. Alteration in bowel elimination D. Call the doctor
44. A client being treated with sodium B. The clients hemoglobin and hematocrit
warfarin has a Protime of 120 seconds. improve.
Which intervention would be most important C. The clients tissue turgor improves.
to include in the nursing care plan? D. The client gains weight.
A. Assess for signs of abnormal bleeding 50. The client is admitted following repair of
B. Anticipate an increase in the Coumadin a fractured tibia and cast application. Which
dosage nursing assessment should be reported to
C. Instruct the client regarding the drug therapy the doctor?
D. Increase the frequency of neurological A. Pain beneath the cast
assessments B. Warm toes
45. Which selection would provide the most C. Pedal pulses weak and rapid
calcium for the client who is 4 months D. Paresthesia of the toes
pregnant?
A. A granola bar NCLEX Questions 9
B. A bran muffin 1. A client with a history of abusing
C. A cup of yogurt barbiturates abruptly stops taking the
D. A glass of fruit juice medication. The nurse should give priority to
46. The client with preeclampsia is admitted assessing the client for:
to the unit with an order for magnesium A. Depression and suicidal ideation
sulfate. Which action by the nurse indicates B. Tachycardia and diarrhea
understanding of the possible side effects of C. Muscle cramping and abdominal pain
magnesium sulfate? D. Tachycardia and euphoric mood
A. The nurse places a sign over the bed not to 2. During the assessment of a laboring client,
check blood pressure in the right arm. the nurse notes that the FHT are loudest in
B. The nurse places a padded tongue blade at the upper-right quadrant. The infant is most
the bedside. likely in which position?
C. The nurse inserts a Foley catheter. A. Right breech presentation
D. The nurse darkens the room. B. Right occiput anterior presentation
47. A 6-year-old client is admitted to the unit C. Left sacral anterior presentation
with a hemoglobin of 6g/dL. The physician D. Left occipital transverse presentation
has written an order to transfuse 2 units of 3. The primary physiological alteration in the
whole blood. When discussing the treatment, development of asthma is:
the childs mother tells the nurse that she A. Bronchiolar inflammation and dyspnea
does not believe in having blood B. Hypersecretion of abnormally viscous mucus
transfusions and that she will not allow her C. Infectious processes causing mucosal edema
child to have the treatment. What nursing D. Spasm of bronchial smooth muscle
action is most appropriate? 4. A client with mania is unable to finish her
A. Ask the mother to leave while the blood dinner. To help her maintain sufficient
transfusion is in progress nourishment, the nurse should:
B. Encourage the mother to reconsider A. Serve high-calorie foods she can carry with
C. Explain the consequences without treatment her
D. Notify the physician of the mothers refusal B. Encourage her appetite by sending out for her
48. A client is admitted to the unit 2 hours favorite foods
after an explosion causes burns to the face. C. Serve her small, attractively arranged
The nurse would be most concerned with the portions
client developing which of the following? D. Allow her in the unit kitchen for extra food
A. Hypovolemia whenever she pleases
B. Laryngeal edema 5. To maintain Bryants traction, the nurse
C. Hypernatremia must make certain that the childs:
D. Hyperkalemia A. Hips are resting on the bed, with the legs
49. The nurse is evaluating nutritional suspended at a right angle to the bed
outcomes for a with anorexia nervosa. Which B. Hips are slightly elevated above the bed and
data best indicates that the plan of care is the legs are suspended at a right angle to the
effective? bed
A. The client selects a balanced diet from the
menu.
C. Hips are elevated above the level of the body B. Perform a pregnancy test
on a pillow and the legs are suspended parallel C. Monitor apical pulse
to the bed D. Obtain a creatinine level
D. Hips and legs are flat on the bed, with the 13. A client with AIDS is taking Zovirax
traction positioned at the foot of the bed (acyclovir). Which nursing intervention is
6. Which action by the nurse indicates most critical during the administration of
understanding of herpes zoster? acyclovir?
A. The nurse covers the lesions with a sterile A. Limit the clients activity
dressing. B. Encourage a high-carbohydrate diet
B. The nurse wears gloves when providing care. C. Utilize an incentive spirometer to improve
C. The nurse administers a prescribed antibiotic. respiratory function
D. The nurse administers oxygen. D. Encourage fluids
7. The client has an order for a trough to be 14. A client is admitted for an MRI. The nurse
drawn on the client receiving Vancomycin. should question the client regarding:
The nurse is aware that the nurse should A. Pregnancy
contact the lab for them to collect the blood: B. A titanium hip replacement
A. 15 minutes after the infusion C. Allergies to antibiotics
B. 30 minutes before the infusion D. Inability to move his feet
C. 1 hour after the infusion 15. The nurse is caring for the client
D. 2 hours after the infusion receiving Amphotericin B. Which of the
8. The client using a diaphragm should be following indicates that the client has
instructed to: experienced toxicity to this drug?
A. Refrain from keeping the diaphragm in longer A. Changes in vision
than 4 hours B. Nausea
B. Keep the diaphragm in a cool location C. Urinary frequency
C. Have the diaphragm resized if she gains 5 D. Changes in skin color
pounds 16. The nurse should visit which of the
D. Have the diaphragm resized if she has any following clients first?
surgery A. The client with diabetes with a blood glucose
9. The nurse is providing postpartum of 95mg/dL
teaching for a mother planning to breastfeed B. The client with hypertension being maintained
her infant. Which of the clients statements on Lisinopril
indicates the need for additional teaching? C. The client with chest pain and a history of
A. Im wearing a support bra. angina
B. Im expressing milk from my breast. D. The client with Raynauds disease
C. Im drinking four glasses of fluid during a 24- 17. A client with cystic fibrosis is taking
hour period. pancreatic enzymes. The nurse should
D. While Im in the shower, Ill allow the water to administer this medication:
run over my breasts. A. Once per day in the morning
10. Damage to the VII cranial nerve results in: B. Three times per day with meals
A. Facial pain C. Once per day at bedtime
B. Absence of ability to smell D. Four times per day
C. Absence of eye movement 18. Cataracts result in opacity of the
D. Tinnitus crystalline lens. Which of the following best
11. A client is receiving Pyridium explains the functions of the lens?
(phenazopyridine hydrochloride) for a A. The lens controls stimulation of the retina.
urinary tract infection. The client should be B. The lens orchestrates eye movement.
taught that the medication may: C. The lens focuses light rays on the retina.
A. Cause diarrhea D. The lens magnifies small objects.
B. Change the color of her urine 19. A client who has glaucoma is to have
C. Cause mental confusion miotic eye drops instilled in both eyes. The
D. Cause changes in taste nurse knows that the purpose of the
12. Which of the following tests should be medication is to:
performed before beginning a prescription of A. Anesthetize the cornea
Accutane? B. Dilate the pupils
A. Check the calcium level C. Constrict the pupils
D. Paralyze the muscles of accommodation emergency room, a chest tube is inserted.
20. A client with a severe corneal ulcer has Which of the following explains the primary
an order for Gentamycin gtt. q 4 hours and rationale for insertion of chest tubes?
Neomycin 1 gtt q 4 hours. Which of the A. The tube will allow for equalization of the lung
following schedules should be used when expansion.
administering the drops? B. Chest tubes serve as a method of draining
A. Allow 5 minutes between the two blood and serous fluid and assist in reinflating
medications. the lungs.
B. The medications may be used together. C. Chest tubes relieve pain associated with a
C. The medications should be separated by a collapsed lung.
cycloplegic drug. D. Chest tubes assist with cardiac function by
D. The medications should not be used in the stabilizing lung expansion.
same client. 27. A client who delivered this morning tells
21. The client with color blindness will most the nurse that she plans to breastfeed her
likely have problems distinguishing which of baby. The nurse is aware that successful
the following colors? breastfeeding is most dependent on the:
A. Orange A. Mothers educational level
B. Violet B. Infants birth weight
C. Red C. Size of the mothers breast
D. White D. Mothers desire to breastfeed
22. The client with a pacemaker should be 28. The nurse is monitoring the progress of a
taught to: client in labor. Which finding should be
A. Report ankle edema reported to the physician immediately?
B. Check his blood pressure daily A. The presence of scant bloody discharge
C. Refrain from using a microwave oven B. Frequent urination
D. Monitor his pulse rate C. The presence of green-tinged amniotic fluid
23. The client with enuresis is being taught D. Moderate uterine contractions
regarding bladder retraining. The nurse 29. The nurse is measuring the duration of
should advise the client to refrain from the clients contractions. Which statement is
drinking after: true regarding the measurement of the
A. 1900 duration of contractions?
B. 1200 A. Duration is measured by timing from the
C. 1000 beginning of one contraction to the beginning of
D. 0700 the next contraction.
24. Which of the following diet instructions B. Duration is measured by timing from the end
should be given to the client with recurring of one contraction to the beginning of the next
urinary tract infections? contraction.
A. Increase intake of meats. C. Duration is measured by timing from the
B. Avoid citrus fruits. beginning of one contraction to the end of the
C. Perform pericare with hydrogen peroxide. same contraction.
D. Drink a glass of cranberry juice every day. D. Duration is measured by timing from the peak
25. The physician has prescribed NPH of one contraction to the end of the same
insulin for a client with diabetes mellitus. contraction.
Which statement indicates that the client 30. The physician has ordered an
knows when the peak action of the insulin intravenous infusion of Pitocin for the
occurs? induction of labor. When caring for the
A. I will make sure I eat breakfast within 2 hours obstetric client receiving intravenous
of taking my insulin. Pitocin, the nurse should monitor for:
B. I will need to carry candy or some form of A. Maternal hypoglycemia
sugar with me all the time. B. Fetal bradycardia
C. I will eat a snack around three oclock each C. Maternal hyperreflexia
afternoon. D. Fetal movement
D. I can save my dessert from supper for a 31. A client with diabetes visits the prenatal
bedtime snack. clinic at 28 weeks gestation. Which
26. The nurse is caring for a 30-year-old male statement is true regarding insulin needs
admitted with a stab wound. While in the during pregnancy?
A. Insulin requirements moderate as the A. Because it is a state law
pregnancy progresses. B. To detect cardiovascular defects
B. A decreased need for insulin occurs during C. Because of her age
the second trimester. D. To detect neurological defects
C. Elevations in human chorionic gonadotrophin 38. A client with hypothyroidism asks the
decrease the need for insulin. nurse if she will still need to take thyroid
D. Fetal development depends on adequate medication during the pregnancy. The
insulin regulation. nurses response is based on the knowledge
32. A client in the prenatal clinic is assessed that:
to have a blood pressure of 180/96. The A. There is no need to take thyroid medication
nurse should give priority to: because the fetuss thyroid produces a thyroid-
A. Providing a calm environment stimulating hormone.
B. Obtaining a diet history B. Regulation of thyroid medication is more
C. Administering an analgesic difficult because the thyroid gland increases in
D. Assessing fetal heart tones size during pregnancy.
33. A primigravida, age 42, is 6 weeks C. It is more difficult to maintain thyroid
pregnant. Based on the clients age, her regulation during pregnancy due to a slowing of
infant is at risk for: metabolism.
A. Down syndrome D. Fetal growth is arrested if thyroid medication
B. Respiratory distress syndrome is continued during pregnancy.
C. Turners syndrome 39. The nurse is responsible for performing a
D. Pathological jaundice neonatal assessment on a full-term infant. At
34. A client with a missed abortion at 29 1 minute, the nurse could expect to find:
weeks gestation is admitted to the hospital. A. An apical pulse of 100
The client will most likely be treated with: B. An absence of tonus
A. Magnesium sulfate C. Cyanosis of the feet and hands
B. Calcium gluconate D. Jaundice of the skin and sclera
C. Dinoprostone (Prostin E.) 40. A client with sickle cell anemia is
D. Bromocriptine (Parlodel) admitted to the labor and delivery unit during
35. A client with preeclampsia has been the first phase of labor. The nurse should
receiving an infusion containing magnesium anticipate the clients need for:
sulfate for a blood pressure that is 160/80; A. Supplemental oxygen
deep tendon reflexes are 1 plus, and the B. Fluid restriction
urinary output for the past hour is 100mL. C. Blood transfusion
The nurse should: D. Delivery by Caesarean section
A. Continue the infusion of magnesium sulfate 41. A client with diabetes has an order for
while monitoring the clients blood pressure ultrasonography. Preparation for an
B. Stop the infusion of magnesium sulfate and ultrasound includes:
contact the physician A. Increasing fluid intake
C. Slow the infusion rate and turn the client on B. Limiting ambulation
her left side C. Administering an enema
D. Administer calcium gluconate IV push and D. Withholding food for 8 hours
continue to monitor the blood pressure 42. An infant who weighs 8 pounds at birth
36. Which statement made by the nurse would be expected to weigh how many
describes the inheritance pattern of pounds at 1 year?
autosomal recessive disorders? A. 14 pounds
A. An affected newborn has unaffected parents. B. 16 pounds
B. An affected newborn has one affected parent. C. 18 pounds
C. Affected parents have a one in four chance of D. 24 pounds
passing on the defective gene. 43. A pregnant client with a history of alcohol
D. Affected parents have unaffected children addiction is scheduled for a nonstress test.
who are carriers. The nonstress test:
37. A pregnant client, age 32, asks the nurse A. Determines the lung maturity of the fetus
why her doctor has recommended a serum B. Measures the activity of the fetus
alpha fetoprotein. The nurse should explain C. Shows the effect of contractions on the fetal
that the doctor has recommended the test: heart rate
D. Measures the neurological well-being of the that the doctor has ordered continuous
fetus observation because:
44. A full-term male has hypospadias. Which A. Hallucinogenic drugs create both stimulant
statement describes hypospadias? and depressant effects.
A. The urethral opening is absent. B. Hallucinogenic drugs induce a state of altered
B. The urethra opens on the dorsal side of the perception.
penis. C. Hallucinogenic drugs produce severe
C. The penis is shorter than usual. respiratory depression.
D. The urethral meatus opens on the underside D. Hallucinogenic drugs induce rapid physical
of the penis. dependence.
45. A gravida 3 para 2 is admitted to the labor
unit. Vaginal exam reveals that the clients NCLEX Questions 10
cervix is 8 cm dilated, with complete 1. A patient arrives at the emergency
effacement. The priority nursing diagnosis at department complaining of midsternal chest
this time is: pain. Which of the following nursing action
A. Alteration in coping related to pain should take priority?
B. Potential for injury related to precipitate A. A complete history with emphasis on
delivery preceding events.
C. Alteration in elimination related to anesthesia B. An electrocardiogram.
D. Potential for fluid volume deficit related to C. Careful assessment of vital signs.
NPO status D. Chest exam with auscultation.
46. The client with varicella will most likely 2. A patient has been hospitalized with
have an order for which category of pneumonia and is about to be discharged. A
medication? nurse provides discharge instructions to a
A. Antibiotics patient and his family. Which
B. Antipyretics misunderstanding by the family indicates the
C. Antivirals need for more detailed information?
D. Anticoagulants A. The patient may resume normal home
47. A client is admitted complaining of chest activities as tolerated but should avoid physical
pain. Which of the following drug orders exertion and get adequate rest.
should the nurse question? B. The patient should resume a normal diet with
A. Nitroglycerin emphasis on nutritious, healthy foods.
B. Ampicillin C. The patient may discontinue the prescribed
C. Propranolol course of oral antibiotics once the symptoms
D. Verapamil have completely resolved.
48. Which of the following instructions D. The patient should continue use of the
should be included in the teaching for the incentive spirometer to keep airways open and
client with rheumatoid arthritis? free of secretions.
A. Avoid exercise because it fatigues the joints. 3. A nurse is caring for an elderly
B. Take prescribed anti-inflammatory Vietnamese patient in the terminal stages of
medications with meals. lung cancer. Many family members are in the
C. Alternate hot and cold packs to affected room around the clock performing unusual
joints. rituals and bringing ethnic foods. Which of
D. Avoid weight-bearing activity. the following actions should the nurse take?
49. A client with acute pancreatitis is A. Restrict visiting hours and ask the family to
experiencing severe abdominal pain. Which limit visitors to two at a time.
of the following orders should be questioned B. Notify visitors with a sign on the door that the
by the nurse? patient is limited to clear fluids only with no solid
A. Meperidine 100 mg IM q 4 hours PRN pain food allowed.
B. Mylanta 30 ccs q 4 hours via NG C. If possible, keep the other bed in the room
C. Cimetidine 300 mg PO q.i.d. unassigned to provide privacy and comfort to the
D. Morphine 8 mg IM q 4 hours PRN pain family.
50. The client is admitted to the chemical D. Contact the physician to report the unusual
dependence unit with an order for rituals and activities.
continuous observation. The nurse is aware 4. The charge nurse on the cardiac unit is
planning assignments for the day. Which of
the following is the most appropriate A. Tinnitus.
assignment for the float nurse that has been B. Diarrhea.
reassigned from labor and delivery? C. Hypertension.
A. A one-week postoperative coronary bypass D. Hepatic damage.
patient, who is being evaluated for placement of 9. A nurse is caring for a cancer patient
a pacemaker prior to discharge. receiving subcutaneous morphine sulfate for
B. A suspected myocardial infarction patient on pain. Which of the following nursing actions
telemetry, just admitted from the Emergency is most important in the care of this patient?
Department and scheduled for an angiogram. A. Monitor urine output.
C. A patient with unstable angina being closely B. Monitor respiratory rate.
monitored for pain and medication titration. C. Monitor heart rate.
D. A postoperative valve replacement patient D. Monitor temperature.
who was recently admitted to the unit because 10. A patient arrives at the emergency
all surgical beds were filled. department with severe lower leg pain after a
5. A newly diagnosed 8-year-old child with fall in a touch football game. Following
type I diabetes mellitus and his mother are routine triage, which of the following is the
receiving diabetes education prior to appropriate next step in assessment and
discharge. The physician has prescribed treatment?
Glucagon for emergency use. The mother A. Apply heat to the painful area.
asks the purpose of this medication. Which B. Apply an elastic bandage to the leg.
of the following statements by the nurse is C. X-ray the leg.
correct? D. Give pain medication.
A. Glucagon enhances the effect of insulin in 11. A nurse caring for several patients on the
case the blood sugar remains high one hour cardiac unit is told that one is scheduled for
after injection. implantation of an automatic internal
B. Glucagon treats hypoglycemia resulting from cardioverter-defibrillator. Which of the
insulin overdose. following patients is most likely to have this
C. Glucagon treats lipoatrophy from insulin procedure?
injections. A. A patient admitted for myocardial infarction
D. Glucagon prolongs the effect of insulin, without cardiac muscle damage.
allowing fewer injections. B. A post-operative coronary bypass patient,
6. An infant with congestive heart failure is recovering on schedule.
receiving diuretic therapy at home. Which of C. A patient with a history of ventricular
the following symptoms would indicate that tachycardia and syncopal episodes.
the dosage may need to be increased? D. A patient with a history of atrial tachycardia
A. Sudden weight gain. and fatigue.
B. Decreased blood pressure. 12. A patient is scheduled for a magnetic
C. Slow, shallow breathing. resonance imaging (MRI) scan for suspected
D. Bradycardia. lung cancer. Which of the following is a
7. A patient taking Dilantin (phenytoin) for a contraindication to the study for this patient?
seizure disorder is experiencing A. The patient is allergic to shellfish.
breakthrough seizures. A blood sample is B. The patient has a pacemaker.
taken to determine the serum drug level. C. The patient suffers from claustrophobia.
Which of the following would indicate a sub- D. The patient takes antipsychotic medication.
therapeutic level? 13. A nurse calls a physician with the
A. 15 mcg/mL. concern that a patient has developed a
B. 4 mcg/mL. pulmonary embolism. Which of the following
C. 10 mcg/dL. symptoms has the nurse most likely
D. 5 mcg/dL. observed?
8. A patient arrives at the emergency A. The patient is somnolent with decreased
department complaining of back pain. He response to the family.
reports taking at least 3 acetaminophen B. The patient suddenly complains of chest pain
tablets every three hours for the past week and shortness of breath.
without relief. Which of the following C. The patient has developed a wet cough and
symptoms suggests acetaminophen the nurse hears crackles on auscultation of the
toxicity? lungs.
D. The patient has a fever, chills, and loss of B. Night sweats and fatigue.
appetite. C. Nausea and vomiting.
14. A patient comes to the emergency D. Weight gain.
department with abdominal pain. Work-up 19. The Hodgkins disease patient described
reveals the presence of a rapidly enlarging in the question above undergoes a lymph
abdominal aortic aneurysm. Which of the node biopsy for definitive diagnosis. If the
following actions should the nurse expect? diagnosis of Hodgkins disease were correct,
A. The patient will be admitted to the medicine which of the following cells would the
unit for observation and medication. pathologist expect to find?
B. The patient will be admitted to the day A. Reed-Sternberg cells.
surgery unit for sclerotherapy. B. Lymphoblastic cells.
C. The patient will be admitted to the surgical C. Gauchers cells.
unit and resection will be scheduled. D. Rieders cells
D. The patient will be discharged home to follow- 20. A patient is about to undergo bone
up with his cardiologist in 24 hours. marrow aspiration and biopsy and expresses
15. A patient with leukemia is receiving fear and anxiety about the procedure. Which
chemotherapy that is known to depress bone of the following is the most effective nursing
marrow. A CBC (complete blood count) response?
reveals a platelet count of 25,000/microliter. A. Warn the patient to stay very still because the
Which of the following actions related smallest movement will increase her pain.
specifically to the platelet count should be B. Encourage the family to stay in the room for
included on the nursing care plan? the procedure.
A. Monitor for fever every 4 hours. C. Stay with the patient and focus on slow, deep
B. Require visitors to wear respiratory masks breathing for relaxation.
and protective clothing. D. Delay the procedure to allow the patient to
C. Consider transfusion of packed red blood deal with her feelings.
cells.
D. Check for signs of bleeding, including NCLEX Questions 11
examination of urine and stool for blood. 1. A mother complains to the clinic nurse
16. A patient is undergoing the induction that her 2 -year-old son is not yet toilet
stage of treatment for leukemia. The nurse trained. She is particularly concerned that,
teaches family members about infectious although he reliably uses the potty seat for
precautions. Which of the following bowel movements, he isnt able to hold his
statements by family members indicates that urine for long periods. Which of the following
the family needs more education? statements by the nurse is correct?
A. We will bring in books and magazines for A. The child should have been trained by age 2
entertainment. and may have a psychological problem that is
B. We will bring in personal care items for responsible for his accidents.
comfort. B. Bladder control is usually achieved before
C. We will bring in fresh flowers to brighten the bowel control, and the child should be required
room. to sit on the potty seat until he passes urine.
D. We will bring in family pictures and get well C. Bowel control is usually achieved before
cards. bladder control, and the average age for
17. A nurse is caring for a patient with acute completion of toilet training varies widely from 24
lymphoblastic leukemia (ALL). Which of the to 36 months.
following is the most likely age range of the D. The child should be told no each time he
patient? wets so that he learns the behavior is
A. 3-10 years. unacceptable.
B. 25-35 years. 2. The mother of a 14-month-old child reports
C. 45-55 years. to the nurse that her child will not fall asleep
D. over 60 years. at night without a bottle of milk in the crib
18. A patient is admitted to the oncology unit and often wakes during the night asking for
for diagnosis of suspected Hodgkins another. Which of the following instructions
disease. Which of the following symptoms is by the nurse is correct?
typical of Hodgkins disease? A. Allow the child to have the bottle at bedtime,
A. Painful cervical lymph nodes. but withhold the one later in the night.
B. Put juice in the bottle instead of milk. C. Headaches are a frequent side effect of
C. Give only a bottle of water at bedtime. nitroglycerine because it causes vasodilation.
D. Do not allow bottles in the crib. D. The headaches are unlikely to be related to
3. Which of the following actions is NOT the nitroglycerin, so you should see your doctor
appropriate in the care of a 2-month-old for further investigation.
infant? 8. A patient received surgery and
A. Place the infant on her back for naps and chemotherapy for colon cancer, completing
bedtime. therapy 3 months previously, and she is now
B. Allow the infant to cry for 5 minutes before in remission. At a follow-up appointment,
responding if she wakes during the night as she she complains of fatigue following activity
may fall back asleep. and difficulty with concentration at her
C. Talk to the infant frequently and make eye weekly bridge games. Which of the following
contact to encourage language development. explanations could account for her
D. Wait until at least 4 months to add infant symptoms?
cereals and strained fruits to the diet. A. The symptoms may be the result of anemia
4. An older patient asks a nurse to caused by chemotherapy.
recommend strategies to prevent B. The patient may be immunosuppressed.
constipation. Which of the following C. The patient may be depressed.
suggestions would be helpful? Note: More D. The patient may be dehydrated.
than one answer may be correct. 9. A clinic patient has a hemoglobin
A. Get moderate exercise for at least 30 minutes concentration of 10.8 g/dL and reports
each day. sticking to a strict vegetarian diet. Which of
B. Drink 6-8 glasses of water each day. the follow nutritional advice is appropriate?
C. Eat a diet high in fiber. A. The diet is providing adequate sources of iron
D. Take a mild laxative if you dont have a bowel and requires no changes.
movement every day. B. The patient should add meat to her diet; a
5. A child is admitted to the hospital with vegetarian diet is not advised.
suspected rheumatic fever. Which of the C. The patient should use iron cookware to
following observations is NOT confirming of prepare foods, such as dark green, leafy
the diagnosis? vegetables and legumes, which are high in iron.
A. A reddened rash visible over the trunk and D. A cup of coffee or tea should be added to
extremities. every meal.
B. A history of sore throat that was self-limited in 10. A hospitalized patient is receiving packed
the past month. red blood cells (PRBCs) for treatment of
C. A negative antistreptolysin O titer. severe anemia. Which of the following is the
D. An unexplained fever. most accurate statement?
6. A patient with a history of congestive heart A. Transfusion reaction is most likely
failure arrives at the clinic complaining of immediately after the infusion is completed.
dyspnea. Which of the following actions is B. PRBCs are best infused slowly through a
the first the nurse should perform? 20g. IV catheter.
A. Ask the patient to lie down on the exam table. C. PRBCs should be flushed with a 5% dextrose
B. Draw blood for chemistry panel and arterial solution.
blood gas (ABG). D. A nurse should remain in the room during the
C. Send the patient for a chest x-ray. first 15 minutes of infusion.
D. Check blood pressure. 11. Emergency department triage is an
7. A clinic patient has recently been important nursing function. A nurse working
prescribed nitroglycerin for treatment of the evening shift is presented with four
angina. He calls the nurse complaining of patients at the same time. Which of the
frequent headaches. Which of the following following patients should be assigned the
responses to the patient is correct? highest priority?
A. Stop taking the nitroglycerin and see if the A. A patient with low-grade fever, headache, and
headaches improve. myalgias for the past 72 hours.
B. Go to the emergency department to be B. A patient who is unable to bear weight on the
checked because nitroglycerin can cause left foot, with swelling and bruising following a
bleeding in the brain. running accident.
C. A patient with abdominal and chest pain C. Nonsteroidal anti-inflammatory drugs are the
following a large, spicy meal. first choice in treatment.
D. A child with a one-inch bleeding laceration on D. Physical activity should be minimized.
the chin but otherwise well after falling while 17. A child is admitted to the hospital several
jumping on his bed. days after stepping on a sharp object that
12. A patient is admitted to the hospital with punctured her athletic shoe and entered the
a calcium level of 6.0 mg/dL. Which of the flesh of her foot. The physician is concerned
following symptoms would you NOT expect about osteomyelitis and has ordered
to see in this patient? parenteral antibiotics. Which of the following
A. Numbness in hands and feet. actions is done immediately before the
B. Muscle cramping. antibiotic is started?
C. Hypoactive bowel sounds. A. The admission orders are written.
D. Positive Chvosteks sign. B. A blood culture is drawn.
13. A nurse cares for a patient who has a C. A complete blood count with differential is
nasogastric tube attached to low suction drawn.
because of a suspected bowel obstruction. D. The parents arrive.
Which of the following arterial blood gas 18. A two-year-old child has sustained an
results might be expected in this patient? injury to the leg and refuses to walk. The
A. pH 7.52, PCO2 54 mmHg. nurse in the emergency department
B. pH 7.42, PCO2 40 mmHg. documents swelling of the lower affected leg.
C. pH 7.25, PCO2 25 mmHg. Which of the following does the nurse
D. pH 7.38, PCO2 36 mmHg. suspect is the cause of the childs
14. A patient is admitted to the hospital for symptoms?
routine elective surgery. Included in the list A. Possible fracture of the tibia.
of current medications is Coumadin B. Bruising of the gastrocnemius muscle.
(warfarin) at a high dose. Concerned about C. Possible fracture of the radius.
the possible effects of the drug, particularly D. No anatomic injury, the child wants his
in a patient scheduled for surgery, the nurse mother to carry him.
anticipates which of the following actions? 19. A toddler has recently been diagnosed
A. Draw a blood sample for prothrombin (PT) with cerebral palsy. Which of the following
and international normalized ratio (INR) level. information should the nurse provide to the
B. Administer vitamin K. parents? Note: More than one answer may
C. Draw a blood sample for type and be correct.
crossmatch and request blood from the blood A. Regular developmental screening is important
bank. to avoid secondary developmental delays.
D. Cancel the surgery after the patient reports B. Cerebral palsy is caused by injury to the
stopping the Coumadin one week previously. upper motor neurons and results in motor
15. The follow lab results are received for a dysfunction, as well as possible ocular and
patient. Which of the following results are speech difficulties.
abnormal? Note: More than one answer may C. Developmental milestones may be slightly
be correct. delayed but usually will require no additional
A. Hemoglobin 10.4 g/dL. intervention.
B. Total cholesterol 340 mg/dL. D. Parent support groups are helpful for sharing
C. Total serum protein 7.0 g/dL. strategies and managing health care issues.
D. Glycosylated hemoglobin A1C 5.4%. 20. A child has recently been diagnosed with
16. A nurse is assigned to the pediatric Duchennes muscular dystrophy. The
rheumatology clinic and is assessing a child parents are receiving genetic counseling
who has just been diagnosed with juvenile prior to planning another pregnancy. Which
idiopathic arthritis. Which of the following of the following statements includes the
statements about the disease is most most accurate information?
accurate? A. Duchennes is an X-linked recessive disorder,
A. The child has a poor chance of recovery so daughters have a 50% chance of being
without joint deformity. carriers and sons a 50% chance of developing
B. Most children progress to adult rheumatoid the disease.
arthritis.
B. Duchennes is an X-linked recessive disorder, D. Hemoglobin
so both daughters and sons have a 50% chance E. Complete Blood Count
of developing the disease. F. White Blood Cell Count
C. Each child has a 1 in 4 (25%) chance of 6. A patient on the cardiac telemetry unit
developing the disorder. unexpectedly goes into ventricular
D. Sons only have a 1 in 4 (25%) chance of fibrillation. The advanced cardiac life support
developing the disorder. team prepares to defibrillate. Which of the
following choices indicates the correct
NCLEX Questions 12 placement of the conductive gel pads?
1. A patient is admitted to the hospital with a A. The left clavicle and right lower sternum.
diagnosis of primary hyperparathyroidism. A B. Right of midline below the bottom rib and the
nurse checking the patients lab results left shoulder.
would expect which of the following changes C. The upper and lower halves of the sternum.
in laboratory findings? D. The right side of the sternum just below the
A. Elevated serum calcium. clavicle and left of the precordium.
B. Low serum parathyroid hormone (PTH). 7. The nurse performs an initial abdominal
C. Elevated serum vitamin D. assessment on a patient newly admitted for
D. Low urine calcium. abdominal pain. The nurse hears what she
2. A patient with Addisons disease asks a describes as clicks and gurgles in all four
nurse for nutrition and diet advice. Which of quadrants as well as swishing or buzzing
the following diet modifications is NOT sound heard in one or two quadrants.
recommended? Which of the following statements is
A. A diet high in grains. correct?
B. A diet with adequate caloric intake. A. The frequency and intensity of bowel sounds
C. A high protein diet. varies depending on the phase of digestion.
D. A restricted sodium diet. B. In the presence of intestinal obstruction,
3. A patient with a history of diabetes bowel sounds will be louder and higher pitched.
mellitus is in the second post-operative day C. A swishing or buzzing sound may represent
following cholecystectomy. She has the turbulent blood flow of a bruit and is not
complained of nausea and isnt able to eat normal.
solid foods. The nurse enters the room to D. All of the above.
find the patient confused and shaky. Which 8. A patient arrives in the emergency
of the following is the most likely explanation department and reports splashing
for the patients symptoms? concentrated household cleaner in his eye.
A. Anesthesia reaction. Which of the following nursing actions is a
B. Hyperglycemia. priority?
C. Hypoglycemia. A. Irrigate the eye repeatedly with normal saline
D. Diabetic ketoacidosis. solution.
4. A nurse assigned to the emergency B. Place fluorescein drops in the eye.
department evaluates a patient who C. Patch the eye.
underwent fiberoptic colonoscopy 18 hours D. Test visual acuity.
previously. The patient reports increasing 9. A nurse is caring for a patient who has had
abdominal pain, fever, and chills. Which of hip replacement. The nurse should be most
the following conditions poses the most concerned about which of the following
immediate concern? findings?
A. Bowel perforation. A. Complaints of pain during repositioning.
B. Viral gastroenteritis. B. Scant bloody discharge on the surgical
C. Colon cancer. dressing.
D. Diverticulitis. C. Complaints of pain following physical therapy.
5. A patient is admitted to the same day D. Temperature of 101.8 F (38.7 C).
surgery unit for liver biopsy. Which of the 10. A child is admitted to the hospital with an
following laboratory tests assesses uncontrolled seizure disorder. The admitting
coagulation? Select all that apply. physician writes orders for actions to be
A. Partial thromboplastin time. taken in the event of a seizure. Which of the
B. Prothrombin time. following actions would NOT be included?
C. Platelet count. A. Notify the physician.
B. Restrain the patients limbs. B. Immunizations provide natural immunity from
C. Position the patient on his/her side with the disease.
head flexed forward. C. Immunizations are risk-free and should be
D. Administer rectal diazepam. universally administered.
11. A patient who has received D. Immunization provides acquired immunity
chemotherapy for cancer treatment is given from some specific diseases.
an injection of Epoetin. Which of the 17. A patient is brought to the emergency
following should reflect the findings in a department after a bee sting. The family
complete blood count (CBC) drawn several reports a history of severe allergic reaction,
days later? and the patient appears to have some oral
A. An increase in neutrophil count. swelling. Which of the following is the most
B. An increase in hematocrit. urgent nursing action?
C. An increase in platelet count. A. Consult a physician.
D. An increase in serum iron. B. Maintain a patent airway.
12. A patient is admitted to the hospital with C. Administer epinephrine subcutaneously.
suspected polycythemia vera. Which of the D. Administer diphenhydramine (Benadryl)
following symptoms is consistent with the orally.
diagnosis? Select all that apply. 18. A mother calls the clinic to report that her
A. Weight loss. son has recently started medication to treat
B. Increased clotting time. attention deficit/hyperactivity disorder
C. Hypertension. (ADHD). The mother fears her son is
D. Headaches. experiencing side effects of the medicine.
13. A nurse is caring for a patient with a Which of the following side effects are
platelet count of 20,000/microliter. Which of typically related to medications used for
the following is an important intervention? ADHD? Note: More than one answer may be
A. Observe for evidence of spontaneous correct:
bleeding. A. Poor appetite.
B. Limit visitors to family only. B. Insomnia.
C. Give aspirin in case of headaches. C. Sleepiness.
D. Impose immune precautions. D. Agitation.
14. A nurse in the emergency department 19. A patient at a mental health clinic is
assesses a patient who has been taking taking Haldol (haloperidol) for treatment of
long-term corticosteroids to treat renal schizophrenia. She calls the clinic to report
disease. Which of the following is a typical abnormal movements of her face and
side effect of corticosteroid treatment? Note: tongue. The nurse concludes that the patient
More than one answer may be correct. is experiencing which of the following
A. Hypertension. symptoms:
B. Cushingoid features. A. Co-morbid depression.
C. Hyponatremia. B. Psychotic hallucinations.
D. Low serum albumin. C. Negative symptoms of schizophrenia.
15. A nurse is caring for patients in the D. Tardive dyskinesia.
oncology unit. Which of the following is the 20. A patient with newly diagnosed diabetes
most important nursing action when caring mellitus is learning to recognize the
for a neutropenic patient? symptoms of hypoglycemia. Which of the
A. Change the disposable mask immediately following symptoms is indicative of
after use. hypoglycemia?
B. Change gloves immediately after use. A. Polydipsia.
C. Minimize patient contact. B. Confusion.
D. Minimize conversation with the patient. C. Blurred vision.
16. A nurse is counseling patients at a health D. Polyphagia.
clinic on the importance of immunizations.
Which of the following information is the NCLEX Questions 13
most accurate regarding immunizations? 1. A child is admitted to the hospital with a
A. All infectious diseases can be prevented with diagnosis of Wilms tumor, stage II. Which of
proper immunization. the following statements most accurately
describes this stage?
A. The tumor is less than 3 cm. in size and following is the most likely route of
requires no chemotherapy. transmission?
B. The tumor did not extend beyond the kidney A. Sexual contact with an infected partner.
and was completely resected. B. Contaminated food.
C. The tumor extended beyond the kidney but C. Blood transfusion.
was completely resected. D. Illegal drug use.
D. The tumor has spread into the abdominal 7. A leukemia patient has a relative who
cavity and cannot be resected. wants to donate blood for transfusion. Which
2. A teen patient is admitted to the hospital of the following donor medical conditions
by his physician who suspects a diagnosis would prevent this?
of acute glomerulonephritis. Which of the A. A history of hepatitis C five years previously.
following findings is consistent with this B. Cholecystitis requiring cholecystectomy one
diagnosis? Note: More than one answer may year previously.
be correct. C. Asymptomatic diverticulosis.
A. Urine specific gravity of 1.040. D. Crohns disease in remission.
B. Urine output of 350 ml in 24 hours. 8. A physician has diagnosed acute gastritis
C. Brown (tea-colored) urine. in a clinic patient. Which of the following
D. Generalized edema. medications would be contraindicated for
3. Which of the following conditions most this patient?
commonly causes acute glomerulonephritis? A. Naproxen sodium (Naprosyn).
A. A congenital condition leading to renal B. Calcium carbonate.
dysfunction. C. Clarithromycin (Biaxin).
B. Prior infection with group A Streptococcus D. Furosemide (Lasix).
within the past 10-14 days. 9. The nurse is conducting nutrition
C. Viral infection of the glomeruli. counseling for a patient with cholecystitis.
D. Nephrotic syndrome. Which of the following information is
4. An infant with hydrocele is seen in the important to communicate?
clinic for a follow-up visit at 1 month of age. A. The patient must maintain a low calorie diet.
The scrotum is smaller than it was at birth, B. The patient must maintain a high protein/low
but fluid is still visible on illumination. Which carbohydrate diet.
of the following actions is the physician C. The patient should limit sweets and sugary
likely to recommend? drinks.
A. Massaging the groin area twice a day until the D. The patient should limit fatty foods.
fluid is gone. 10. A patient admitted to the hospital with
B. Referral to a surgeon for repair. myocardial infarction develops severe
C. No treatment is necessary; the fluid is pulmonary edema. Which of the following
reabsorbing normally. symptoms should the nurse expect the
D. Keeping the infant in a flat, supine position patient to exhibit?
until the fluid is gone. A. Slow, deep respirations.
5. A nurse is caring for a patient with B. Stridor.
peripheral vascular disease (PVD). The C. Bradycardia.
patient complains of burning and tingling of D. Air hunger.
the hands and feet and cannot tolerate touch 11. A nurse is evaluating
of any kind. Which of the following is the a postoperative patient and notes a moderate
most likely explanation for these symptoms? amount of serous drainage on the dressing
A. Inadequate tissue perfusion leading to nerve 24 hours after surgery. Which of the
damage. following is the appropriate nursing action?
B. Fluid overload leading to compression of A. Notify the surgeon about evidence of infection
nerve tissue. immediately.
C. Sensation distortion due to psychiatric B. Leave the dressing intact to avoid disturbing
disturbance. the wound site.
D. Inflammation of the skin on the hands and C. Remove the dressing and leave the wound
feet. site open to air.
6. A nurse is assessing a clinic patient with a D. Change the dressing and document the clean
diagnosis of hepatitis A. Which of the appearance of the wound site.
12. A patient returns to the emergency the patient sitting up in bed, dyspneic and
department less than 24 hours after having a uncomfortable. On assessment, crackles are
fiberglass cast applied for a fractured right heard in the bases of both lungs, probably
radius. Which of the following patient indicating that the patient is experiencing a
complaints would cause the nurse to be complication of transfusion. Which of the
concerned about impaired perfusion to the following complications is most likely the
limb? cause of the patients symptoms?
A. Severe itching under the cast. A. Febrile non-hemolytic reaction.
B. Severe pain in the right shoulder. B. Allergic transfusion reaction.
C. Severe pain in the right lower arm. C. Acute hemolytic reaction.
D. Increased warmth in the fingers. D. Fluid overload.
18. A patient in labor and delivery has just
13. An older patient with osteoarthritis is received an amniotomy. Which of the
preparing for discharge. Which of the following is correct? Note: More than one
following information is correct? answer may be correct.
A. Increased physical activity and daily exercise A. Frequent checks for cervical dilation will be
will help decrease discomfort associated with needed after the procedure.
the condition. B. Contractions may rapidly become stronger
B. Joint pain will diminish after a full night of rest. and closer together after the procedure.
C. Nonsteroidal anti-inflammatory medications C. The FHR (fetal heart rate) will be followed
should be taken on an empty stomach. closely after the procedure due to the possibility
D. Acetaminophen (Tylenol) is a more effective of cord compression.
anti-inflammatory than ibuprofen (Motrin). D. The procedure is usually painless and is
14. Which patient should NOT be prescribed followed by a gush of amniotic fluid.
alendronate (Fosamax) for osteoporosis? 19. A nurse is counseling the mother of a
A. A female patient being treated for high blood newborn infant with hyperbilirubinemia.
pressure with an ACE inhibitor. Which of the following instructions by the
B. A patient who is allergic to iodine/shellfish. nurse is NOT correct?
C. A patient on a calorie restricted diet. A. Continue to breastfeed frequently, at least
D. A patient on bed rest who must maintain a every 2-4 hours.
supine position. B. Follow up with the infants physician within 72
15. Which of the following strategies is NOT hours of discharge for a recheck of the serum
effective for prevention of Lyme disease? bilirubin and exam.
A. Insect repellant on the skin and clothes when C. Watch for signs of dehydration, including
in a Lyme endemic area. decreased urinary output and changes in skin
B. Long sleeved shirts and long pants. turgor.
C. Prophylactic antibiotic therapy prior to D. Keep the baby quiet and swaddled, and place
anticipated exposure to ticks. the bassinet in a dimly lit area.
D. Careful examination of skin and hair for ticks 20. A nurse is giving discharge instructions
following anticipated exposure. to the parents of a healthy newborn. Which
16. A nurse is performing routine of the following instructions should the
assessment of an IV site in a patient nurse provide regarding car safety and the
receiving both IV fluids and medications trip home from the hospital?
through the line. Which of the following A. The infant should be restrained in an infant
would indicate the need for discontinuation car seat, properly secured in the back seat in a
of the IV line as the next nursing action? rear-facing position.
A. The patient complains of pain on movement. B. The infant should be restrained in an infant
B. The area proximal to the insertion site is car seat, properly secured in the front passenger
reddened, warm, and painful. seat.
C. The IV solution is infusing too slowly, C. The infant should be restrained in an infant
particularly when the limb is elevated. car seat facing forward or rearward in the back
D. A hematoma is visible in the area of the IV seat.
insertion site. D. For the trip home from the hospital, the
17. A hospitalized patient has received parent may sit in the back seat and hold the
transfusions of 2 units of blood over the past newborn.
few hours. A nurse enters the room to find
NCLEX Questions 14 C. Smoking.
1. A nurse is administering IV furosemide to D. Age.
a patient admitted with congestive heart 7. Claudication is a well-known effect of
failure. After the infusion, which of the peripheral vascular disease. Which of the
following symptoms is NOT expected? following facts about claudication is correct?
A. Increased urinary output. More than one answer may be correct.
B. Decreased edema. A. It results when oxygen demand is greater
C. Decreased pain. than oxygen supply.
D. Decreased blood pressure. B. It is characterized by pain that often occurs
2. There are a number of risk factors during rest.
associated with coronary artery disease. C. It is a result of tissue hypoxia.
Which of the following is a modifiable risk D. It is characterized by cramping and
factor? weakness.
A. Obesity. 8. A nurse is providing discharge information
B. Heredity. to a patient with peripheral vascular disease.
C. Gender. Which of the following information should be
D. Age. included in instructions?
3. Tissue plasminogen activator (t-PA) is A. Walk barefoot whenever possible.
considered for treatment of a patient who B. Use a heating pad to keep feet warm.
arrives in the emergency department C. Avoid crossing the legs.
following onset of symptoms of myocardial D. Use antibacterial ointment to treat skin
infarction. Which of the following is a lesions at risk of infection.
contraindication for treatment with t-PA? 9. A patient who has been diagnosed with
A. Worsening chest pain that began earlier in vasospastic disorder (Raynauds disease)
the evening. complains of cold and stiffness in the
B. History of cerebral hemorrhage. fingers. Which of the following descriptions
C. History of prior myocardial infarction. is most likely to fit the patient?
D. Hypertension. A. An adolescent male.
4. Following myocardial infarction, a B. An elderly woman.
hospitalized patient is encouraged to C. A young woman.
practice frequent leg exercises and ambulate D. An elderly man.
in the hallway as directed by his physician. 10. A 23 year old patient in the 27th week of
Which of the following choices reflects the pregnancy has been hospitalized on
purpose of exercise for this patient? complete bed rest for 6 days. She
A. Increases fitness and prevents future heart experiences sudden shortness of breath,
attacks. accompanied by chest pain. Which of the
B. Prevents bedsores. following conditions is the most likely cause
C. Prevents DVT (deep vein thrombosis). of her symptoms?
D. Prevent constipations. A. Myocardial infarction due to a history of
5. A patient arrives in the emergency atherosclerosis.
department with symptoms of myocardial B. Pulmonary embolism due to deep vein
infarction, progressing to cardiogenic shock. thrombosis (DVT).
Which of the following symptoms should the C. Anxiety attack due to worries about her
nurse expect the patient to exhibit with babys health.
cardiogenic shock? D. Congestive heart failure due to fluid overload.
A. Hypertension. 11. Thrombolytic therapy is frequently used
B. Bradycardia. in the treatment of suspected stroke. Which
C. Bounding pulse. of the following is a significant complication
D. Confusion. associated with thrombolytic therapy?
6. A patient in the cardiac unit is concerned A. Air embolus.
about the risk factors associated with B. Cerebral hemorrhage.
atherosclerosis. Which of the following are C. Expansion of the clot.
hereditary risk factors for developing D. Resolution of the clot.
atherosclerosis? 12. An infant is brought to the clinic by his
A. Family history of heart disease. mother, who has noticed that he holds his
B. Overweight. head in an unusual position and always
faces to one side. Which of the following is B. The rash begins on the trunk and spreads
the most likely explanation? outward.
A. Torticollis, with shortening of the C. There is low-grade fever.
sternocleidomastoid muscle. D. The lesions have a teardrop on a rose petal
B. Craniosynostosis, with premature closure of appearance.
the cranial sutures. 18. A child is seen in the emergency
C. Plagiocephaly, with flattening of one side of department for scarlet fever. Which of the
the head. following descriptions of scarlet fever is NOT
D. Hydrocephalus, with increased head size. correct?
13. An adolescent brings a physicians note A. Scarlet fever is caused by infection with group
to school stating that he is not to participate A Streptococcus bacteria.
in sports due to a diagnosis of Osgood- B. Strawberry tongue is a characteristic sign.
Schlatter disease. Which of the following C. Petechiae occur on the soft palate.
statements about the disease is correct? D. The pharynx is red and swollen.
A. The condition was caused by the students 19. A child weighing 30 kg arrives at the
competitive swimming schedule. clinic with diffuse itching as the result of an
B. The student will most likely require surgical allergic reaction to an insect bite.
intervention. Diphenhydramine (Benadryl) 25 mg 3 times a
C. The student experiences pain in the inferior day is prescribed. The correct pediatric dose
aspect of the knee. is 5 mg/kg/day. Which of the following best
D. The student is trying to avoid participation in describes the prescribed drug dose?
physical education. A. It is the correct dose.
14. The clinic nurse asks a 13-year-old B. The dose is too low.
female to bend forward at the waist with C. The dose is too high.
arms hanging freely. Which of the following D. The dose should be increased or decreased,
assessments is the nurse most likely depending on the symptoms.
conducting? 20. The mother of a 2-month-old infant brings
A. Spinal flexibility. the child to the clinic for a well baby check.
B. Leg length disparity. She is concerned because she feels only one
C. Hypostatic blood pressure. testis in the scrotal sac. Which of the
D. Scoliosis. following statements about the undescended
15. A clinic nurse interviews a parent who is testis is the most accurate?
suspected of abusing her child. Which of the A. Normally, the testes are descended by birth.
following characteristics is the nurse LEAST B. The infant will likely require surgical
likely to find in an abusing parent? intervention.
A. Low self-esteem. C. The infant probably has with only one testis.
B. Unemployment. D. Normally, the testes descend by one year of
C. Self-blame for the injury to the child. age.
D. Single status.
16. A nurse in the emergency department is NURSING RESEARCH 1
observing a 4-year-old child for signs of 1. What should be included in scholarly
increased intracranial pressure after a fall literature? Select all that apply.
from a bicycle, resulting in head trauma. A. Research reports reported in primary sources
Which of the following signs or symptoms only
would be cause for concern? B. Conceptual and theoretical literature from
A. Bulging anterior fontanel. primary sources only
B. Repeated vomiting. C. Published and unpublished reports of
C. Signs of sleepiness at 10 PM. research
D. Inability to read short words from a distance D. Primary and secondary sources
of 18 inches. 2. Reviews of the literature are conducted for
17. A nonimmunized child appears at the PURPOSES OF RESEARCH as well as for the
clinic with a visible rash. Which of the CONSUMER OF RESEARCH. How are these
following observations indicates the child reviews similar? Select all that apply.
may have rubeola (measles)? A. Amount of literature required to be reviewed
A. Small blue-white spots are visible on the oral B. Degree of critical reading required
mucosa. C. Importance of conceptual literature
D. Purpose of the review B. Research question study purpose
3. What are characteristics of the literature C. Literature review
review required for a quantitative research D. Design
study? Select all that apply. E. Sample
A. The review is exhaustive and must include all F. Legal-ethical issues
studies conducted in the area G. Data-collection procedure
B. Doctoral dissertations and masters theses 10. What does a level-of-evidence model use
are excellent sources of information to evaluate the strength of a research study
C. Computer-accessed materials are acceptable and its findings? Select all that apply.
D. Primary sources are not as important as A. Creativity
secondary sources B. Quality
4. Which of the following is an example of a C. Quantity
primary source in a research study? D. Consistency
A. A published commentary on the findings of E. Efficiency
another study 11. What are the critiquing criteria used to
B. A doctoral dissertation that critiques all judge the worth of a research study? Select
research in the area of attention deficit disorder all that apply.
C. A textbook of medical-surgical nursing A. Measures
D. A journal article about a study that used B. Objectives
large, previously unpublished databases C. Standards
generated by the United States census D. Effectiveness
5. What is the best source to use when E. Evaluation guides
conducting a level I systematic meta- F. Questions
analysis of the literature? 12. Which statement best describes
A. An electronic database qualitative research? Select all that apply.
B. Doctoral dissertations A. Studies are conducted in natural settings.
C. The Cochrane Statistical Methods B. Data are collected from a large number of
D. An electronic database and Doctoral subjects.
dissertations C. Data collected tend to be numeric.
6. What is a characteristic of an audio D. The research design is systematic and
recording of an unpublished research study subjective.
reported at a professional conference? 13. What does a critique of a research study
A. Databased literature always include? Select all that apply.
B. Secondary Sources A. Determining its strengths and weaknesses
C. Are more difficult to analyze than written B. Researching similar studies
reports. C. Using critical reading skills
D. Are not useful because they are not D. Explaining your own personal opinions
published 14. For which of the following research
7. What is the first step in the qualitative questions would qualitative methods be
research process? most appropriate?
A. Data analysis A. Which pain medications decrease the need
B. Sample for sleep medication in elderly patients?
C. Review of literature B. What is the meaning of health for migrant
D. Study design farm-worker women?
8. Which mode of clinical application for C. Under what conditions does a decubitus ulcer
qualitative research is considered to be the heal most quickly?
sharing of qualitative findings with the D. How does frequency of medication
patient? administration impact the degree of pain
A. Insight or empathy experienced following knee replacement
B. Anticipatory guidance surgery?
C. Assessment of status or progress 15. Which of the following phrases would be
D. Coaching found in a report of a qualitative study?
9. Which research process steps may be A. The hypothesis of this study is?
noted in an articles abstract? Select all that B. Perceived pain was measured using the
apply. Abbott pain scale?
A. Identifying the phenomenon C. The control group received no instruction?
D. Subjects were asked to relate their B. Hypotheses help frame a test of the validity of
perceptions of pain? a theory.
16. Which of the following phrases would be C. Hypotheses provide the means to test
found in a report of a quantitative study? nursing theory.
A. A convenience sample was chosen? D. A hypothesis can also be called a problem
B. The phenomenon studied was? statement.
C. Data were analyzed and interpreted? 2. A nurse wants to study the effectiveness
D. Researchers sought to explore the meaning of meditation on people with anxiety
of the hospital experience? disorder. Which variable would be most
17. Which of the following hypotheses are relevant to explore in the literature on this
indicative of an experimental research topic?
design? Select all that apply. A. Use of meditation during childbirth
A. Frequent irrigation of Foley catheters will be B. Meditation techniques found to be effective
positively related to urinary tract infections. C. Pain management for people with anxiety
B. The incidence of urinary tract infections will disorders
be greater in patients whose Foley catheters are D. Outcomes of meditation when used by elderly
irrigated frequently than in those whose Foley populations
catheters are irrigated less frequently. 3. What is a characteristic of an independent
C. Frequent irrigation of Foley catheters is variable?
associated with urinary tract infections. A. It is the variable that is predicted to change.
D. The incidence of urinary tract infections will B. It varies with a change in the dependent
not differ between patients with or without Foley variable.
catheters. C. It is manipulated by the researcher.
18. Which statements are part of the criteria D. It can be identified only by changes in the
used to judge the soundness of a stated dependent variable.
research question? Select all that apply. 4. Which statement is most accurate
A. A relationship between two or more variables regarding hypotheses?
B. An operational definition of each variable A. Hypotheses operationally define the
C. The nature of the population being tested dependent variables.
D. The possibility of empirical testing B. Hypotheses are statements about the
19. Which criteria are used to determine relationships among variables.
testability of a hypothesis? Select all that C. Hypotheses describe the effect of the
apply. dependent variable on the independent variable.
A. Use of quantifiable words such as greater D. Hypotheses must include a definition of the
than or less than treatment or intervention used.
B. A hypothesis stated in such a way that it can 5. What is a characteristic of a hypothesis?
be clearly supported or not supported A. It flows from interpretation of the data
C. The use of value-laden words in a hypothesis collected.
D. Data-collection efforts that prove the validity B. It operationally defines the variable to be
of the hypothesis studied.
20. What are the advantages to using C. It eliminates the need to designate a
directional hypotheses? Select all that apply. dependent variable.
A. The indication of the use of a theory base to D. It implies a causative or associative
derive the hypothesis relationship.
B. The provision of a specific theoretical frame 6. When should a hypothesis be developed
of reference by the researcher during the research
C. Ensurance that findings will be generalizable process?
D. The indication of a nonbiased selection of A. Before development of the research question
subjects B. After development of the research question
C. After a research design is determined
NURSING RESEARCH 2 D. Before any statistical analysis
1. Which one of the following statements 7. Which research hypothesis is most
about hypotheses is most accurate? testable?
A. Hypotheses represent the main idea to be A. There is a relationship between meditation
studied and are the foundations of research and anxiety disorders.
studies.
B. Patients with anxiety disorders who learn aggressive treatment for breast cancer than
meditation techniques have less anxiety than do younger women. Which variable would be
those who do not. considered to be the independent variable?
C. Teaching one meditation technique to A. Degree of treatment received
patients with anxiety disorders will be better than B. Age of the patient
teaching multiple techniques. C. Type of cancer being treated
D. The ability to meditate causes lower anxiety D. Use of inpatient treatment
in patients with anxiety disorder than those who 14. The following are considered steps in the
do not meditate. qualitative research process, except?
8. What is a characteristic of a statistical A. Literature review
hypothesis? B. Hypothesis
A. It is a null hypothesis. C. Sample
B. It predicts a positive relationship among D. Data collection
variables. 15. Which of the following could be
C. It is a complex hypothesis. considered the context of a study? Select
D. It describes data-analysis methods. all that apply.
9. When will a null hypothesis be rejected? A. Cultural understandings and beliefs of study
A. There is no association among variables. participants
B. There is evidence of significance. B. The physical setting of the study
C. The independent and dependent variables C. The sample selected for the study
are related. D. The number of subjects in the study
D. The research hypothesis is rejected. 16. Which beliefs guide the constructivist
10. Which level is characteristic of the paradigm? Select all that apply.
strength of the evidence provided by the A. There are multiple realities.
results of a quasi-experimental study? B. The truth is objective.
A. Level I C. Context does not matter as much as truth.
B. Level II D. The participant (subject) is an active part of
C. Level III the study
D. Level IV 17. Which of the following are consistent
11. A researcher wants to discover why with the constructivist paradigm? Select all
patients of certain ethnic backgrounds are that apply.
reluctant to ask for pain medication. Because A. Subjectivism is valued.
there are little data in the literature on this B. Natural laws exist.
topic, the researcher designs a study to C. Time and place are important.
explore the relationships between cultural D. Generalizability is valued.
belief systems, the experience of pain, and 18. Which paradigm provides the basis for
the effective use of medication to relieve qualitative research?
pain. The researcher plans to use the A. Empirical analytical research
findings of this study to formulate B. Constructivism
hypotheses for a future study. What is a C. Postpositivism
characteristic of this study? D. Naturalistic research
A. It is a quasi-experimental study. 19. Which type of research allows
B. It will lead to level II data. researchers to be neutral observers?
C. It has a directional hypothesis. A. Qualitative research
D. It is a hypothesis-generating study. B. Ethnographic research
12. The nurse develops the following C. Quantitative research
hypothesis: Elderly women receive less D. Case studies
aggressive treatment for breast cancer than 20. Which type of research study can be
do younger women. Which variable would be affected by detracting values of the
considered to be the dependent variable? researcher?
A. Degree of treatment received A. Qualitative
B. Age of the patient B. Naturalistic
C. Type of cancer being treated C. Ethnographic
D. Use of inpatient treatment D. Quantitative
13. The nurse develops the following
hypothesis: Elderly women receive less NURSING RESEARCH 3
1. What is the purpose of grounded theory? 7. Which question will critique the credibility
A. To support theoretical frameworks of a research project?
B. To generate theory from data A. Is the strategy used for analysis compatible
C. To develop explanatory models with the purpose of the study?
D. To find significant differences among groups B. Does the researcher document the research
of people process?
2. Why is it important to understand the C. Are the researchers conceptualizations true
philosophy underlying each type of to the data?
research? D. Has adequate time been allowed to
A. Conclusions reached should be congruent understand fully the phenomenon?
with the research question. 8. Which question will critique the
B. The research method that best meets auditability of a research project?
intended purpose of the study should be used. A. Is the strategy used for analysis compatible
C. The paradigm of the method should be the with the purpose of the study?
same as that of the researcher. B. Does the researcher document the research
D. The reader should understand the level of process?
abstraction of the study. C. Are the researchers conceptualizations true
3. Which conceptual analysis point of the to the data?
framework for rigor used for interpretive D. Has adequate time been allowed to fully
phenomenology refers to how the study understand the phenomenon?
findings will continue to have meaning for 9. Which question will critique the fittingness
the reader? of a research project?
A. Resonance A. Is the strategy used for analysis compatible
B. Concreteness with the purpose of the study?
C. Actualization B. Does the researcher document the research
D. Openness process?
4. Which question will critique the sampling C. Are the researchers conceptualizations true
of a research project? to the data?
A. Is the strategy used for analysis compatible D. Has adequate time been allowed to fully
with the purpose of the study? understand the phenomenon?
B. What is the projected significance of the work 10. What are the uses of qualitative research
to nursing? methods? Select all that apply.
C. Are the informants who were chosen A. Guiding nursing practice
appropriate to inform the research? B. Studying the effects of nursing care on an
D. What are the philosophic underpinnings of outcome variable
the research method? C. Developing survey instruments
5. Which question will critique the method of D. Developing nursing theory
a research project? 11. What are scientific criteria appropriate for
A. Is the strategy used for analysis compatible qualitative research? Select all that apply.
with the purpose of the study? A. Auditability
B. What is the projected significance of the work B. Credibility
to nursing? C. Fittingness
C. Are the informants who were chosen D. Reliability
appropriate to inform the research? 12. What are ethical concerns for qualitative
D. What are the philosophic underpinnings of researchers? Select all that apply.
the research method? A. Because the study emerges over time, the
6. Which question will critique the purpose of researcher may not anticipate and inform the
a research project? participants of a potential threat.
A. Is the strategy used for analysis compatible B. To maintain a naturalistic environment for
with the purpose of the study? interviews, formal documents such as consent
B. What is the projected significance of the work forms are not used.
to nursing? C. Because there are so few participants in a
C. Are the informants who were chosen qualitative study, no participant can opt out of
appropriate to inform the research? the study.
D. What are the philosophic underpinnings of D. Because the researcher and participant
the research method? interact over a period of time, relationships
developed between them may change the focus B. Can the reader follow the researchers
of the interaction thinking?
13. Which of the following is most accurate C. Are the results meaningful to individuals not
regarding the grounded-theory method? involved in the research?
A. Data are collected using an etic perspective. D. Does the researcher document the research
B. It is a process of constructing human process?
experience. 20. Which question is helpful in determining
C. Secondary sources are sometimes used. the studys credibility?
D. It is an inductive approach. A. Do the participants recognize the experience
14. What is the term used for the coding and as their own?
clustering of data to form categories in the B. What strategies were used to analyze the
grounded-theory method? data?
A. Theoretical sampling C. How were human subjects protected?
B. Constant-comparative method D. Are the findings applicable outside the study
C. Emic method situation?
D. Metasynthesis
15. What is a characteristic of an intrinsic Prioritization, Delegation and Assignment 1
case study? 1. A 16-year old patient with cystic fibrosis is
A. It yields a better understanding of each case. admitted with increased shortness of breath
B. It provides a foundation to challenge a and possible pneumonia. Which nursing
generalization. activity is most important to include in the
C. It does not include quantitative data. patients care?
D. It can scrutinize only uncomplicated A. Perform postural drainage and chest
phenomena. physiotherapy every 4 hours
16. What is a characteristic of B. Allow the patient to decide whether she
metasynthesis? needs aerosolized medications
A. It is useful for triangulating research. C. Place the patient in a private room to
B. It synthesizes critical masses of qualitative decrease the risk of further infection
findings. D. Plan activities to allow at least 8 hours of
C. It leads to a higher reliability of research uninterrupted sleep
findings. 2. A patient with a pulmonary embolus is
D. It cannot be conducted on historical or case receiving anticoagulation with IV heparin.
study findings. What instructions would you give the
17. What is meant by the fittingness of a nursing assistant who will help the patient
research study? with activities of daily living? Select all that
A. Truth of findings as judged by the participants apply.
B. The appropriateness of the interview A. Use a lift sheet when moving and positioning
questions posed the patient in bed
C. Faithfulness to everyday reality of the B. Use an electric razor when shaving the
participants patient each day
D. The adequacy of the coding system used C. Use a soft-bristled toothbrush or tooth sponge
18. How can qualitative outcome analysis be for oral care
used? Select all that apply. D. Use a rectal thermometer to obtain a more
A. To determine the reliability of intervention accurate body temperature
outcomes in a study E. Be sure the patients footwear has a firm sole
B. To confirm the applicability of clinical when the patient ambulates
strategies 3. A patient with acute respiratory distress
C. To develop interventions and then test those syndrome (ARDS) is receiving oxygen by a
selected nonrebreather mask, but arterial blood gas
D. To build theory measurements still show poor oxygenation.
19. When critiquing a qualitative study, As the nurse responsible for this patients
which of the following questions are helpful care, you would anticipate a physician order
in determining the studys auditability? for what action?
Select all that apply. A. Perform endotracheal intubation and initiate
A. Has adequate time been allowed to mechanical ventilation
understand the phenomenon fully?
B. Immediately begin continuous positive airway appropriate to the scope of practice of an
pressure (CPAP) via the patients nose and experienced LPN? Select all that apply.
mouth A. Auscultate breath sounds
C. Administer furosemide (Lasix) 100 mg IV B. Administer medications via metered-dose
push stat inhaler (MDI)
D. Call a code for respiratory arrest C. Complete in-depth admission assessment
4. A patient with chronic obstructive D. Initiate the nursing care plan
pulmonary disease (COPD). Which E. Evaluate the patients technique for using
intervention for airway management should MDIs
you delegate to a nursing assistant (PCT)? 9. The charge nurse is making assignments
A. Assisting the patient to sit up on the side of for the next shift. Which patient should be
the bed assigned to the fairly new nurse (6 months
B. Instructing the patient to cough effectively experience) pulled from the surgical unit to
C. Teaching the patient to use incentive the medical unit?
spirometry A. A 58-year old on airborne precautions for
D. Auscultation of breath sounds every 4 hours tuberculosis (TB)
5. A patient with sleep apnea has a nursing B. A 68-year old just returned from
diagnosis of Sleep Deprivation related to bronchoscopy and biopsy
disrupted sleep cycle. Which action should C. A 72-year old who needs teaching about the
you delegate to the nursing assistant (PCT)? use of incentive spirometry
A. Discuss weight-loss strategies such as diet D. A 69-year old with COPD who is ventilator
and exercise with the patient dependent
B. Teach the patient how to set up the BiPAP 10. The high-pressure alarm on a patients
machine before sleeping ventilator goes off. When you enter the room
C. Remind the patient to sleep on his side to assess the patient, who has ARDS, the
instead of his back. oxygen saturation monitor reads 87% and
D. Administer modafinil (Provigil) to promote the patient is struggling to sit up. Which
daytime wakefulness action should you take next?
6. After change of shift, you are assigned to A. Reassure the patient that the ventilator will do
care for the following patients. Which patient the work of breathing for him
should you assess first? B. Manually ventilate the patient while assessing
A. A 60-year old patient on a ventilator for whom possible reasons for the high-pressure alarm
a sterile sputum specimen must be sent to the C. Increase the fraction of inspired oxygen on
lab the ventilator to 100% in preparation for
B. A 55-year old with COPD and a pulse endotracheal suctioning
oximetry reading from the previous shift of 90% D. Insert an oral airway to prevent the patient
saturation from biting on the endotracheal tube
C. A 70-year old with pneumonia who needs to 11. The nursing assistant tells you that a
be started on intravenous (IV) antibiotics patient who is receiving oxygen at a flow rate
D. A 50-year old with asthma who complains of of 6 L/min by nasal cannula is complaining of
shortness of breath after using a bronchodilator nasal passage discomfort. What intervention
7. After the respiratory therapist performs should you suggest to improve the patients
suctioning on a patient who is intubated, the comfort for this problem?
nursing assistant measures vital signs for A. Suggest that the patients oxygen be
the patient. Which vital sign value should the humidified
nursing assistant report to the RN B. Suggest that a simple face mask be used
immediately? instead of a nasal cannula.
A. Heart rate of 98 beats/min C. Suggest that the patient be provided with an
B. Respiratory rate of 24 breaths/min extra pillow
C. Blood pressure of 168/90 mm Hg D. Suggest that the patient sit up in a chair at
D. Tympanic temperature of 101.4 F (38.6 C) the bedside
8. An experienced LPN, under the 12. The patient with COPD has a nursing
supervision of the team leader RN, is diagnosis of Ineffective Breathing Pattern.
providing nursing care for a patient with a Which is an appropriate action to delegate to
respiratory problem. Which actions are the experienced LPN under your
supervision?
A. Observe how well the patient performs D. Switch the patient to a nonrebreather mask at
pursed-lip breathing 95% to 100% oxygen and call the physician to
B. Plan a nursing care regimen that gradually discuss the patients status.
increases activity intolerance 17. Which intervention for a patient with a
C. Assist the patient with basic activities of daily pulmonary embolus could be delegated to
living the LPN on your patient care team?
D. Consult with the physical therapy department A. Evaluating the patients complaint of chest
about reconditioning exercises pain
13. The patient with COPD tells the nursing B. Monitoring laboratory values for changes in
assistant that he did not get his annual flu oxygenation
shot this year and has not had a pneumonia C. Assessing for symptoms of respiratory failure
vaccination. You would be sure to instruct D. Auscultating the lungs for crackles
the nursing assistant to report which of 18. Which of these medication orders for a
these? patient with a pulmonary embolism is more
A. Blood pressure of 152/84 mm Hg important to clarify with the prescribing
B. Respiratory rate of 27 breaths/min physician before administration?
C. Heart rate of 92 beats/min A. Warfarin (Coumadin) 1.0 mg by mouth (PO)
D. Oral temperature of 101.2 F (38.4C) B. Morphine sulfate 2 to 4 mg IV
14. To improve respiratory status, which C. Cephalexin (Keflex) 250 mg PO
medication should you be prepared to D. Heparin infusion at 900 units/hr
administer to the newborn infant with RDS? 19. You are a team leader RN working with a
A. Theophylline (Theolair, Theochron) student nurse. The student nurse is to teach
B. Surfactant (Exosurf) a patient how to use and MDI without a
C. Dexamethasone (Decadron) spacer. Put in correct order the steps that
D. Albuterol (Proventil) the student nurse should teach the patient.
15. When a patient with TB is being prepared A. Remove the inhaler cap and shake the
for discharge, which statement by the patient inhaler
indicates the need for further teaching? B. Open your mouth and place the mouthpiece 1
A. Everyone in my family needs to go and see to 2 inches away
the doctor for TB testing. C. Tilt your head back and breathe out fully
B. I will continue to take my isoniazid until I am D. Hold your breath for at least 10 seconds
feeling completely well. E. Press down firmly on the canister and breathe
C. I will cover my mouth and nose when I deeply through your mouth
sneeze or cough and put my used tissues in a F. Wait at least 1 minute between puffs.
plastic bag. 20. You are acting as preceptor for a newly
D. I will change my diet to include more foods graduated RN during her second week of
rich in iron, protein, and vitamin C. orientation. You would assign the new RN
16. When assessing a 22-year old patient under your supervision to provide care to
who required emergency surgery and which patients? Select all that apply.
multiple transfusion 3 days ago, you find that A. A 38-year old with moderate persistent
the patient looks anxious and has labored asthma awaiting discharge
respirations at the rate of 38 breaths/min. B. A 63-year old with a tracheostomy needing
The oxygen saturation is 90% with the tracheostomy care every shift.
oxygen delivery at 6 L/min via nasal cannula. C. A 56-year old with lung cancer who has just
Which action is most appropriate? undergone left lower lobectomy
A. Increase the flow rate on the oxygen to 10 D. A 49-year old just admitted with a new
L/min and reassess the patient after about 10 diagnosis of esophageal cancer.
minutes
B. Assist the patient in using the incentive Prioritization, Delegation and Assignment 2
spirometer and splint his chest with a pillow 1. Jenna is a nurse from the medical-surgical
while he coughs unit of a tertiary hospital. She was asked to
C. Administer the ordered morphine sulfate to float on the orthopedic in which she has no
the patient to decrease his anxiety and reduce prior experience on working on. Which client
the hyperventilation should be assigned to her?
A. A client with a cast for a fractured femur and D. A 78-year-old female client with pressure
who has numbness and discoloration of the ulcer that needs dressing change.
toes. 7. Nurse Janus enters a room and finds a
B. A client with balanced skeletal traction and client lying on the floor. Which of the
needs assistance with morning care. following actions should the nurse perform
C. A client who had an above-the-knee first?
amputation yesterday and has currently has a A. Call for help to get the client back in bed.
temperature of 101.4F. B. Establish whether the client is responsive.
D. A client who had a total hip replacement two C. Assist the client back to bed.
days ago and needs blood glucose monitoring. D. Ask the client what happened.
2. Sally is a nurse working in an emergency 8. Paige is a nurse preceptor who is working
department and receives a client after a with a new nurse Joyce. She notes that the
radiological incident. Which task is utmost Joyce is reluctant to delegate tasks to
priority for the nurse to do first? members of the care team. Paige recognizes
A. Decontaminate the clients clothing. that this reluctance is mostly likely due to:
B. Decontaminate the open wound on the A. Role modeling behaviors of the preceptor.
clients thigh. B. The philosophy of the new nurses school of
C. Decontaminate the examination room the nursing.
client is placed in. C. The orientation provided to the new nurse.
D. Save the clients vomitus for analysis by the D. Lack of trust to the members of the
radiation safety staff. healthcare team.
3. The nurse plans care for a client in the 9. Nurse Paul is developing a care plan for a
post anesthesia care unit. The nurse should client after bariatric surgery for morbid
assess first the clients: obesity. The nurse should include which of
A. respiratory status. the following on the care plan as the priority
B. level of consciousness. complication to prevent:
C. level of pain. A. pain.
D. reflexes and movement of extremities. B. wound infection.
4. Nurse Jackie is reviewing the diet of a 28- C. depression.
year-old female who reports several months D. thrombophlebitis.
of intermittent abdominal pain, abdominal 10. A client presents to the emergency room
bloating, and flatulence. The nurse should with dyspnea, chest pain, and syncope. The
tell the client to avoid: nurse assesses the client and notes that the
A. fiber. following assessment cues: pale,
B. broccoli. diaphoretic, blood pressure of 90/60,
C. yogurt. respirations of 33. The client is also anxious
D. simple carbohydrates. and fearing death. Which action should the
5. Nurse Joriz of Nurseslabs Medical Center nurse take first?
is planning care for a client who will undergo A. Administer pain medications.
a colposcopy. Which of the following actions B. Administer IV fluids.
should Joriz take first? C. Administer dopamine.
A. Discuss the clients fear regarding potential D. Administer oxygen via nasal cannula.
cervical cancer. 11. Nurse Pietro receives a 11-month old
B. Assist with silver nitrate application to the child with a fracture of the left femur on the
cervix to control bleeding. pediatric unit. Which action is important for
C. Give instructions regarding douching and the nurse to take first?
sexual relations. A. Call for a social worker to meet with the
D. Administer pain medications. family.
6. Nurse Channing is caring for four clients B. Check the childs blood pressure, pulse,
and is preparing to do his initial rounds. respiration, and temperature.
Which client should the nurse assess first? C. Administer pain medications.
A. A client with diabetes being discharged today. D. Speak with the parents about how the
B. A 35-year-old male with tracheostomy and fracture occurred.
copious secretions. 12. Nurse Skye is on the cardiac unit caring
C. A teenager scheduled for physical therapy for four clients. He is preparing to do initial
this morning.
rounds. Which client should the nurse D. The patient was on bed rest for 6 hours after
assess first? a diagnostic procedure
A. A client scheduled for cardiac ultrasound this 2. You are assigned to provide nursing care
morning. for a patient receiving mechanical
B. A client with syncope being discharged today. ventilation. Which action should you
C. A client with chronic bronchitis on nasal delegate to an experienced nursing
oxygen. assistant?
D. A client with diabetic foot ulcer that needs a A. Assessing the patients respiratory status
dressing change. every 4 hours
13. A nurse enters a room and finds a patient B. Taking vital signs and pulse oximetry
lying face down on the floor and bleeding readings every 4 hours
from a gash in the head. Which action should C. Checking the ventilator settings to make sure
the nurse perform first? they are as prescribed
A. Determine the level of consciousness. D. Observing whether the patients tube needs
B. Push the call button for help. suctioning every 2 hours
C. Turn the client face up to assess. 3. You are caring for a patient with
D. Go out in the hall to get the nursing assistant emphysema and respiratory failure who is
to stay with the client while the nurse calls the receiving mechanical ventilation through an
physician. endotracheal tube. To prevent ventilator-
14. Nurse Adonai is working on the night associated pneumonia (VAP), which action is
shift with a nursing assistant. The nursing most important to include in the plan of
assistant comes to the nurse stating that the care?
other nurse working on the unit is not A. Administer ordered antibiotics as scheduled
assessing a client with abdominal pain B. Hyperoxygenate the patient before suctioning
despite multiple requests. Which of the C. Maintain the head of the bed at a 30 to 45-
following actions by the nurse is best? degree angle
A. Ask the other nurse if she needs help. D. Suction the airway when coarse crackles are
B. Assess the client and let the other nurse audible
know what should be done. 4. You are evaluating and assessing a patient
C. Ask the client if he is satisfied with his care. with a diagnosis of chronic emphysema. The
D. Contact the nursing supervisor to address the patient is receiving oxygen at a flow rate of 5
situation. L/min by nasal cannula. Which finding
15. Nurse Vivian is reviewing immunizations concerns you immediately?
with the caregiver of a 72-year-old client with A. The patient has fine bibasilar crackles
a history of cerebrovascular disease. The B. The patients respiratory rate is 8 breaths/min.
caregiver learns that which immunization is a C. The patient sits up and leans over the night
priority for the client? table.
A. Hepatitis A vaccine. D. The patient has a large barrel chest.
B. Lymes disease vaccine. 5. You are initiating a nursing care plan for a
C. Hepatitis B vaccine. patient with pneumonia. Which intervention
D. Pneumococcal vaccine. for cough enhancement should you delegate
to a nursing assistant?
Prioritization, Delegation and Assignment 3 A. Teaching the patient about the importance of
1. You are admitting a patient for whom a adequate of fluid intake and hydration.
diagnosis of pulmonary embolus must be B. Assisting the patient to a sitting position with
ruled out. The patients history and neck flexed, shoulders relaxed, and knees
assessment reveal all of these findings. flexed
Which finding supports the diagnosis of C. Reminding the patient to use an incentive
pulmonary embolus? spirometer every 1 to 2 hours while awake
A. The patient was recently in a motor vehicle D. Encouraging the patient to take a deep
accident breath, hold it for 2 seconds, then cough two or
B. The patient participated in an aerobic three times in succession.
exercise program for 6 months 6. You are making a home visit to a 50-year
C. The patient gave birth to her youngest child 1 old patient who was recently hospitalized
year ago with a right leg deep vein thrombosis and a
pulmonary embolism. The patients only
medication is enoxaparin (Lovenox) B. Continuous bubbling in the water seal
subcutaneously. Which assessment chamber
information will you need to communicate to C. Complaints of pain at the chest tube site
the physician? D. Chest tube dressing dated yesterday
A. The patient says that her right leg aches all 11. You are supervising a student nurse who
night is performing tracheostomy care for a
B. The right calf is warm to the touch and is patient. For which action by the student
larger than the left calf should you intervene?
C. The patient is unable to remember her A. Suctioning the tracheostomy tube before
husbands first name performing tracheostomy care
D. There are multiple ecchymotic areas on the B. Removing old dressings and cleaning off
patients arms excess secretions
7. You are providing care for a patient with C. Removing the inner cannula and cleaning
recently diagnosed asthma. Which key using universal precautions
points would you be sure to include in your D. Replacing the inner cannula and cleaning the
teaching plan for this patient? (Select all that stoma site.
apply) E. Changing the soiled tracheostomy ties and
A. Avoid potential environmental asthma triggers securing the tube in place
such as smoke 12. You are supervising an RN who was
B. Use the inhaler 30 minutes before exercising pulled from the medical-surgical floor to the
to prevent bronchospasm emergency department. The nurse is
C. Wash all bedding in cold water to reduce and providing care for a patient admitted with
destroy dust mites. anterior epistaxis (nosebleed). Which of
D. Be sure to get at least 8 hours of rest and these directions would you clearly proved to
sleep every night. the RN? (Select all that apply)
E. Avoid foods prepared with monosodium A. Position the patient supine and turned on his
glutamate (MSG) side
8. You are providing nursing care for a B. Apply direct lateral pressure to the nose for 5
newborn infant with respiratory distress minutes
syndrome (RDS) who is receiving nasal C. Maintain universal body substances
CPAP ventilation. What complications precautions.
should you monitor for this infant? D. Apply ice or cool compresses to the nose
A. Pulmonary embolus E. Instruct the patient not to blow the nose for
B. Bronchitis several hours.
C. Pneumothorax 13. You are the preceptor for an RN who is
D. Pneumonia undergoing orientation to the intensive care
9. You are responsible for the care of a unit. The RN is providing care for a patient
postoperative patient with a thoracotomy. with ARDS who has just been intubated in
The patient has been given a nursing preparation for mechanical ventilation. You
diagnosis of Activity Intolerance. Which observe the nurse perform all of these
action should you delegate to the nursing actions. For which action must you intervene
assistant? immediately?
A. Instructing the patient to alternate rest and A. Assessing for bilateral breath sounds and
activity periods symmetrical chest movements
B. Encouraging, monitoring, and recording B. Auscultating over the stomach to rule out
nutritional intake esophageal intubation
C. Monitoring cardiorespiratory response to C. Marking the tube 1 cm from where it touches
activity the incisor tooth or nares
D. Planning activities for periods when the D. Ordering a chest radiograph to verify that
patient has the most energy tube placement is correct
10. You are supervising a nursing student 14. You have just finished assisting the
who is providing care for a thoracotomy physician with a thoracentesis for a patient
patient with a chest tube. What finding would with recurrent left pleural effusion caused by
you clearly instruct the nursing student to lung cancer. The thoracentesis removed
notify you about immediately? 1800 mL of fluid. Which patient assessment
A. Chest tube drainage of 10 to 15 mL/hr
information is important to report to the B. Respiratory Acidosis, Partially Compensated
physician? C. Metabolic Alkalosis, Uncompensated
A. The patient starts crying and says she cant D. Metabolic Acidosis, Partially, Compensated
go on with treatment much longer 3. A cigarette vendor was brought to the
B. The patient complains of sharp, stabbing emergency department of a hospital after
chest pain with every deep breath she fell into the ground and hurt her left leg.
C. The patients blood pressure is 100/48 mm She is noted to be tachycardic and
Hg and her heart rate is 102 beats/ min tachypneic. Painkillers were carried out to
D. The patients dressing at the thoracentesis lessen her pain. Suddenly, she started
site has 1 cm of bloody drainage complaining that she is still in pain and now
15. You have obtained the following experiencing muscle cramps, tingling, and
assessment information about a 3-year old paraesthesia. Measurement of arterial blood
who has just returned to the pediatric unit gas reveals pH 7.6, PaO2 120 mm Hg, PaCO2
after having a tonsillectomy. Which finding 31 mm Hg, and HCO3 25 mmol/L. What does
requires the most immediate follow-up? this mean?
A. Frequent swallowing A. Respiratory Alkalosis, Uncompensated
B. Hypotonic bowel sounds B. Respiratory Acidosis, Partially Compensated
C. Complaints of a sore throat C. Metabolic Alkalosis, Uncompensated
D. Heart rate of 112 beats/min D. Metabolic Alkalosis, Partially Compensated
4. Rickys grandmother is suffering from
Acid-Base Balance Analysis 1 persistent vomiting for two days now. She
1. George Kent is a 54 year old widower with appears to be lethargic and weak and has
a history of chronic obstructive pulmonary myalgia. She is noted to have dry mucus
disease and was rushed to the emergency membranes and her capillary refill takes >4
department with increasing shortness of seconds. She is diagnosed as having
breath, pyrexia, and a productive cough with gastroenteritis and dehydration.
yellow-green sputum. He has difficulty in Measurement of arterial blood gas shows pH
communicating because of his inability to 7.5, PaO2 85 mm Hg, PaCO2 40 mm Hg, and
complete a sentence. One of his sons, HCO3 34 mmol/L. What acid-base disorder is
Jacob, says he has been unwell for three shown?
days. Upon examination, crackles and A. Respiratory Alkalosis, Uncompensated
wheezes can be heard in the lower lobes; he B. Respiratory Acidosis, Partially Compensated
has a tachycardia and a bounding pulse. C. Metabolic Alkalosis, Uncompensated
Measurement of arterial blood gas shows pH D. Metabolic Alkalosis, Partially Compensated
7.3, PaCO2 68 mm Hg, HCO3 28 mmol/L, and 5. Mrs. Johansson, who had undergone
PaO2 60 mm Hg. How would you interpret surgery in the post-anesthesia care unit
this? (PACU), is difficult to arouse two hours
A. Respiratory Acidosis, Uncompensated following surgery. Nurse Florence in the
B. Respiratory Acidosis, Partially Compensated PACU has been administering Morphine
C. Metabolic Alkalosis, Uncompensated Sulfate intravenously to the client for
D. Metabolic Acidosis, Partially Compensated complaints of post-surgical pain. The clients
2. Carl, an elementary student, was rushed to respiratory rate is 7 per minute and
the hospital due to vomiting and a decreased demonstrates shallow breathing. The patient
level of consciousness. The patient displays does not respond to any stimuli! The nurse
slow and deep (Kussmaul breathing), and he assesses the ABCs (remember Airway,
is lethargic and irritable in response to Breathing, Circulation!) and obtains ABGs
stimulation. He appears to be dehydrated STAT! Measurement of arterial blood gas
his eyes are sunken and mucous membranes shows pH 7.10, PaCO2 70 mm Hg and HCO3
are dryand he has a two week history of 24 mEq/L. What does this mean?
polydipsia, polyuria, and weight loss. A. Respiratory Alkalosis, Partially Compensated
Measurement of arterial blood gas shows pH B. Respiratory Acidosis, Uncompensated
7.0, PaO2 90 mm Hg, PaCO2 23 mm Hg, and C. Metabolic Alkalosis, Partially Compensated
HCO3 12 mmol/L; other results are Na+ 126 D. Metabolic Acidosis, Uncompensated
mmol/L, K+ 5 mmol/L, and Cl- 95 mmol/L. 6. Baby Angela was rushed to the
What is your assessment? Emergency Room following her mothers
A. Respiratory Acidosis, Uncompensated complaint that the infant has been irritable,
difficult to breastfeed and has had diarrhea 10. Anne, who is drinking beer at a party,
for the past 3 days. The infants respiratory falls and hits her head on the ground. Her
rate is elevated and the fontanels are friend Liza dials 911 because Anne is
sunken. The Emergency Room physician unconscious, depressed ventilation (shallow
orders ABGs after assessing the ABCs. The and slow respirations), rapid heart rate, and
results from the ABG results show pH 7.39, is profusely bleeding from both ears. Which
PaCO2 27 mmHg and HCO3 19 mEq/L. What primary acid-base imbalance is Anne at risk
does this mean? for if medical attention is not provided?
A. Respiratory Alkalosis, Fully Compensated A. Metabolic Acidosis
B. Metabolic Acidosis, Uncompensated B. Metabolic Alkalosis
C. Metabolic Acidosis, Fully Compensated C. Respiratory Acidosis
D. Respiratory Acidosis, Uncompensated D. Respiratory Alkalosis
7. Mr. Wales, who underwent post-abdominal
surgery, has a nasogastric tube. The nurse Acid Base Balance Analysis 2
on duty notes that the nasogastric tube 1. Dave, a 6-year-old boy, was rushed to the
(NGT) is draining a large amount (900 cc in 2 hospital following her mothers complaint
hours) of coffee ground secretions. The that her son has been vomiting, nauseated
client is not oriented to person, place, or and has overall weakness. After series of
time. The nurse contacts the attending tests, the nurse notes the laboratory results:
physician and STAT ABGs are ordered. The potassium: 2.9 mEq. Which primary acid-
results from the ABGs show pH 7.57, PaCO2 base imbalance is this boy at risk for if
37 mmHg and HCO3 30 mEq/L. What is your medical intervention is not carried out?
assessment? A. Respiratory Acidosis
A. Metabolic Acidosis, Uncompensated B. Respiratory Alkalosis
B. Metabolic Alkalosis, Uncompensated C. Metabolic Acidosis
C. Respiratory Alkalosis, Uncompensated D. Metabolic Alkalosis
D. Metabolic Alkalosis, Partially Compensated 2. An old beggar was admitted to the
8. Client Z is admitted to the hospital and is emergency department due to shortness of
to undergo brain surgery. The client is very breath, fever, and a productive cough. Upon
anxious and scared of the upcoming examination, crackles and wheezes are
surgery. He begins to hyperventilate and noted in the lower lobes; he appears to be
becomes very dizzy. The client loses tachycardic and has a bounding pulse.
consciousness and the STAT ABGs reveal Measurement of arterial blood gas shows pH
pH 7.61, PaCO2 22 mmHg and HCO3 25 7.2, PaCO2 66 mm Hg, HCO3 27 mmol/L, and
mEq/L. What is the ABG interpretation based PaO2 65 mm Hg. As a knowledgeable nurse,
on the findings? you know that the normal value for pH is:
A. Metabolic Acidosis, Uncompensated A. 7.20
B. Respiratory Alkalosis, Partially Compensated B. 7.30
C. Respiratory Alkalosis, Uncompensated C. 7.40
D. Metabolic Alkalosis, Partially Compensated D. 7.50
9. Three-year-old Adrian is admitted to the 3. Lizas mother is seen in the emergency
hospital with a diagnosis of asthma and department at a community hospital. She
respiratory distress syndrome. The mother admits that her mother is taking many tablets
of the child reports to the nurse on duty that of aspirin (salicylates) over the last 24-hour
she has witnessed slight tremors and period because of a severe headache. Also,
behavioral changes in her child over the past the mother complains of an inability to
four days. The attending physician orders urinate. The nurse on duty took her vital
routine ABGs following an assessment of the signs and noted the following: Temp = 97.8
ABCs. The ABG results are pH 7.35, PaCO2 F; apical pulse = 95; respiration = 32 and
72 mmHg and HCO3 38 mEq/L. What acid- deep. Which primary acid-base imbalance is
base disorder is shown? the gentleman at risk for if medical attention
A. Respiratory Acidosis, Uncompensated is not provided?
B. Respiratory Acidosis, Fully Compensated A. Respiratory Acidosis
C. Respiratory Alkalosis, Fully Compensated B. Respiratory Alkalosis
D. Metabolic Alkalosis, Partially Compensated C. Metabolic Acidosis
D. Metabolic Alkalosis
4. A patient who is hospitalized due to B. The pH will rise and PCO2 will fall.
vomiting and a decreased level of C. Both will be higher than normal due to the
consciousness displays slow and deep physical exertion.
(Kussmaul breathing), and he is lethargic D. The pH will fall and PCO2 will rise
and irritable in response to stimulation. The 9. A young woman is found comatose,
doctor diagnosed him of having dehydration. having taken an unknown number of
Measurement of arterial blood gas shows pH sleeping pills an unknown time before. An
7.0, PaO2 90 mm Hg, PaCO2 22 mm Hg, and arterial blood sample yields the following
HCO3 14 mmol/L; other results are Na+ 120 values: pH 6.90, HCO3- 13 meq/liter and
mmol/L, K+ 2.5 mmol/L, and Cl- 95 mmol/L. PaCO2 68 mmHg. This patients acid-base
As a knowledgeable nurse, you know that status is most accurately described as:
the normal value for PaCO2 is: A. Metabolic Acidosis
A. 22 mm Hg B. Respiratory Acidosis
B. 36 mm Hg C. Simultaneous Respiratory and Metabolic
C. 48 mm Hg Acidosis
D. 50 mm Hg D. Respiratory Acidosis with Complete Renal
5. A company driver is found at the scene of Compensation
an automobile accident in a state of 10. A mother is admitted in the emergency
emotional distress. He tells the paramedics department following complaints of fever
that he feels dizzy, tingling in his fingertips, and chills. The nurse on duty took her vital
and does not remember what happened to signs and noted the following: Temp = 100
his car. Respiratory rate is rapid at F; apical pulse = 95; respiration = 20 and
34/minute. Which primary acid-base deep. Measurement of arterial blood gas
disturbance is the young man at risk for if shows pH 7.37, PaO2 90 mm Hg, PaCO2 40
medical attention is not provided? mm Hg, and HCO3 24 mmol/L. What is your
A. Respiratory Acidosis assessment?
B. Respiratory Alkalosis A. Hyperthermia
C. Metabolic Acidosis B. Hyperthermia and Respiratory Alkalosis
D. Metabolic Alkalosis C. Hypothermia
6. An elderly client was admitted to hospital D. Hypothermia and Respiratory Alkalosis
in a coma. Analysis of the arterial blood gave
the following values: PCO2 16 mm Hg, Acid Base Balance Analysis 3
HCO3- 5 mmol/L and pH 7.1. As a well- In acid-base balance the normal plasma
rounded nurse, you know that the normal PCO2 and bicarbonate levels are disturbed.
value for HCO3 is: Match the changes in these parameters
A. 20 mmol/L given below with the disorders in the given
B. 24 mmol/L choices.
C. 29 mmol/L 1. Low plasma PaCO2
D. 31 mmol/L A. Metabolic Acidosis
7. In a patient undergoing surgery, it was B. Respiratory Alkalosis
vital to aspirate the contents of the upper C. Metabolic Alkalosis
gastrointestinal tract. After the operation, the D. Respiratory Acidosis
following values were acquired from an 2. High plasma PaCO2
arterial blood sample: pH 7.55, PCO2 52 mm A. Metabolic Acidosis
Hg and HCO3- 40 mmol/l. What is the B. Respiratory Alkalosis
underlying disorder? C. Metabolic Alkalosis
A. Respiratory Acidosis D. Respiratory Acidosis
B. Respiratory Alkalosis 3. Decreased plasma bicarbonate (HCO3-)
C. Metabolic Acidosis A. Metabolic Acidosis
D. Metabolic Alkalosis B. Respiratory Alkalosis
8. A mountaineer attempts an assault on a C. Metabolic Alkalosis
high mountain in the Andes and reaches an D. Respiratory Acidosis
altitude of 5000 meters (16,400 ft) above sea 4. Increased plasma bicarbonate (HCO3-)
level. What will happen to his arterial PCO2 A. Metabolic Acidosis
and pH? B. Respiratory Alkalosis
A. Both will be lower than normal. C. Metabolic Alkalosis
D. Respiratory Acidosis 12. pH 7.39, PaCO2 44, HCO3- 26
5. What two organs in the body serve as a A. Respiratory Acidosis
compensatory function to maintain acid base B. Metabolic Acidosis
balance? C. Respiratory Alkalosis
A. Kidneys and Lungs D. Normal
B. Lungs and Spleen 13. pH 7.55, PaCO2 25, HCO3- 22
C. Heart and Liver A. Respiratory Acidosis, Partially Compensated
D. Gallbladder and Appendix B. Respiratory Alkalosis, Uncompensated
6. Arterial blood gas (ABG) measurement will C. Metabolic Alkalosis, Partially Compensated
give the information needed to determine if D. Metabolic Acidosis, Uncompensated
the primary disturbance of acid-base balance 14. pH 7.17, PaCO2 48, HCO3- 36
is respiratory or metabolic in nature. A. Respiratory Acidosis, Uncompensated
A. True B. Metabolic Acidosis, Partially Compensated
B. False C. Respiratory Alkalosis, Partially Compensated
C. Both Carbonic Acid Excess and Deficit Only D. Respiratory Acidosis, Partially Compensated
D. Both Bicarbonate Excess and Deficit Only 15. pH 7.34, PaCO2 24, HCO3- 20
7. The major effect of acidosis is A. Respiratory Acidosis, Partially Compensated
overexcitement of the central nervous B. Metabolic Acidosis, Partially Compensated
system. C. Metabolic Acidosis, Uncompensated
A. True D. Metabolic Alkalosis, Partially Compensated
B. False 16. pH 7.64, PaCO2 25, HCO3- 19
C. Maybe A. Respiratory Acidosis, Uncompensated
D. Both Acidosis and Alkalosis result in B. Respiratory Alkalosis, Partially Compensated
overexcitement of the central nervous system. C. Respiratory Alkalosis, Uncompensated
8. Alkalosis is characterized by D. Metabolic Alkalosis, Partially Compensated
overexcitement of the nervous system. 17. pH 7.45, PaCO2 50, HCO3- 30
A. True A. Metabolic Alkalosis, Fully Compensated
B. False B. Respiratory Alkalosis, Fully Compensated
C. The major effect of Alkalosis is a depression C. Metabolic Alkalosis, Partially Compensated
of the central nervous system. D. Respiratory Acidosis, Partially Compensated
D. Both Acidosis and Alkalosis result in 18. pH 7.6, PaCO2 53, HCO3- 38
overexcitement of the central nervous system. A. Metabolic Alkalosis, Partially Compensated
9. The human body functions optimally in a B. Metabolic Alkalosis, Fully Compensated
state of homeostasis. C. Respiratory Acidosis, Partially Compensated
A. True D. Respiratory Alkalosis, Fully Compensated
B. False 19. pH 7.5, PaCO2 19, HCO3- 22
C. Maybe A. Respiratory Alkalosis, Partially Compensated
D. Homeostasis has nothing to do with B. Metabolic Alkalosis, Partially Compensated
metabolic balance. C. Respiratory Acidosis, Uncompensated
10. Acids have no hydrogen ions and are D. Respiratory Alkalosis, Uncompensated
able to bind in a solution. 20. pH 7.4, PaCO2 59, HCO3- 35
A. True A. Respiratory Acidosis, Uncompensated
B. False B. Metabolic Alkalosis, Uncompensated
C. Acid is a substance that is not capable of C. Respiratory Acidosis, Fully Compensated
donating hydrogen ions. D. Metabolic Alkalosis, Partially Compensated
D. Acids and bases have nothing to do with
hydrogen ions. Nursing Leadership and Management 1
Match the acid-base status of the following 1. Ms. Caputo is newly-promoted to a patient
blood samples to the disorders in the given care manager position. She updates her
choices. (PaCO2 values are in mm Hg and knowledge on the theories in management
bicarbonate values in mmol/l). and leadership in order to become effective
11. pH 7.57, PaCO2 22, HCO3- 17 in her new role. She learns that some
A. Respiratory Acidosis, Partially Compensated managers have low concern for services and
B. Respiratory Alkalosis, Uncompensated high concern for staff. Which style of
C. Metabolic Acidosis, Partially Compensated management refers to this?
D. Respiratory Alkalosis, Partially Compensated A. Organization Man
B. Impoverished Management following styles best fits a situation when the
C. Country Club Management followers are self-directed, experts and are
D. Team Management matured individuals?
2. Her former manager demonstrated A. Democratic
passion for serving her staff rather than B. Authoritarian
being served. She takes time to listen, C. Laissez faire
prefers to be a teacher first before being a D. Bureaucratic
leader, which is characteristic of 8. She surfs the internet for more information
A. Transformational leader about leadership styles. She reads about
B. Transactional leader shared leadership as a practice in some
C. Servant leader magnet hospitals. Which of the following
D. Charismatic leader describes this style of leadership?
3. On the other hand, Ms. Caputo notices that A. Leadership behavior is generally determined
the Chief Nurse Executive has charismatic by the relationship between the leaders
leadership style. Which of the following personality and the specific situation
behaviors best describes this style? B. Leaders believe that people are basically
A. Possesses inspirational quality that makes good and need not be closely controlled
followers gets attracted of him and regards him C. Leaders rely heavily on visioning and inspire
with reverence members to achieve results
B. Acts as he does because he expects that his D. Leadership is shared at the point of care.
behavior will yield positive results 9. Ms. Caputo learns that some leaders are
C. Uses visioning as the core of his leadership transactional leaders. Which of the following
D. Matches his leadership style to the situation does NOT characterize a transactional
at hand. leader?
4. Which of the following conclusions of Ms. A. Focuses on management tasks
Caputo about leadership characteristics is B. Is a caretaker
TRUE? C. Uses trade-offs to meet goals
A. There is a high correlation between the D. Inspires others with vision
communication skills of a leader and the ability 10. She finds out that some managers have
to get the job done. benevolent-authoritative style of
B. A manager is effective when he has the ability management. Which of the following
to plan well. behaviors will she exhibit most likely?
C. Assessment of personal traits is a reliable A. Have condescending trust and confidence in
tool for predicting a managers potential. their subordinates
D. There is good evidence that certain personal B. Gives economic or ego awards
qualities favor success in managerial role. C. Communicates downward to the staff
5. She reads about Path Goal theory. Which D. Allows decision making among subordinates
of the following behaviors is manifested by 11. Henry is a Unit Manager I the Medical
the leader who uses this theory? Unit. He is not satisfied with the way things
A. Recognizes staff for going beyond are going in his unit. Patient satisfaction rate
expectations by giving them citations is 60% for two consecutive months and staff
B. Challenges the staff to take individual morale is at its lowest. He decides to plan
accountability for their own practice and initiate changes that will push for a
C. Admonishes staff for being laggards. turnaround in the condition of the unit.
D. Reminds staff about the sanctions for non Which of the following actions is a priority
performance. for Henry?
6. One leadership theory states that leaders A. Call for a staff meeting and take this up in the
are born and not made, which refers to agenda.
which of the following theories? B. Seek help from her manager.
A. Trait C. Develop a strategic action on how to deal
B. Charismatic with these concerns.
C. Great Man D. Ignore the issues since these will be resolved
D. Situational naturally.
7. She came across a theory which states 12. He knows that there are external forces
that the leadership style is effective that influence changes in his unit. Which of
dependent on the situation. Which of the the following is NOT an external force?
A. Memo from the CEO to cut down on electrical D. Coordinating with other members of the team
consumption 19. When Henry uses team nursing as a care
B. Demands of the labor sector to increase delivery system, he and his team need to
wages assess the priority of care for a group of
C. Low morale of staff in his unit patients, which of the following should be a
D. Exacting regulatory and accreditation priority?
standards A. Each patient as listed on the worksheet
13. After discussing the possible effects of B. Patients who needs least care
the low patient satisfaction rate, the staff C. Medications and treatments required for all
started to list down possible strategies to patients
solve the problems head-on. Should they D. Patients who need the most care
decide to vote on the best change strategy, 20. He is hopeful that his unit will make a big
which of the following strategies is referred turnaround in the succeeding months. Which
to this? of the following actions of
A. Collaboration Henry demonstrates that he has reached the
B. Majority rule third stage of change?
C. Dominance A. Wonders why things are not what it used to
D. Compromise be
14. One staff suggests that they review the B. Finds solutions to the problems
pattern of nursing care that they are using, C. Integrate the solutions to his day-to-day
which is described as a: activities
A. job description D. Selects the best change strategy
B. system used to deliver care 21. Joey is a newly-appointed nurse manager
C. manual of procedure of The Holy Spirit Medical Center, a tertiary
D. rules to be followed hospital located within the heart of the
15. Which of the following is TRUE about metropolis. He thinks of scheduling planning
functional nursing? workshop with his staff in order to ensure an
A. Concentrates on tasks and activities effective and efficient management of the
B. Emphasizes use of group collaboration department. Should he decide to conduct a
C. One-to-one nurse-patient ratio strategic planning workshop, which of the
D. Provides continuous, coordinated and following is NOT a characteristic of this
comprehensive nursing services activity?
16. Functional nursing has some A. Long-term goal-setting
advantages, which one is an EXCEPTION? B. Extends to 3-5 years in the future
A. Psychological and sociological needs are C. Focuses on routine tasks
emphasized. D. Determines directions of the organization
B. Great control of work activities. 22. Which of the following statements refer
C. Most economical way of delivering nursing to the vision of the hospital?
services. A. The Holy Spirit Medical Center is a
D. Workers feel secure in dependent role trendsetter in tertiary health care in the next five
17. He raised the issue on giving priority to year
patient needs. Which of the following offers B. The officers and staff of The Holy
the best way for setting priority? Spirit Medical Center believe in the unique
A. Assessing nursing needs and problems nature of the human person
B. Giving instructions on how nursing care C. All the nurses shall undergo continuing
needs are to be met competency training program.
C. Controlling and evaluating the delivery of D. The Holy Spirit Medical Center aims to
nursing care provide a patient-centered care in a total healing
D. Assigning safe nurse: patient ratio environment.
18. Which of the following is the best 23. The statement, The Holy Spirit Medical
guarantee that the patients priority needs Center aims to provide patient-centered care
are met? in a total healing environment refers to
A. Checking with the relative of the patient which of the following?
B. Preparing a nursing care plan in collaboration A. Vision
with the patient B. Goal
C. Consulting with the physician C. Philosophy
D. Mission C. Establish rapport with patients.
24. Joey plans to revisit the organizational D. Reduce response time to two minutes.
chart of the department. He plans to create a 30. He wants to influence the customary way
new position of a Patient Educator who has a of thinking and behaving that is shared by
coordinating relationship with the head the members of the department. Which of the
nurse in the unit. Which of the following will following terms refer to this?
likely depict this organizational relationship? A. Organizational chart
A. Box B. Cultural network
B. Solid line C. Organizational structure
C. Broken line D. Organizational culture
D. Dotted line 31. He asserts the importance of promoting a
25. He likewise stresses the need for all the positive organizational culture in their unit.
employees to follow orders and instructions Which of the following behaviors indicate
from him and not from anyone else. Which of that this is attained by the group?
the following principles does he refer to? A. Proactive and caring with one another
A. Scalar chain B. Competitive and perfectionist
B. Discipline C. Powerful and oppositional
C. Unity of command D. Obedient and uncomplaining
D. Order 32. Stephanie is a new Staff Educator of a
26. Joey orients his staff on the patterns of private tertiary hospital. She conducts
reporting relationship throughout the orientation among new staff nurses in her
organization. Which of the following department. Joseph, one of the new staff
principles refer to this? nurses, wants to understand the channel of
A. Span of control communication, span of control and lines of
B. Hierarchy communication. Which of the following will
C. Esprit d corps provide this information?
D. Unity of direction A. Organizational structure
27. He emphasizes to the team that they need B. Policy
to put their efforts together towards the C. Job description
attainment of the goals of the program. D. Manual of procedures
Which of the following principles refers to 33. Stephanie is often seen interacting with
this? the medical intern during coffee breaks and
A. Span of control after duty hours. What type of organizational
B. Unity of direction structure is this?
C. Unity of command A. Formal
D. Command responsibility B. Informal
28. Joey stresses the importance of C. Staff
promoting esprit d corps among the D. Line
members of the unit. Which of the following 34. She takes pride in saying that the
remarks of the staff indicates that they hospital has a decentralized structure. Which
understand what he pointed out? of the following is NOT compatible with this
A. Lets work together in harmony; we need to type of model?
be supportive of one another A. Flat organization
B. In order that we achieve the same results; B. Participatory approach
we must all follow the directives of Julius and not C. Shared governance
from other managers. D. Tall organization
C. We will ensure that all the resources we 35. Centralized organizations have some
need are available when needed. advantages. Which of the following
D. We need to put our efforts together in order statements are TRUE?
to raise the bar of excellence in the care we 1. Highly cost-effective
provide to all our patients. 2. Makes management easier
29. He discusses the goal of the department. 3. Reflects the interest of the worker
Which of the following statements is a goal? 4. Allows quick decisions or actions.
A. Increase the patient satisfaction rate A. 1 & 2
B. Eliminate the incidence of delayed B. 2 & 4
administration of medications C. 2, 3& 4
D. 1, 2, & 4 B. Disregard what she feels and continue to
36. Stephanie delegates effectively if she has work independently
authority to act, which is BEST defined as: C. Seek help from the Director of Nursing
A. having responsibility to direct others D. Quit her job and look for another
B. being accountable to the organization employment.
C. having legitimate right to act 2. As a young manager, she knows that
D. telling others what to do conflict occurs in any organization. Which of
37. Regardless of the size of a work group, the following statements regarding conflict is
enough staff must be available at all times to NOT true?
accomplish certain purposes. Which of these A. Can be destructive if the level is too high
purposes is NOT included? B. Is not beneficial; hence it should be prevented
A. Meet the needs of patients at all times
B. Provide a pair of hands to other units as C. May result in poor performance
needed D. May create leaders
C. Cover all time periods adequately. 3. Katherine tells one of the staff, I dont
D. Allow for growth and development of nursing have time to discuss the matter with you
staff. now. See me in my office later when the
38. Which of the following guidelines should latter asks if they can talk about an issue.
be least considered in formulating objectives Which of the following conflict resolution
for nursing care? strategies did she use?
A. Written nursing care plan A. Smoothing
B. Holistic approach B. Compromise
C. Prescribed standards C. Avoidance
D. Staff preferences D. Restriction
39. Stephanie considers shifting to 4. Kathleen knows that one of her staff is
transformational leadership. Which of the experiencing burnout. Which of the following
following statements best describes this is the best thing for her to do?
type of leadership? A. Advise her staff to go on vacation.
A. Uses visioning as the essence of leadership. B. Ignore her observations; it will be resolved
B. Serves the followers rather than being even without intervention
served. C. Remind her to show loyalty to the institution.
C. Maintains full trust and confidence in the D. Let the staff ventilate her feelings and ask
subordinates how she can be of help.
D. Possesses innate charisma that makes 5. She knows that performance appraisal
others feel good in his presence. consists of all the following activities
40. As a manager, she focuses her energy on EXCEPT:
both the quality of services rendered to the A. Setting specific standards and activities for
patients as well as the welfare of the staff of individual performance.
her unit. Which of the following management B. Using agency standards as a guide.
styles does she adopt? C. Determine areas of strength and weaknesses
A. Country club management D. Focusing activity on the correction of
B. Organization man management identified behavior.
C. Team management 6. Which of the following statements is NOT
D. Authority-obedience management true about performance appraisal?
A. Informing the staff about the specific
Nursing Leadership and Management 2 impressions of their work help improve their
1. Katherine is a young Unit Manager of the performance.
Pediatric Ward. Most of her staff nurses are B. A verbal appraisal is an acceptable substitute
senior to her, very articulate, confident and for a written report
sometimes aggressive. Katherine feels C. Patients are the best source of information
uncomfortable believing that she is the regarding personnel appraisal.
scapegoat of everything that goes wrong in D. The outcome of performance appraisal rests
her department. Which of the following is the primarily with the staff.
best action that she must take? 7. There are times when Katherine evaluates
A. Identify the source of the conflict and her staff as she makes her daily rounds.
understand the points of friction
Which of the following is NOT a benefit of C. Membership to accredited professional
conducting an informal appraisal? organization
A. The staff member is observed in natural D. Professional identification card
setting. 13. Which phase of the employment process
B. Incidental confrontation and collaboration is includes getting on the payroll and
allowed. completing documentary requirements?
C. The evaluation is focused on objective data A. Orientation
systematically. B. Induction
D. The evaluation may provide valid information C. Selection
for compilation of a formal report. D. Recruitment
8. She conducts a 6-month performance 14. She tries to design an organizational
review session with a staff member. Which of structure that allows communication to flow
the following actions is appropriate? in all directions and involve workers in
A. She asks another nurse to attest the session decision making. Which form of
as a witness. organizational structure is this?
B. She informs the staff that she may ask A. Centralized
another nurse to read the appraisal before the B. Decentralized
session is over. C. Matrix
C. She tells the staff that the session is D. Informal
manager-centered. 15. In a horizontal chart, the lowest level
D. The session is private between the two worker is located at the
members. A. Left most box
9. Alexandra is tasked to organize the new B. Middle
wing of the hospital. She was given the C. Right most box
authority to do as she deems fit. She is D. Bottom
aware that the director of nursing has 16. She decides to have a decentralized
substantial trust and confidence in her staffing system. Which of the following is an
capabilities, communicates through advantage of this system of staffing?
downward and upward channels and usually A. greater control of activities
uses the ideas and opinions of her staff. B. Conserves time
Which of the following is her style of C. Compatible with computerization
management? D. Promotes better interpersonal relationship
A. Benevolent authoritative 17. Aubrey thinks about primary nursing as a
B. Consultative system to deliver care. Which of the
C. Exploitive-authoritative following activities is NOT done by a primary
D. Participative nurse?
10. She decides to illustrate the A. Collaborates with the physician
organizational structure. Which of the B. Provides care to a group of patients together
following elements is NOT included? with a group of nurses
A. Level of authority C. Provides care for 5-6 patients during their
B. Lines of communication hospital stay.
C. Span of control D. Performs comprehensive initial assessment
D. Unity of direction 18. Which pattern of nursing care involves
11. She plans of assigning competent people the care given by a group of
to fill the roles designed in the hierarchy. paraprofessional workers led by a
Which process refers to this? professional nurse who take care of patients
A. Staffing with the same disease conditions and are
B. Scheduling located geographically near each other?
C. Recruitment A. Case method
D. Induction B. Modular nursing
12. She checks the documentary C. Nursing case management
requirements for the applicants for staff D. Team nursing
nurse position. Which one is NOT 19. St. Raphael Medical Center just opened
necessary? its new Performance Improvement
A. Certificate of previous employment Department. Ms. Valencia is appointed as the
B. Record of related learning experience (RLE) Quality Control Officer. She commits herself
to her new role and plans her strategies to 25. Ms. Valencia prepares the process
realize the goals and objectives of the standards. Which of the following is NOT a
department. Which of the following is a process standard?
primary task that they should perform to A. Initial assessment shall be done to all patients
have an effective control system? within twenty four hours upon admission.
A. Make an interpretation about strengths and B. Informed consent shall be secured prior to
weaknesses any invasive procedure
B. Identify the values of the department C. Patients reports 95% satisfaction rate prior to
C. Identify structure, process, outcome discharge from the hospital.
standards & criteria D. Patient education about their illness and
D. Measure actual performances treatment shall be provided for all patients and
20. Ms. Valencia develops the standards to their families.
be followed. Among the following standards, 26. Which of the following is evidence that
which is considered as a structure standard? the controlling process is effective?
A. The patients verbalized satisfaction of the A. The things that were planned are done
nursing care received B. Physicians do not complain.
B. Rotation of duty will be done every four C. Employees are contended
weeks for all patient care personnel. D. There is an increase in customer satisfaction
C. All patients shall have their weights taken rate.
recorded 27. Ms. Valencia is responsible to the
D. Patients shall answer the evaluation form number of personnel reporting to her. This
before discharge principle refers to:
21. When she presents the nursing A. Span of control
procedures to be followed, she refers to what B. Unity of command
type of standards? C. Carrot and stick principle
A. Process D. Esprit d corps
B. Outcome 28. She notes that there is an increasing
C. Structure unrest of the staff due to fatigue brought
D. Criteria about by shortage of staff. Which action is a
22. The following are basic steps in the priority?
controlling process of the department. Which A. Evaluate the overall result of the unrest
of the following is NOT included? B. Initiate a group interaction
A. Measure actual performance C. Develop a plan and implement it
B. Set nursing standards and criteria D. Identify external and internal forces.
C. Compare results of performance to standards 29. Kevin is a member of the Nursing
and objectives Research Council of the hospital. His first
D. Identify possible courses of action assignment is to determine the level of
23. Which of the following statements refers patient satisfaction on the care they received
to criteria? from the hospital. He plans to include all
A. Agreed on level of nursing care adult patients admitted from April to May,
B. Characteristics used to measure the level of with average length of stay of 3-4 days, first
nursing care admission, and with no complications.
C. Step-by-step guidelines Which of the following is an extraneous
D. Statement which guide the group in decision variable of the study?
making and problem solving A. Date of admission
24. She wants to ensure that every task is B. Length of stay
carried out as planned. Which of the C. Age of patients
following tasks is NOT included in the D. Absence of complications
controlling process? 30. He thinks of an appropriate theoretical
A. Instructing the members of the standards framework. Whose theory addresses the four
committee to prepare policies modes of adaptation?
B. Reviewing the existing policies of the hospital A. Martha Rogers
C. Evaluating the credentials of all nursing staff B. Sr. Callista Roy
D. Checking if activities conform to schedule C. Florence Nightingale
D. Jean Watson
31. He opts to use a self-report method. 38. He plans for his sampling method. Which
Which of the following is NOT TRUE about sampling method gives equal chance to all
this method? units in the population to get picked?
A. Most direct means of gathering information A. Random
B. Versatile in terms of content coverage B. Accidental
C. Most accurate and valid method of data C. Quota
gathering D. Judgment
D. Yields information that would be difficult to 39. Raphael is interested to learn more about
gather by another method transcultural nursing because he is assigned
32. Which of the following articles would at the family suites where most patients
Kevin least consider for his review of come from different cultures and countries.
literature? Which of the following designs is
A. Story-Telling and Anxiety Reduction Among appropriate for this study?
Pediatric Patients A. Grounded theory
B. Turnaround Time in Emergency Rooms B. Ethnography
C. Outcome Standards in Tertiary Health Care C. Case study
Institutions D. Phenomenology
D. Environmental Manipulation and Client 40. The nursing theorist who developed
Outcomes transcultural nursing theory is
33. Which of the following variables will he A. Dorothea Orem
likely EXCLUDE in his study? B. Madeleine Leininger
A. Competence of nurses C. Betty Newman
B. Caring attitude of nurses D. Sr. Callista Roy
C. Salary of nurses
D. Responsiveness of staff
34. He plans to use a Likert Scale to
determine
A. degree of agreement and disagreement
B. compliance to expected standards
C. level of satisfaction
D. degree of acceptance
35. He checks if his instruments meet the
criteria for evaluation. Which of the following
criteria refers to the consistency or the
ability to yield the same response upon its
repeated administration?
A. Validity
B. Reliability
C. Sensitivity
D. Objectivity
36. Which criteria refer to the ability of the
instrument to detect fine differences among
the subjects being studied?
A. Sensitivity
B. Reliability
C. Validity
D. Objectivity
37. Which of the following terms refer to the
degree to which an instrument measures
what it is supposed to be measure?
A. Validity
B. Reliability
C. Meaningfulness
D. Sensitivity

Potrebbero piacerti anche